264
This document consists of 20 printed pages. IB17 06_9702_11_VI_BLU/FP © UCLES 2017 [Turn over *09 1 6 9 6 2 2 1 3 * Cambridge International Examinations Cambridge International Advanced Subsidiary and Advanced Level PHYSICS 9702/11 Paper 1 Multiple Choice May/June 2017 1 hour 15 minutes Additional Materials: Multiple Choice Answer Sheet Soft clean eraser Soft pencil (type B or HB is recommended) READ THESE INSTRUCTIONS FIRST Write in soft pencil. Do not use staples, paper clips, glue or correction fluid. Write your name, Centre number and candidate number on the Answer Sheet in the spaces provided unless this has been done for you. DO NOT WRITE IN ANY BARCODES. There are forty questions on this paper. Answer all questions. For each question there are four possible answers A, B, C and D. Choose the one you consider correct and record your choice in soft pencil on the separate Answer Sheet. Read the instructions on the Answer Sheet very carefully. Each correct answer will score one mark. A mark will not be deducted for a wrong answer. Any working should be done in this booklet. Electronic calculators may be used.

PHYSICS 9702/11 READ THESE INSTRUCTIONS FIRSTmaxpapers.com/wp-content/uploads/2012/11/9702_s17… ·  · 2017-10-17PHYSICS 9702/11 Paper 1 Multiple Choice May/June 2017 1 hour 15

  • Upload
    vutruc

  • View
    351

  • Download
    1

Embed Size (px)

Citation preview

Page 1: PHYSICS 9702/11 READ THESE INSTRUCTIONS FIRSTmaxpapers.com/wp-content/uploads/2012/11/9702_s17… ·  · 2017-10-17PHYSICS 9702/11 Paper 1 Multiple Choice May/June 2017 1 hour 15

This document consists of 20 printed pages.

IB17 06_9702_11_VI_BLU/FP © UCLES 2017 [Turn over

*0916962213*

Cambridge International Examinations Cambridge International Advanced Subsidiary and Advanced Level

PHYSICS 9702/11

Paper 1 Multiple Choice May/June 2017

1 hour 15 minutes

Additional Materials: Multiple Choice Answer Sheet Soft clean eraser Soft pencil (type B or HB is recommended)

READ THESE INSTRUCTIONS FIRST Write in soft pencil. Do not use staples, paper clips, glue or correction fluid. Write your name, Centre number and candidate number on the Answer Sheet in the spaces provided unless this has been done for you. DO NOT WRITE IN ANY BARCODES. There are forty questions on this paper. Answer all questions. For each question there are four possible answers A, B, C and D. Choose the one you consider correct and record your choice in soft pencil on the separate Answer Sheet. Read the instructions on the Answer Sheet very carefully. Each correct answer will score one mark. A mark will not be deducted for a wrong answer. Any working should be done in this booklet. Electronic calculators may be used.

Page 2: PHYSICS 9702/11 READ THESE INSTRUCTIONS FIRSTmaxpapers.com/wp-content/uploads/2012/11/9702_s17… ·  · 2017-10-17PHYSICS 9702/11 Paper 1 Multiple Choice May/June 2017 1 hour 15

2

© UCLES 2017 9702/11/M/J/17

Data

speed of light in free space c = 3.00 × 108 m s–1

permeability of free space µ0 = 4π × 10–7 H m–1

permittivity of free space ε0 = 8.85 × 10–12 F m–1

(04

1επ

= 8.99 × 109 m F–1)

elementary charge e = 1.60 × 10–19 C

the Planck constant h = 6.63 × 10–34 J s

unified atomic mass unit 1 u = 1.66 × 10–27 kg

rest mass of electron me = 9.11 × 10–31 kg

rest mass of proton mp = 1.67 × 10–27 kg

molar gas constant R = 8.31 J K–1 mol–1

the Avogadro constant NA = 6.02 × 1023 mol–1

the Boltzmann constant k = 1.38 × 10–23 J K–1

gravitational constant G = 6.67 × 10–11 N m2

kg–2

acceleration of free fall g = 9.81 m s–2

Page 3: PHYSICS 9702/11 READ THESE INSTRUCTIONS FIRSTmaxpapers.com/wp-content/uploads/2012/11/9702_s17… ·  · 2017-10-17PHYSICS 9702/11 Paper 1 Multiple Choice May/June 2017 1 hour 15

3

© UCLES 2017 9702/11/M/J/17 [Turn over

Formulae

uniformly accelerated motion s = ut + 221 at

v 2 = u

2 + 2as

work done on/by a gas W = p∆V

gravitational potential φ = – rGm

hydrostatic pressure p = ρ gh

pressure of an ideal gas p = VNm

31 <c

2>

simple harmonic motion a = – ω 2x

velocity of particle in s.h.m. v = v0 cos ωt

v = ± ω )( 220 xx −

Doppler effect of = s

s

vvvf±

electric potential V = rQ

04 επ

capacitors in series 1 / C = 1 / C1 + 1 / C2 + . . .

capacitors in parallel C = C1 + C2 + . . .

energy of charged capacitor W = QV21

electric current I = Anvq

resistors in series R = R1 + R2 + . . .

resistors in parallel 1 / R = 1 / R1 + 1 / R2 + . . .

Hall voltage VH = ntqBI

alternating current/voltage x = x0 sin ωt

radioactive decay x = x0 exp(–λt)

decay constant λ = 21

0.693t

Page 4: PHYSICS 9702/11 READ THESE INSTRUCTIONS FIRSTmaxpapers.com/wp-content/uploads/2012/11/9702_s17… ·  · 2017-10-17PHYSICS 9702/11 Paper 1 Multiple Choice May/June 2017 1 hour 15

4

© UCLES 2017 9702/11/M/J/17

1 A student creates a table to show reasonable estimates of some physical quantities.

Which row is not a reasonable estimate?

quantity value

A current in a fan heater 12 A

B mass of an adult person 70 kg

C speed of an Olympic sprint runner 10 m s–1

D water pressure at the bottom of a garden pond 106 Pa

2 A particle travels in a straight line with speed v.

The particle slows down and changes direction. The new speed of the particle is 2v .

The new velocity has a component of 4v in the same direction as the initial path of the particle.

Through which angle has the particle turned?

A 27° B 30° C 45° D 60° 3 The speed v of a liquid leaving a tube depends on the change in pressure ∆P and the density ρ of

the liquid. The speed is given by the equation

v = n

Pk

∆ρ

where k is a constant that has no units.

What is the value of n ?

A 21 B 1 C

23 D 2

4 The values of displacement, velocity and acceleration of a vehicle can be deduced from graphs

representing its motion. Often the areas under these graphs, or the gradients of the graphs, are used.

What would not give a value for a displacement, a velocity or an acceleration?

A area under a velocity-time graph

B gradient of a displacement-time graph

C gradient of a velocity-time graph

D gradient of an acceleration-time graph

Page 5: PHYSICS 9702/11 READ THESE INSTRUCTIONS FIRSTmaxpapers.com/wp-content/uploads/2012/11/9702_s17… ·  · 2017-10-17PHYSICS 9702/11 Paper 1 Multiple Choice May/June 2017 1 hour 15

5

© UCLES 2017 9702/11/M/J/17 [Turn over

5 A ball is released from rest above a hard, horizontal surface. The graph shows how the velocity of the bouncing ball varies with time.

At which point on the graph does the ball reach its maximum height after the first bounce?

00

A

B

C

Dvelocity

time

6 A ball is kicked upwards at an angle of 45° to horizontal ground. After a short flight, the ball

returns to the ground.

It may be assumed that air resistance is negligible.

What is never zero during the flight of the ball?

A the horizontal component of the ball’s acceleration

B the horizontal component of the ball’s velocity

C the vertical component of the ball’s momentum

D the vertical component of the ball’s velocity 7 The mass of a rocket-propelled truck is approximately equal to the mass of the fuel in its tank.

The fuel is ignited and the truck is propelled along horizontal tracks by a constant force. The effect of air resistance is negligible.

During a test run the fuel is consumed at a constant rate.

Which statement describes the acceleration of the truck during the test run?

A The acceleration of the truck decreases as the fuel is consumed.

B The acceleration of the truck increases as the fuel is consumed.

C The acceleration of the truck remains constant.

D The acceleration of the truck is zero and the truck moves at a constant velocity.

Page 6: PHYSICS 9702/11 READ THESE INSTRUCTIONS FIRSTmaxpapers.com/wp-content/uploads/2012/11/9702_s17… ·  · 2017-10-17PHYSICS 9702/11 Paper 1 Multiple Choice May/June 2017 1 hour 15

6

© UCLES 2017 9702/11/M/J/17

8 An object is dropped at time t = 0 from a high building. Air resistance is significant.

Three graphs are plotted against time.

the height of the object above the ground

the speed of the object

the magnitude of the resultant force on the object

00

X

time

00

Y

time

00

Z

time

What are the quantities X, Y and Z?

height of the object above the ground speed of the object

magnitude of the resultant force

on the object

A X Y Z

B X Z Y

C Y Z X

D Z Y X

Page 7: PHYSICS 9702/11 READ THESE INSTRUCTIONS FIRSTmaxpapers.com/wp-content/uploads/2012/11/9702_s17… ·  · 2017-10-17PHYSICS 9702/11 Paper 1 Multiple Choice May/June 2017 1 hour 15

7

© UCLES 2017 9702/11/M/J/17 [Turn over

9 A student attempts to find the density ρ of aluminium by taking measurements of a rectangular sheet.

mass m = 51.6 ± 0.1 g

length l = 100.0 ± 0.1 cm

width w = 10.0 ± 0.1 cm

thickness t = 0.20 ± 0.01 mm

He uses the equation ρ = tw

m l

to calculate the density.

What is the calculated value of density with its uncertainty?

A 0.26 ± 0.01 g cm–3

B 0.26 ± 0.02 g cm–3

C 2.6 ± 0.1 g cm–3

D 2.6 ± 0.2 g cm–3 10 The graph shows how the momentum of a motorcycle changes with time.

5000

00 10time / s

momentum/ kg m s–1

What is the resultant force on the motorcycle?

A 500 N B 5000 N C 25 000 N D 50 000 N

Page 8: PHYSICS 9702/11 READ THESE INSTRUCTIONS FIRSTmaxpapers.com/wp-content/uploads/2012/11/9702_s17… ·  · 2017-10-17PHYSICS 9702/11 Paper 1 Multiple Choice May/June 2017 1 hour 15

8

© UCLES 2017 9702/11/M/J/17

11 A particle with mass and charge is moving from left to right in a uniform gravitational field and a uniform electric field. The gravitational field is downwards. The gravitational force and the electric force on this particle act in opposite directions.

What could be the sign of the charge on the particle and the direction of the electric field?

sign of charge direction of electric field

A negative down

B negative up

C positive left

D positive right 12 A sphere is acted upon by various forces, all of the same magnitude.

Which system of forces provides a resultant torque but zero resultant force on the sphere?

A B C D

13 A uniform horizontal footbridge is 12 m long and weighs 4000 N. It rests on two supports X and Y

as shown.

supportX

supportY

12 m

4 m

A man of weight 600 N is a distance of 4 m from support X.

What is the upward force on the footbridge from support X?

A 2200 N B 2300 N C 2400 N D 2600 N

Page 9: PHYSICS 9702/11 READ THESE INSTRUCTIONS FIRSTmaxpapers.com/wp-content/uploads/2012/11/9702_s17… ·  · 2017-10-17PHYSICS 9702/11 Paper 1 Multiple Choice May/June 2017 1 hour 15

9

© UCLES 2017 9702/11/M/J/17 [Turn over

14 A metal block has a mass of 750 g. 60% of the mass is magnesium and the remainder is copper.

The density of magnesium is 1.7 g cm–3.

The density of copper is 9.0 g cm–3.

What is the density of the block?

A 2.5 g cm–3 B 4.6 g cm–3 C 5.4 g cm–3 D 10.7 g cm–3 15 A man climbs slowly at a steady speed to the top of a ladder.

What is the main energy transfer taking place for the man as he climbs?

A chemical potential to gravitational potential

B chemical potential to kinetic

C kinetic to gravitational potential

D thermal (heat) to kinetic 16 During an interval of time, fuel supplies energy X to a car.

Some of this energy is converted into kinetic energy as the car accelerates.

The rest of the energy Y is lost as thermal energy.

What is the efficiency of the car?

A YX

X−

B YX

Y−

C X

YX − D Y

YX −

17 A railway engine accelerates a train of total mass 800 tonnes (1 tonne = 1000 kg) from rest to a

speed of 50 m s–1.

How much useful work must be done on the train to reach this speed?

A 1.0 × 106 J B 2.0 × 106

J C 1.0 × 109 J D 2.0 × 109

J 18 A mass is raised vertically. In time t, the increase in its gravitational potential energy is Ep and the

increase in its kinetic energy is Ek.

What is the average power input to the mass?

A (Ep – Ek)t B (Ep + Ek)t C t

EE kp − D t

EE kp +

Page 10: PHYSICS 9702/11 READ THESE INSTRUCTIONS FIRSTmaxpapers.com/wp-content/uploads/2012/11/9702_s17… ·  · 2017-10-17PHYSICS 9702/11 Paper 1 Multiple Choice May/June 2017 1 hour 15

10

© UCLES 2017 9702/11/M/J/17

19 Water flows from a lake into a turbine that is a vertical distance of 90 m below the lake, as shown.

turbine

lake

90 m

The mass flow rate of the water is 2400 kg min–1. The turbine has an efficiency of 75%.

What is the output power of the turbine?

A 26 kW B 35 kW C 1.6 MW D 2.1 MW 20 A wire of diameter d and length l hangs vertically from a fixed point. The wire is extended by

hanging a mass M on its end. The Young modulus of the wire is E. The acceleration of free fall is g.

Which equation is used to determine the extension x of the wire?

A x = Ed

M2πl B x =

EdMg

2πl C x =

dEMgπ

l4 D x = Ed

Mg2

l

Page 11: PHYSICS 9702/11 READ THESE INSTRUCTIONS FIRSTmaxpapers.com/wp-content/uploads/2012/11/9702_s17… ·  · 2017-10-17PHYSICS 9702/11 Paper 1 Multiple Choice May/June 2017 1 hour 15

11

© UCLES 2017 9702/11/M/J/17 [Turn over

21 The variation of the compression of a spring with the force applied to it is shown in the graph.

5.0

4.0

3.0

2.0

1.0

0

compression/ cm

0 2.0 4.0 6.0

force / N

8.0 10.0

A block slides along a horizontal frictionless surface towards the spring, as shown.

block

spring

The block is brought to rest by the spring. When the spring reaches a compression of 4.0 cm, all of the kinetic energy of the block is transferred to the elastic potential energy of the spring.

What is the kinetic energy of the block when it first makes contact with the spring?

A 0.16 J B 0.32 J C 16 J D 32 J 22 A longitudinal wave travels through a long spring. The spring is shown at one instant.

What is the wavelength of the wave?

A

B

C

D

spring

Page 12: PHYSICS 9702/11 READ THESE INSTRUCTIONS FIRSTmaxpapers.com/wp-content/uploads/2012/11/9702_s17… ·  · 2017-10-17PHYSICS 9702/11 Paper 1 Multiple Choice May/June 2017 1 hour 15

12

© UCLES 2017 9702/11/M/J/17

23 A sound wave has a frequency of 2500 Hz and a speed of 1500 m s–1.

What is the shortest distance from a point of maximum pressure in the wave to a point of minimum pressure?

A 0.15 m B 0.30 m C 0.60 m D 1.20 m 24 A sound wave is displayed on the screen of a cathode-ray oscilloscope (c.r.o.) as shown.

1 cm

The time-base of the c.r.o. is set at 2.5 ms cm–1.

What is the frequency of the sound wave?

A 50 Hz B 100 Hz C 200 Hz D 400 Hz 25 A car travelling in a straight line at a speed of 30 m s–1 passes near a stationary observer while

sounding its horn. The true frequency of sound from the horn is 400 Hz.

The speed of sound in air is 336 m s–1.

What is the change in the frequency of the sound heard by the observer as the car passes?

A 39 Hz B 66 Hz C 72 Hz D 78 Hz 26 Which list shows electromagnetic waves in order of increasing frequency?

A radio waves → gamma rays → ultraviolet → infra-red

B radio waves → infra-red → ultraviolet → gamma rays

C ultraviolet → gamma rays → radio waves → infra-red

D ultraviolet → infra-red → radio waves → gamma rays

Page 13: PHYSICS 9702/11 READ THESE INSTRUCTIONS FIRSTmaxpapers.com/wp-content/uploads/2012/11/9702_s17… ·  · 2017-10-17PHYSICS 9702/11 Paper 1 Multiple Choice May/June 2017 1 hour 15

13

© UCLES 2017 9702/11/M/J/17 [Turn over

27 The diagram shows a steel wire clamped at one end. The other end is attached to a weight hanging over a pulley.

weight

fixed stand fixed supportvibrator

A vibrator is attached to the wire near the clamped end. A stationary wave with one loop is produced. The frequency of the vibrator is f.

Which frequency should be used to produce a stationary wave with two loops?

A 4f B

2f C 2f D 4f

28 A parallel beam of light of wavelength 600 nm is incident normally on a diffraction grating. The

grating has 300 lines per millimetre.

What is the total number of intensity maxima from the grating?

A 1 B 3 C 11 D 13 29 A pattern of interference fringes is produced using a red laser, a double slit and a screen. The

screen is 3.5 m from the double slit. The light from the laser has a wavelength of 640 nm.

The pattern of fringes is shown.

72 mm

not toscale

brightfringe

What is the separation of the slits?

A 1.2 × 10–4 m B 1.6 × 10–4

m C 3.1 × 10–5 m D 3.3 × 10–9

m

Page 14: PHYSICS 9702/11 READ THESE INSTRUCTIONS FIRSTmaxpapers.com/wp-content/uploads/2012/11/9702_s17… ·  · 2017-10-17PHYSICS 9702/11 Paper 1 Multiple Choice May/June 2017 1 hour 15

14

© UCLES 2017 9702/11/M/J/17

30 The diagram shows two points P and Q which lie 90° apart on a circle of radius r.

A positive point charge at the centre of the circle creates an electric field of magnitude E at both P and Q.

P

Q r

+

Which expression gives the work done in moving a unit positive charge from P to Q?

A 0 B E × r C E ×

π

2r D E × (πr )

31 The diagram shows two parallel horizontal metal plates. The top plate is positively charged and

the bottom plate is earthed.

+

liquid dropplates

A small charged liquid drop, midway between the plates, is held in equilibrium by the combination of its weight and the electric force acting on it.

The acceleration of free fall is g and the electric field strength is E.

What is the polarity of the charge on the drop, and the ratio of charge to mass of the drop?

polarity masscharge

A negative gE

B negative Eg

C positive gE

D positive Eg

Page 15: PHYSICS 9702/11 READ THESE INSTRUCTIONS FIRSTmaxpapers.com/wp-content/uploads/2012/11/9702_s17… ·  · 2017-10-17PHYSICS 9702/11 Paper 1 Multiple Choice May/June 2017 1 hour 15

15

© UCLES 2017 9702/11/M/J/17 [Turn over

32 The diagram shows the symbol for a wire carrying a current I.

I

What does this current represent?

A the charge flowing past a point in the wire per unit time

B the number of electrons flowing past a point in the wire per unit time

C the number of positive ions flowing past a point in the wire per unit time

D the number of protons flowing past a point in the wire per unit time 33 Which values of current and resistance will produce a rate of energy transfer of 16 J s–1?

current / A resistance / Ω

A 1 4

B 2 8

C 4 1

D 16 1 34 Which component has the I-V graph shown?

I

V0

0

A filament lamp

B metallic conductor at constant temperature

C resistor of fixed resistance

D semiconductor diode

Page 16: PHYSICS 9702/11 READ THESE INSTRUCTIONS FIRSTmaxpapers.com/wp-content/uploads/2012/11/9702_s17… ·  · 2017-10-17PHYSICS 9702/11 Paper 1 Multiple Choice May/June 2017 1 hour 15

16

© UCLES 2017 9702/11/M/J/17

35 The circuit shown includes a cell of constant internal resistance and an external resistor R.

R

cell

V

A

A student records the ammeter and voltmeter readings. She then connects a second identical external resistor in parallel with the first external resistor.

What happens to the ammeter reading and to the voltmeter reading?

ammeter reading voltmeter reading

A decreases decreases

B decreases stays the same

C increases decreases

D increases stays the same

Page 17: PHYSICS 9702/11 READ THESE INSTRUCTIONS FIRSTmaxpapers.com/wp-content/uploads/2012/11/9702_s17… ·  · 2017-10-17PHYSICS 9702/11 Paper 1 Multiple Choice May/June 2017 1 hour 15

17

© UCLES 2017 9702/11/M/J/17 [Turn over

36 A computer is used to detect the change of position of a switch.

To detect the change of position, the computer requires a potential difference (p.d.) of 0 V to its input at one switch position and a p.d. of between 5 V and 7 V at the other switch position.

For each of the circuits, assume the battery has negligible internal resistance.

Which circuit provides an input voltage to the computer that enables it to detect the change of position of the switch?

A

9 V

1 kΩ

1 kΩcomputer

inputvoltage

C

9 V

1 kΩ

2 kΩcomputer

inputvoltage

B

9 V

2 kΩ

500 Ωcomputer

inputvoltage

D

9 V

500 Ω

2 kΩcomputer

inputvoltage

Page 18: PHYSICS 9702/11 READ THESE INSTRUCTIONS FIRSTmaxpapers.com/wp-content/uploads/2012/11/9702_s17… ·  · 2017-10-17PHYSICS 9702/11 Paper 1 Multiple Choice May/June 2017 1 hour 15

18

© UCLES 2017 9702/11/M/J/17

37 In the circuit shown, the battery and ammeter have negligible resistance.

12 V

A X Y

The following combinations of resistors are each separately placed between the terminals X and Y of the circuit.

Which combination would give an ammeter reading of 8 A?

3 Ω

2 Ω

1 Ω

A

3 Ω

1 Ω

2 Ω

B

3 Ω

2 Ω1 Ω

C

3 Ω

2 Ω

1 Ω

D

Page 19: PHYSICS 9702/11 READ THESE INSTRUCTIONS FIRSTmaxpapers.com/wp-content/uploads/2012/11/9702_s17… ·  · 2017-10-17PHYSICS 9702/11 Paper 1 Multiple Choice May/June 2017 1 hour 15

19

© UCLES 2017 9702/11/M/J/17 [Turn over

38 The table lists the nucleon number and the proton number of various nuclei. The nuclei are represented by the letters L to T.

nucleus nucleon number

proton number

L 227 89

M 226 89

N 225 89

O 227 90

P 226 90

Q 225 90

R 227 91

S 226 91

T 225 91

Which row in the following table correctly shows three nuclei of the same element, and three nuclei that have the same number of neutrons?

same element same number of neutrons

A L M N R P N

B M P S R S T

C O P Q M P S

D R P N O P Q

Page 20: PHYSICS 9702/11 READ THESE INSTRUCTIONS FIRSTmaxpapers.com/wp-content/uploads/2012/11/9702_s17… ·  · 2017-10-17PHYSICS 9702/11 Paper 1 Multiple Choice May/June 2017 1 hour 15

20

Permission to reproduce items where third-party owned material protected by copyright is included has been sought and cleared where possible. Every reasonable effort has been made by the publisher (UCLES) to trace copyright holders, but if any items requiring clearance have unwittingly been included, the publisher will be pleased to make amends at the earliest possible opportunity. To avoid the issue of disclosure of answer-related information to candidates, all copyright acknowledgements are reproduced online in the Cambridge International Examinations Copyright Acknowledgements Booklet. This is produced for each series of examinations and is freely available to download at www.cie.org.uk after the live examination series. Cambridge International Examinations is part of the Cambridge Assessment Group. Cambridge Assessment is the brand name of University of Cambridge Local Examinations Syndicate (UCLES), which is itself a department of the University of Cambridge. © UCLES 2017 9702/11/M/J/17

39 A radioactive nucleus is formed by β– decay. This nucleus then decays by α-emission.

Which graph of nucleon number N plotted against proton number Z shows the β– decay followed by the α-emission?

88 90 92 94 230

232

234

236 N

Z

A

88 90 92 94 230

232

234

236 N

Z

B

88 90 92 94 230

232

234

236 N

Z

C

88 90 92 94 230

232

234

236 N

Z

D

40 What are the structures of the proton and of the neutron in terms of quarks?

proton neutron

up quark down quark up quark down quark

A 1 1 2 2

B 1 2 2 1

C 2 1 1 2

D 2 2 1 1

Page 21: PHYSICS 9702/11 READ THESE INSTRUCTIONS FIRSTmaxpapers.com/wp-content/uploads/2012/11/9702_s17… ·  · 2017-10-17PHYSICS 9702/11 Paper 1 Multiple Choice May/June 2017 1 hour 15

This document consists of 18 printed pages and 2 blank pages.

IB17 06_9702_12/2RP © UCLES 2017 [Turn over

*1315253710*

Cambridge International Examinations Cambridge International Advanced Subsidiary and Advanced Level

PHYSICS 9702/12

Paper 1 Multiple Choice May/June 2017

1 hour 15 minutes

Additional Materials: Multiple Choice Answer Sheet Soft clean eraser Soft pencil (type B or HB is recommended)

READ THESE INSTRUCTIONS FIRST

Write in soft pencil.

Do not use staples, paper clips, glue or correction fluid.

Write your name, Centre number and candidate number on the Answer Sheet in the spaces provided unless this has been done for you.

DO NOT WRITE IN ANY BARCODES.

There are forty questions on this paper. Answer all questions. For each question there are four possible answers A, B, C and D.

Choose the one you consider correct and record your choice in soft pencil on the separate Answer Sheet.

Read the instructions on the Answer Sheet very carefully.

Each correct answer will score one mark. A mark will not be deducted for a wrong answer.

Any working should be done in this booklet.

Electronic calculators may be used.

Page 22: PHYSICS 9702/11 READ THESE INSTRUCTIONS FIRSTmaxpapers.com/wp-content/uploads/2012/11/9702_s17… ·  · 2017-10-17PHYSICS 9702/11 Paper 1 Multiple Choice May/June 2017 1 hour 15

2

© UCLES 2017 9702/12/M/J/17

Data

speed of light in free space c = 3.00 × 108 m s–1

permeability of free space µ0 = 4π × 10–7

H m–1

permittivity of free space ε0 = 8.85 × 10–12

F m–1

(0

4

1

επ

= 8.99 × 109 m F–1)

elementary charge e = 1.60 × 10–19 C

the Planck constant h = 6.63 × 10–34 J s

unified atomic mass unit 1 u = 1.66 × 10–27 kg

rest mass of electron me = 9.11 × 10–31 kg

rest mass of proton mp = 1.67 × 10–27 kg

molar gas constant R = 8.31 J K–1 mol–1

the Avogadro constant NA = 6.02 × 1023 mol–1

the Boltzmann constant k = 1.38 × 10–23 J K–1

gravitational constant G = 6.67 × 10–11 N m2

kg–2

acceleration of free fall g = 9.81 m s–2

Page 23: PHYSICS 9702/11 READ THESE INSTRUCTIONS FIRSTmaxpapers.com/wp-content/uploads/2012/11/9702_s17… ·  · 2017-10-17PHYSICS 9702/11 Paper 1 Multiple Choice May/June 2017 1 hour 15

3

© UCLES 2017 9702/12/M/J/17 [Turn over

Formulae

uniformly accelerated motion s = ut + 2

2

1at

v

2 = u

2 + 2as

work done on/by a gas W = p∆V

gravitational potential φ = –

r

Gm

hydrostatic pressure p = ρ gh

pressure of an ideal gas p = V

Nm3

1 <c

2>

simple harmonic motion a = – ω

2x

velocity of particle in s.h.m. v = v0 cos ωt

v = ± ω )( 22

0xx −

Doppler effect of =

s

s

vv

vf

±

electric potential V = r

Q

04 επ

capacitors in series 1 / C = 1 / C1 + 1 / C2 + . . .

capacitors in parallel C = C1 + C2 + . . .

energy of charged capacitor W = QV2

1

electric current I = Anvq

resistors in series R = R1 + R2 + . . .

resistors in parallel 1 / R = 1 / R1 + 1 / R2 + . . .

Hall voltage VH = ntq

BI

alternating current/voltage x = x0 sin ωt

radioactive decay x = x0 exp(–λt)

decay constant λ =

2

1

0.693

t

Page 24: PHYSICS 9702/11 READ THESE INSTRUCTIONS FIRSTmaxpapers.com/wp-content/uploads/2012/11/9702_s17… ·  · 2017-10-17PHYSICS 9702/11 Paper 1 Multiple Choice May/June 2017 1 hour 15

4

© UCLES 2017 9702/12/M/J/17

1 What is the approximate average speed of a winning female Olympic athlete running a 100 m race?

A 6 m s–1 B 9 m s–1 C 12 m s–1 D 15 m s–1 2 Two forces act on a circular disc as shown.

3 N

4 N

Which diagram shows the line of action of the resultant force?

5 N

A5 N

B

5 N

C D

5 N

3 What correctly expresses the volt in terms of SI base units?

A A Ω

B W A–1

C kg m2 s–1

A–1

D kg m2 s–3

A–1

Page 25: PHYSICS 9702/11 READ THESE INSTRUCTIONS FIRSTmaxpapers.com/wp-content/uploads/2012/11/9702_s17… ·  · 2017-10-17PHYSICS 9702/11 Paper 1 Multiple Choice May/June 2017 1 hour 15

5

© UCLES 2017 9702/12/M/J/17 [Turn over

4 The current in a block of semiconductor is 30.0 mA when there is a potential difference (p.d.) of 10.0 V across it. The dimensions of the block and the direction of the current in it are as shown.

30.0 mA

30.0 mm

15.0 mm

15.0 mm

The electrical meters used are accurate to ± 0.1 mA and ± 0.1 V. The dimensions of the block are

accurate to ± 0.2 mm.

What is the resistivity of the semiconductor?

A 10.0 ± 0.2 Ω m

B 10.0 ± 0.3 Ω m

C 10.0 ± 0.5 Ω m

D 10.0 ± 0.8 Ω m 5 The diameter of a cylindrical metal rod is measured using a micrometer screw gauge.

The diagram below shows an enlargement of the scale on the micrometer screw gauge when taking the measurement.

40

32

0.5 mm / rev30

What is the cross-sectional area of the rod?

A 3.81 mm2 B 11.4 mm2 C 22.8 mm2 D 45.6 mm2

Page 26: PHYSICS 9702/11 READ THESE INSTRUCTIONS FIRSTmaxpapers.com/wp-content/uploads/2012/11/9702_s17… ·  · 2017-10-17PHYSICS 9702/11 Paper 1 Multiple Choice May/June 2017 1 hour 15

6

© UCLES 2017 9702/12/M/J/17

6 A ball is set in motion at P on a frictionless surface. It moves up slope PQ, along the horizontal surface QR and finally descends slope RS.

P S

Q R

Which graph could represent the variation with time t of the ball’s speed v as the ball moves from P to S?

00

v

t

A

00

v

t

B

00

v

t

C

00

v

t

D

7 A rubber ball is dropped onto a table and bounces back up. The table exerts a force F on the ball.

Which graph best shows the variation with time t of the force F for the short time that the ball is in contact with the table?

00

F

t

A

F

t

B

F

t

C

F

t

D

00

00

00

Page 27: PHYSICS 9702/11 READ THESE INSTRUCTIONS FIRSTmaxpapers.com/wp-content/uploads/2012/11/9702_s17… ·  · 2017-10-17PHYSICS 9702/11 Paper 1 Multiple Choice May/June 2017 1 hour 15

7

© UCLES 2017 9702/12/M/J/17 [Turn over

8 A golf ball of mass m is dropped onto a hard surface from a height h1 and rebounds to a height h2.

The momentum of the golf ball just as it reaches the surface is different from its momentum just as it leaves the surface.

What is the total change in the momentum of the golf ball between these two instants? (Ignore air resistance.)

A 12ghm – 2

2ghm

B 12ghm +

22ghm

C )(221

hhgm −

D )(221

hhgm +

9 A book of weight W is at rest on a table. A student attempts to state Newton’s third law of motion

by saying that ‘action equals reaction’.

W

book

table

If the weight of the book is the ‘action’ force, what is the ‘reaction’ force?

A the force W acting downwards on the Earth from the table

B the force W acting upwards on the book from the table

C the force W acting upwards on the Earth from the book

D the force W acting upwards on the table from the floor

Page 28: PHYSICS 9702/11 READ THESE INSTRUCTIONS FIRSTmaxpapers.com/wp-content/uploads/2012/11/9702_s17… ·  · 2017-10-17PHYSICS 9702/11 Paper 1 Multiple Choice May/June 2017 1 hour 15

8

© UCLES 2017 9702/12/M/J/17

10 A metal cylinder is suspended vertically in equilibrium by a cord. The diagram shows the cylinder in four different positions P, Q, R and S.

R

S

Q

P

T T T T

water

metal cylinder

Which statement explains the variation of the tension T in the cord?

A At P and at Q, the tension T in the cord is the same because the difference in pressure between the top and bottom of the cylinder is the same.

B At Q, the tension T in the cord is less than at P because, at smaller depth, liquid pressure is smaller.

C At R, the tension T in the cord is less than at P because atmospheric pressure is less than water pressure.

D At S, the tension T in the cord is greater than at P because atmospheric pressure at S exerts no force on the top or bottom of the cylinder.

Page 29: PHYSICS 9702/11 READ THESE INSTRUCTIONS FIRSTmaxpapers.com/wp-content/uploads/2012/11/9702_s17… ·  · 2017-10-17PHYSICS 9702/11 Paper 1 Multiple Choice May/June 2017 1 hour 15

9

© UCLES 2017 9702/12/M/J/17 [Turn over

11 In a machine, many couples act on a rotating object as shown.

16 N

16 N

30 N

30 N

6.5 cm

25 N

25 N

50 N

50 N

4.0 cm

12 cm

10 cm

What is the resultant torque acting on the rotating object?

A 4.7 N m B 8.6 N m C 9.3 N m D 17.1 N m 12 A uniform beam is pivoted at P as shown. Weights of 10 N and 20 N are attached to its ends.

The length of the beam is marked at 0.1 m intervals. The weight of the beam is 100 N.

At which point should a further weight of 20 N be attached to achieve equilibrium?

A B C

P10 N

0.1 m

20 N

0.6 m 0.4 m

D

13 What are the SI base units of the quantity density

pressure?

A s–2 B kg2 s–2 C kg2

m2 s–2 D m2

s–2

Page 30: PHYSICS 9702/11 READ THESE INSTRUCTIONS FIRSTmaxpapers.com/wp-content/uploads/2012/11/9702_s17… ·  · 2017-10-17PHYSICS 9702/11 Paper 1 Multiple Choice May/June 2017 1 hour 15

10

© UCLES 2017 9702/12/M/J/17

14 Which quantities are conserved in an inelastic collision?

kinetic energy total energy linear momentum

A conserved not conserved conserved

B conserved not conserved not conserved

C not conserved conserved conserved

D not conserved conserved not conserved

15 A cyclist is travelling at a constant speed up a hill. The frictional force resisting the cyclist’s motion

is 8.0 N.

The cyclist uses 450 J of energy to travel a distance of 20 m.

What is the increase in the gravitational potential energy of the cyclist?

A 160 J B 290 J C 440 J D 610 J 16 A stone of mass m falls from rest at the top of a cliff of height h into the sea below. Just before

hitting the sea the stone has speed v.

What is the average force of air resistance acting on the stone during its fall?

A mg B h

ghvm )2( 2−

C

h

vgm

2

2

D

2

2vghm

17 A railway engine accelerates a train of total mass 1200 tonnes (1 tonne = 1000 kg) from rest to a

speed of 75 m s–1.

How much useful work must be done on the train to reach this speed?

A 3.4 × 106 J B 6.8 × 106

J C 3.4 × 109 J D 6.8 × 109

J

Page 31: PHYSICS 9702/11 READ THESE INSTRUCTIONS FIRSTmaxpapers.com/wp-content/uploads/2012/11/9702_s17… ·  · 2017-10-17PHYSICS 9702/11 Paper 1 Multiple Choice May/June 2017 1 hour 15

11

© UCLES 2017 9702/12/M/J/17 [Turn over

18 What is a correct derivation of the equation relating power, force and velocity?

A power = taken time

done work and work done = force × displacement

so power = taken time

ntdisplaceme force×

so power = force × velocity

B power = taken time

done work and work done = force × distance

so power = taken time

distance force ×

so power = force × velocity

C power = taken time

done work and work done = ntdisplaceme

force

so power = ntdisplaceme

force × time taken

so power = velocity

force

D power = taken time

done work and work done = distance

force

so power = distance

force × time taken

so power = velocity

force

19 A cable on a suspension bridge supports a weight of 19.3 × 105 N. This weight causes the cable

to stretch by 47 mm.

A lorry crossing the bridge then increases the force on the cable to 23.3 × 105 N. The

force-extension graph for the cable is shown.

00

47 57

23.3 × 105

19.3 × 105

extension / mm

force / N

What is the increase in strain energy in the cable when the lorry is crossing the bridge?

A 21 kJ B 23 kJ C 45 kJ D 66 kJ

Page 32: PHYSICS 9702/11 READ THESE INSTRUCTIONS FIRSTmaxpapers.com/wp-content/uploads/2012/11/9702_s17… ·  · 2017-10-17PHYSICS 9702/11 Paper 1 Multiple Choice May/June 2017 1 hour 15

12

© UCLES 2017 9702/12/M/J/17

20 What are the units of stress, strain and the Young modulus?

stress strain Young

modulus

A newton metre pascal

B newton no unit newton

C pascal metre newton

D pascal no unit pascal

21 A rubber band is stretched and then relaxed to its original length. The diagram shows the

force-extension graph for this process.

force

extension0

0O

P

R area X

area Y

e

Q

As the force is increased, the curve follows the path OPQ to extension e. As the force is reduced, the curve follows the path QRO to return to zero extension.

The area labelled X is between the curves OPQ and QRO. The area labelled Y is bounded by the curve QRO and the horizontal axis.

Which statement about the process is correct?

A Area X is the energy which heats the band as it is stretched to extension e.

B (Area X + area Y) is the minimum energy required to stretch the band to extension e.

C Area X is the elastic potential energy stored in the band when it is stretched to extension e.

D (Area Y – area X) is the net work done on the band during the process.

Page 33: PHYSICS 9702/11 READ THESE INSTRUCTIONS FIRSTmaxpapers.com/wp-content/uploads/2012/11/9702_s17… ·  · 2017-10-17PHYSICS 9702/11 Paper 1 Multiple Choice May/June 2017 1 hour 15

13

© UCLES 2017 9702/12/M/J/17 [Turn over

22 The period of an electromagnetic wave is 1.0 ns.

What are the frequency and wavelength of the wave?

frequency / Hz wavelength / m

A 1.0 3.0 × 108

B 1.0 × 106 300

C 1.0 × 109 0.30

D 1.0 × 1012 3.0 × 10–4

23 Which statement about progressive longitudinal waves is not correct?

A The oscillations of the particles are parallel to the direction of travel of the wave energy.

B They have a series of nodes and antinodes.

C They need a medium through which to travel.

D They transfer energy. 24 A bicycle gear wheel is a disc with 50 ‘teeth’ equally spaced around its edge, as shown. The gear

wheel is rotated 10 times each second. A springy strip of metal is vibrated by the rotating ‘teeth’. The metal strip produces a sound of frequency that is equal to the frequency of vibration of the strip.

gear wheel

teeth

fixed clamp

springy stripof metal

The speed of sound in air is 330 m s–1.

What is the wavelength of the emitted sound?

A 0.66 m B 1.5 m C 6.6 m D 500 m

Page 34: PHYSICS 9702/11 READ THESE INSTRUCTIONS FIRSTmaxpapers.com/wp-content/uploads/2012/11/9702_s17… ·  · 2017-10-17PHYSICS 9702/11 Paper 1 Multiple Choice May/June 2017 1 hour 15

14

© UCLES 2017 9702/12/M/J/17

25 An ambulance travels along a straight road at a speed of 30.0 m s–1. Its siren emits sound of frequency 2000 Hz. The speed of sound in the air is 340 m s–1. The ambulance passes a man standing at the side of the road.

What is the frequency of the sound heard by the man as the ambulance moves towards him and as the ambulance moves away from him?

frequency heard as ambulance moves towards man / Hz

frequency heard as ambulance moves away from man / Hz

A 1820 2180

B 1840 2190

C 2180 1820

D 2190 1840

26 Three different electromagnetic waves P, Q and R have the frequencies shown.

frequency / Hz

P 3 × 1010

Q 3 × 1013

R 6 × 1014

Which row identifies P, Q and R?

P Q R

A infra-red visible ultraviolet

B microwave infra-red visible

C ultraviolet X-ray gamma ray

D visible ultraviolet X-ray

27 Which row describes the oscillations of two moving particles in a stationary wave that are

separated by a distance of half a wavelength?

phase

difference amplitude

A 90° different

B 90° same

C 180° different

D 180° same

Page 35: PHYSICS 9702/11 READ THESE INSTRUCTIONS FIRSTmaxpapers.com/wp-content/uploads/2012/11/9702_s17… ·  · 2017-10-17PHYSICS 9702/11 Paper 1 Multiple Choice May/June 2017 1 hour 15

15

© UCLES 2017 9702/12/M/J/17 [Turn over

28 A parallel beam of red light of wavelength 700 nm is incident normally on a diffraction grating that has 400 lines per millimetre.

What is the total number of intensity maxima from the grating?

A 6 B 7 C 8 D 9

29 Two wave sources are oscillating in phase. Each source produces a wave of wavelength λ. The

two waves from the sources meet at point X with a phase difference of 90°.

What is a possible difference in the distances from the two wave sources to point X?

A 8

λ B

4

λ C

2

λ D λ

30 Which diagram best illustrates the electric field around a positive point charge?

+

A

+

B

+ +

C D

31 The path of an electron with initial speed v in the uniform electric field between two parallel plates

is shown.

xplates

The vertical deflection x is measured at the right-hand edge of the plates.

The distance between the plates is halved. The potential difference between the plates remains the same.

What will be the new deflection of the electron with the same initial speed v ?

A x B x2 C 2x D 4x

Page 36: PHYSICS 9702/11 READ THESE INSTRUCTIONS FIRSTmaxpapers.com/wp-content/uploads/2012/11/9702_s17… ·  · 2017-10-17PHYSICS 9702/11 Paper 1 Multiple Choice May/June 2017 1 hour 15

16

© UCLES 2017 9702/12/M/J/17

32 The current in a circuit component is 2.00 µA.

How many electrons pass through the component each second?

A 1.25 × 1013 B 1.25 × 1016 C 1.25 × 1019 D 1.25 × 1025 33 The filament of a 240 V, 100 W electric lamp heats up from room temperature to its operating

temperature. As it heats up, its resistance increases by a factor of 16.

What is the resistance of the filament at room temperature?

A 36 Ω B 580 Ω C 1.5 kΩ D 9.2 kΩ 34 Two wires have the same length and the same resistance. Wire X is made of a metal of resistivity

1.7 × 10–8 Ω m and wire Y is made of a metal of resistivity 5.6 × 10–8

Ω m.

The diameter of wire X is 0.315 mm.

What is the diameter of wire Y?

A 0.17 mm B 0.33 mm C 0.57 mm D 1.0 mm 35 A cell has a constant electromotive force.

A variable resistor is connected between the terminals of the cell.

The resistance of the variable resistor is decreased.

Which statement about the change of the cell’s terminal potential difference (p.d.) is correct?

A The terminal p.d. is decreased because more work is done moving unit charge through the internal resistance of the cell.

B The terminal p.d. is decreased because the current in the variable resistor is decreased.

C The terminal p.d. is increased because more work is done moving unit charge through the variable resistor.

D The terminal p.d. is increased because the current in the variable resistor is increased.

Page 37: PHYSICS 9702/11 READ THESE INSTRUCTIONS FIRSTmaxpapers.com/wp-content/uploads/2012/11/9702_s17… ·  · 2017-10-17PHYSICS 9702/11 Paper 1 Multiple Choice May/June 2017 1 hour 15

17

© UCLES 2017 9702/12/M/J/17 [Turn over

36 Four resistors are connected in a square as shown.

8 Ω 6 Ω

2 Ω 4 Ω

Q

S

RP

The resistance may be measured between any two junctions.

Between which two junctions is the measured resistance greatest?

A P and Q B Q and S C R and S D S and P 37 A circuit is set up as shown.

A

V V

P Q

The variable resistor is adjusted so that the ammeter reading decreases.

How do the readings of the voltmeters change?

reading on voltmeter P

reading on voltmeter Q

A decreases decreases

B decreases increases

C increases decreases

D increases increases

Page 38: PHYSICS 9702/11 READ THESE INSTRUCTIONS FIRSTmaxpapers.com/wp-content/uploads/2012/11/9702_s17… ·  · 2017-10-17PHYSICS 9702/11 Paper 1 Multiple Choice May/June 2017 1 hour 15

18

© UCLES 2017 9702/12/M/J/17

38 In a television programme to illustrate scientific models, a presenter fires a gun many times at a bale of hay. Two small cannon balls are embedded within the hay some distance apart from each other.

The hay bale measures approximately 2 m × 2 m × 2 m and the cannon balls are made of iron, approximately spherical, and about 5 cm in diameter.

What might the presenter be illustrating?

A α-particle scattering

B β– decay

C conservation of momentum

D double-slit interference 39 A certain nuclide, uranium-235, has nucleon number 235, proton number 92 and neutron number

143. Data on four other nuclides are given below.

Which nuclide is an isotope of uranium-235?

nucleon number proton number neutron number

A 235 91 144

B 236 92 144

C 237 94 143

D 238 95 143

40 During β– decay, which change takes place to the quark composition of the nucleus that emits the

β– particle, and which other particle is emitted?

quark change other particle

emitted

A down to up antineutrino

B down to up neutrino

C up to down antineutrino

D up to down neutrino

Page 39: PHYSICS 9702/11 READ THESE INSTRUCTIONS FIRSTmaxpapers.com/wp-content/uploads/2012/11/9702_s17… ·  · 2017-10-17PHYSICS 9702/11 Paper 1 Multiple Choice May/June 2017 1 hour 15

19

© UCLES 2017 9702/12/M/J/17

BLANK PAGE

Page 40: PHYSICS 9702/11 READ THESE INSTRUCTIONS FIRSTmaxpapers.com/wp-content/uploads/2012/11/9702_s17… ·  · 2017-10-17PHYSICS 9702/11 Paper 1 Multiple Choice May/June 2017 1 hour 15

20

Permission to reproduce items where third-party owned material protected by copyright is included has been sought and cleared where possible. Every reasonable effort has been made by the publisher (UCLES) to trace copyright holders, but if any items requiring clearance have unwittingly been included, the publisher will be pleased to make amends at the earliest possible opportunity. To avoid the issue of disclosure of answer-related information to candidates, all copyright acknowledgements are reproduced online in the Cambridge International Examinations Copyright Acknowledgements Booklet. This is produced for each series of examinations and is freely available to download at www.cie.org.uk after the live examination series.

Cambridge International Examinations is part of the Cambridge Assessment Group. Cambridge Assessment is the brand name of University of Cambridge Local Examinations Syndicate (UCLES), which is itself a department of the University of Cambridge.

© UCLES 2017 9702/12/M/J/17

BLANK PAGE

Page 41: PHYSICS 9702/11 READ THESE INSTRUCTIONS FIRSTmaxpapers.com/wp-content/uploads/2012/11/9702_s17… ·  · 2017-10-17PHYSICS 9702/11 Paper 1 Multiple Choice May/June 2017 1 hour 15

This document consists of 19 printed pages and 1 blank page.

IB17 06_9702_13/3RP © UCLES 2017 [Turn over

*2836591272*

Cambridge International Examinations Cambridge International Advanced Subsidiary and Advanced Level

PHYSICS 9702/13

Paper 1 Multiple Choice May/June 2017

1 hour 15 minutes

Additional Materials: Multiple Choice Answer Sheet Soft clean eraser Soft pencil (type B or HB is recommended)

READ THESE INSTRUCTIONS FIRST

Write in soft pencil.

Do not use staples, paper clips, glue or correction fluid.

Write your name, Centre number and candidate number on the Answer Sheet in the spaces provided unless this has been done for you.

DO NOT WRITE IN ANY BARCODES.

There are forty questions on this paper. Answer all questions. For each question there are four possible answers A, B, C and D.

Choose the one you consider correct and record your choice in soft pencil on the separate Answer Sheet.

Read the instructions on the Answer Sheet very carefully.

Each correct answer will score one mark. A mark will not be deducted for a wrong answer.

Any working should be done in this booklet.

Electronic calculators may be used.

Page 42: PHYSICS 9702/11 READ THESE INSTRUCTIONS FIRSTmaxpapers.com/wp-content/uploads/2012/11/9702_s17… ·  · 2017-10-17PHYSICS 9702/11 Paper 1 Multiple Choice May/June 2017 1 hour 15

2

© UCLES 2017 9702/13/M/J/17

Data

speed of light in free space c = 3.00 × 108 m s–1

permeability of free space µ0 = 4π × 10–7

H m–1

permittivity of free space ε0 = 8.85 × 10–12

F m–1

(0

4

1

επ

= 8.99 × 109 m F–1)

elementary charge e = 1.60 × 10–19 C

the Planck constant h = 6.63 × 10–34 J s

unified atomic mass unit 1 u = 1.66 × 10–27 kg

rest mass of electron me = 9.11 × 10–31 kg

rest mass of proton mp = 1.67 × 10–27 kg

molar gas constant R = 8.31 J K–1 mol–1

the Avogadro constant NA = 6.02 × 1023 mol–1

the Boltzmann constant k = 1.38 × 10–23 J K–1

gravitational constant G = 6.67 × 10–11 N m2

kg–2

acceleration of free fall g = 9.81 m s–2

Page 43: PHYSICS 9702/11 READ THESE INSTRUCTIONS FIRSTmaxpapers.com/wp-content/uploads/2012/11/9702_s17… ·  · 2017-10-17PHYSICS 9702/11 Paper 1 Multiple Choice May/June 2017 1 hour 15

3

© UCLES 2017 9702/13/M/J/17 [Turn over

Formulae

uniformly accelerated motion s = ut + 2

2

1at

v

2 = u

2 + 2as

work done on/by a gas W = p∆V

gravitational potential φ = –

r

Gm

hydrostatic pressure p = ρ gh

pressure of an ideal gas p = V

Nm3

1 <c

2>

simple harmonic motion a = – ω

2x

velocity of particle in s.h.m. v = v0 cos ωt

v = ± ω )( 22

0xx −

Doppler effect of =

s

s

vv

vf

±

electric potential V = r

Q

04 επ

capacitors in series 1 / C = 1 / C1 + 1 / C2 + . . .

capacitors in parallel C = C1 + C2 + . . .

energy of charged capacitor W = QV2

1

electric current I = Anvq

resistors in series R = R1 + R2 + . . .

resistors in parallel 1 / R = 1 / R1 + 1 / R2 + . . .

Hall voltage VH = ntq

BI

alternating current/voltage x = x0 sin ωt

radioactive decay x = x0 exp(–λt)

decay constant λ =

2

1

0.693

t

Page 44: PHYSICS 9702/11 READ THESE INSTRUCTIONS FIRSTmaxpapers.com/wp-content/uploads/2012/11/9702_s17… ·  · 2017-10-17PHYSICS 9702/11 Paper 1 Multiple Choice May/June 2017 1 hour 15

4

© UCLES 2017 9702/13/M/J/17

1 What is the best estimate of the kinetic energy of a family car travelling at 50 km h–1?

A 1.5 × 103 J B 1.5 × 105

J C 1.5 × 107 J D 1.5 × 109

J 2 The diagram shows two vectors X and Y. The vectors are perpendicular to one another.

X = 8.0 NY = 6.0 N

What is the magnitude and direction of vector (X – Y)?

A 10.0 N at an angle of 37° downwards from the direction of X

B 10.0 N at an angle of 37° upwards from the direction of X

C 14.0 N at an angle of 53° downwards from the direction of X

D 14.0 N at an angle of 53° upwards from the direction of X

3 Which expression using SI base units is equivalent to the volt?

A kg m2 s–1

A–1

B kg m s–2 A

C kg m2 s–1

A

D kg m2 s–3

A–1 4 A voltage is carefully measured with a high-quality instrument and found to be 2.321 V.

Two students, using two different methods, conclude that the voltage is 2.33 V and 2.344 V respectively.

Which statement is correct?

A 2.33 V is less accurate and less precise than 2.344 V.

B 2.33 V is less accurate and more precise than 2.344 V.

C 2.33 V is more accurate and less precise than 2.344 V.

D 2.33 V is more accurate and more precise than 2.344 V.

Page 45: PHYSICS 9702/11 READ THESE INSTRUCTIONS FIRSTmaxpapers.com/wp-content/uploads/2012/11/9702_s17… ·  · 2017-10-17PHYSICS 9702/11 Paper 1 Multiple Choice May/June 2017 1 hour 15

5

© UCLES 2017 9702/13/M/J/17 [Turn over

5 On a planet, a vertically-launched projectile takes 12.5 s to return to its starting position. The projectile gains a maximum height of 170 m. The planet does not have an atmosphere.

What is the acceleration of free fall on this planet?

A 2.2 m s–2 B 8.7 m s–2 C 27 m s–2 D 54 m s–2 6 A displacement-time graph for a toy car is shown.

2

0

–2

0 2 4 6 8 10

displacement / m

time / s

Which graph shows the variation with time of the velocity v of the car?

2

0

–2

0 2 4 6

A

8 10

v / m s–1

time / s

2

0

–2

0 2 4 6

B

8 10

v / m s–1

time / s

2

0

–2

0 2 4 6

C

8 10

v / m s–1

time / s

2

0

–2

0 2 4 6

D

8 10

v / m s–1

time / s

Page 46: PHYSICS 9702/11 READ THESE INSTRUCTIONS FIRSTmaxpapers.com/wp-content/uploads/2012/11/9702_s17… ·  · 2017-10-17PHYSICS 9702/11 Paper 1 Multiple Choice May/June 2017 1 hour 15

6

© UCLES 2017 9702/13/M/J/17

7 A driver stops his car in time t by gradually increasing the total braking force on the car. The graph shows the resultant force on the car.

00

ttime

force

Which graph shows how the speed of the car will vary during this time?

00

ttime

speed

A

00

ttime

speed

B

00

ttime

speed

C

00

ttime

speed

D

Page 47: PHYSICS 9702/11 READ THESE INSTRUCTIONS FIRSTmaxpapers.com/wp-content/uploads/2012/11/9702_s17… ·  · 2017-10-17PHYSICS 9702/11 Paper 1 Multiple Choice May/June 2017 1 hour 15

7

© UCLES 2017 9702/13/M/J/17 [Turn over

8 The graph shows the variation of a quantity y with a quantity x for a body that is falling in air at constant (terminal) velocity in a uniform gravitational field.

quantity y

quantity x00

Which quantities could x and y represent?

x y

A air resistance acceleration

B loss of height gain in kinetic energy

C loss of potential energy work done against air resistance

D time velocity

9 A ball of mass 2.0 kg travels horizontally with a speed of 4.0 m s–1. The ball collides with a wall

and rebounds in the opposite direction with a speed of 2.8 m s–1. The time of the collision is 150 ms.

What is the average force exerted on the wall?

A 16 N B 37 N C 53 N D 91 N 10 An ice-hockey puck of mass 150 g moves with an initial speed of 2.0 m s–1 along the surface of an

ice rink.

The puck slides a distance of 30 m in a straight line before stopping.

What is the average frictional force acting on the puck?

A 0.010 N B 0.020 N C 0.067 N D 0.44 N

Page 48: PHYSICS 9702/11 READ THESE INSTRUCTIONS FIRSTmaxpapers.com/wp-content/uploads/2012/11/9702_s17… ·  · 2017-10-17PHYSICS 9702/11 Paper 1 Multiple Choice May/June 2017 1 hour 15

8

© UCLES 2017 9702/13/M/J/17

11 A uniform beam of mass 1.4 kg is pivoted at P as shown. The beam has a length of 0.60 m and P is 0.20 m from one end. Loads of 3.0 kg and 6.0 kg are suspended 0.35 m and 0.15 m from the pivot as shown.

0.15 m0.35 m

3.0 kg 6.0 kg

0.20 m0.40 m

centre of beamof mass 1.4 kg

P

What is the torque that must be applied to the beam in order to maintain it in equilibrium?

A 0.010 N m B 0.10 N m C 0.29 N m D 2.8 N m 12 An air bubble is rising through a liquid at a constant speed. The forces on it are the upthrust U,

the viscous drag D and its weight W.

Which diagram shows the directions and relative sizes of the forces?

A B C D

U U UU

D

DDD

W WW

W

Page 49: PHYSICS 9702/11 READ THESE INSTRUCTIONS FIRSTmaxpapers.com/wp-content/uploads/2012/11/9702_s17… ·  · 2017-10-17PHYSICS 9702/11 Paper 1 Multiple Choice May/June 2017 1 hour 15

9

© UCLES 2017 9702/13/M/J/17 [Turn over

13 The diagram represents a sphere under water. P, Q, R and S are forces acting on the sphere due to the pressure of the water.

P

QS

R

water surface

Each force acts perpendicularly to the sphere’s surface. P and R act in opposite directions vertically. Q and S act in opposite directions horizontally.

Which information about the magnitudes of the forces is correct?

A P < R and S = Q

B P > R and S = Q

C P = R and S = Q and P < S

D P = R and S = Q and P = S 14 The first column in the table gives four examples of work being done. The second column gives

more detail of the action.

Which row is not correct?

example detail

A

a girl dives from a diving board into a swimming pool

work is done by the girl against the gravitational field as she falls

B

a man pushes a car along a level road

work is done by the man against friction

C

an electron is accelerated towards a positively charged plate

work is done on the electron by the electric field of the plate

D

a piston is pushed outwards as a gas expands

work is done on the atmosphere by the gas

Page 50: PHYSICS 9702/11 READ THESE INSTRUCTIONS FIRSTmaxpapers.com/wp-content/uploads/2012/11/9702_s17… ·  · 2017-10-17PHYSICS 9702/11 Paper 1 Multiple Choice May/June 2017 1 hour 15

10

© UCLES 2017 9702/13/M/J/17

15 A steam turbine is used to drive a generator. The input power to the turbine is PI and the output power is PO. The power loss in the turbine is PL, as shown below.

input power PI

turbine output power PO

power loss PL

generator

What is the efficiency of the turbine?

A O

L

P

P B

OP

PI

C I

P

PL D

IP

PO

16 A constant force pushes a block along a horizontal frictionless surface. The block moves from

rest through a fixed distance.

What is the relationship between the final speed v of the block and its mass m?

A v ∝ m

1 B v ∝ m C v ∝

m

1 D v ∝ m

17 A man has a mass of 80 kg. He ties himself to one end of a rope which passes over a single fixed

pulley. He pulls on the other end of the rope to lift himself up at an average speed of 50 cm s–1.

What is the average useful power at which he is working?

A 40 W B 0.39 kW C 4.0 kW D 39 kW 18 Two wires with the same Young modulus E and cross-sectional area A, but different lengths L,

are subject to different tensile forces F. The extension e of each wire is the same.

The column headings in the table show four different quantities.

Which quantities have the same value and which quantities have different values for the two wires?

e

FL L

Ae FL

E

A different different same

B different same same

C same different different

D same different same

Page 51: PHYSICS 9702/11 READ THESE INSTRUCTIONS FIRSTmaxpapers.com/wp-content/uploads/2012/11/9702_s17… ·  · 2017-10-17PHYSICS 9702/11 Paper 1 Multiple Choice May/June 2017 1 hour 15

11

© UCLES 2017 9702/13/M/J/17 [Turn over

19 Two springs X and Y stretch elastically. The graphs show the variation with extension x of the force F applied to each spring.

20

00 10

F / N

spring X

80

00 5

F / N

x / cm x / cm

spring Y

Which statement is correct?

A When each spring is given the same extension, the energy stored in Y is 4 times the energy stored in X.

B When each spring is given the same extension, the energy stored in Y is 8 times the energy stored in X.

C When the same force is applied to each spring, the energy stored in Y is 4 times the energy stored in X.

D When the same force is applied to each spring, the energy stored in Y is 8 times the energy stored in X.

20 The diagram shows the force-extension graph for a steel wire, up to its breaking point.

0 5 10

250

200

150

100

50

0

force / N

extension / mm

What is the best estimate of the work done to break the wire?

A 2.1 J B 2.3 J C 2.4 J D 2.5 J

Page 52: PHYSICS 9702/11 READ THESE INSTRUCTIONS FIRSTmaxpapers.com/wp-content/uploads/2012/11/9702_s17… ·  · 2017-10-17PHYSICS 9702/11 Paper 1 Multiple Choice May/June 2017 1 hour 15

12

© UCLES 2017 9702/13/M/J/17

21 Which statement about electromagnetic waves in a vacuum is correct?

A Amplitude is inversely proportional to velocity.

B Frequency is inversely proportional to wavelength.

C Intensity is proportional to amplitude.

D Velocity is proportional to wavelength. 22 A transverse wave travels along a rope. The graph shows the variation of the displacement of the

particles in the rope with distance along it at a particular instant.

displacement

0 distance / m

direction of travel of wave

0 1.0 2.0 3.0

At which distance along the rope do the particles have maximum upwards velocity?

A 0.5 m B 1.0 m C 1.5 m D 2.0 m

Page 53: PHYSICS 9702/11 READ THESE INSTRUCTIONS FIRSTmaxpapers.com/wp-content/uploads/2012/11/9702_s17… ·  · 2017-10-17PHYSICS 9702/11 Paper 1 Multiple Choice May/June 2017 1 hour 15

13

© UCLES 2017 9702/13/M/J/17 [Turn over

23 A trace is shown on the screen of a cathode-ray oscilloscope (c.r.o.).

1.0 cm

1.0 cm

The time-base setting is 2.5 ms cm–1 and the Y-gain is 2.0 V cm–1.

What is the frequency and the amplitude of the waveform displayed by the c.r.o.?

frequency

/ Hz amplitude

/ V

A 0.00375 4.0

B 0.00375 8.0

C 267 4.0

D 267 8.0

24 A high-speed train approaches a stationary observer at a speed of 80 m s–1. The train’s horn

emits a sound of frequency 250 Hz.

The speed of sound is 340 m s–1.

What is the observed frequency of the sound from the horn?

A 190 Hz B 200 Hz C 310 Hz D 330 Hz 25 Which row shows a correct frequency in Hz for each of the four principal radiations?

X-rays ultraviolet microwaves infra-red

A 1010 1014 1018 1015

B 1014 1018 1015 1010

C 1015 1010 1014 1018

D 1018 1015 1010 1014

Page 54: PHYSICS 9702/11 READ THESE INSTRUCTIONS FIRSTmaxpapers.com/wp-content/uploads/2012/11/9702_s17… ·  · 2017-10-17PHYSICS 9702/11 Paper 1 Multiple Choice May/June 2017 1 hour 15

14

© UCLES 2017 9702/13/M/J/17

26 A pipe of length 100 cm is open at both ends. A loudspeaker situated at one end of the pipe can emit sound of different wavelengths.

pipe

loudspeaker

100 cm

At which wavelength can a stationary wave be produced in the pipe?

A 50 cm B 75 cm C 150 cm D 300 cm 27 Monochromatic light is incident on a diffraction grating and a diffraction pattern is observed.

Which row shows possible effects of replacing the grating with one that has twice as many lines per millimetre?

number of orders of

diffraction visible angle between first and

second orders of diffraction

A decreases decreases

B decreases increases

C increases decreases

D increases increases

Page 55: PHYSICS 9702/11 READ THESE INSTRUCTIONS FIRSTmaxpapers.com/wp-content/uploads/2012/11/9702_s17… ·  · 2017-10-17PHYSICS 9702/11 Paper 1 Multiple Choice May/June 2017 1 hour 15

15

© UCLES 2017 9702/13/M/J/17 [Turn over

28 Monochromatic light of wavelength λ is incident on two narrow slits S1 and S2, a small distance apart. A series of bright and dark fringes are observed on a screen a long distance away from the slits.

P

screenS1

S2

lightwavelength λ

The n th dark fringe from the central bright fringe is observed at point P on the screen.

Which equation is correct for all positive values of n?

A S2P – S1P = 2

λn

B S2P – S1P = nλ

C S2P – S1P = (n – 2

1 )λ

D S2P – S1P = (n + 2

1 )λ

29 A dipole is a pair of charges of equal magnitude, one negative and one positive. The electric field

of a dipole is shown below.

In which direction does the force act on an electron when at point X?

XA D

CB

+–

Page 56: PHYSICS 9702/11 READ THESE INSTRUCTIONS FIRSTmaxpapers.com/wp-content/uploads/2012/11/9702_s17… ·  · 2017-10-17PHYSICS 9702/11 Paper 1 Multiple Choice May/June 2017 1 hour 15

16

© UCLES 2017 9702/13/M/J/17

30 In a uniform electric field, which statement is correct?

A All charged particles experience the same force.

B All charged particles move with the same velocity.

C All electric field lines are directed towards positive charges.

D All electric field lines are parallel. 31 Two metal plates are a distance of 30 cm apart in a vacuum.

A current exists between the two plates consisting of electrons moving at a constant speed of

1.5 × 106 m s–1.

At any instant, there is always just one electron travelling between the plates.

What is the current between the plates?

A 3.2 × 10–26 A

B 8.0 × 10–13 A

C 1.6 × 10–12 A

D 2.0 × 10–7 A

32 The diagram shows a portable generator connected by cables to floodlights. The generator

produces a current of 10 A at a constant potential difference (p.d.) of 240 V.

The total resistance of the cables is 2 Ω.

generator

240 V10 A

floodlights

1 Ω

1 Ω

cables

What is the p.d. V across, and the power P delivered to, the floodlights?

V / V P / W

A 220 2000

B 220 2200

C 230 2000

D 230 2300

Page 57: PHYSICS 9702/11 READ THESE INSTRUCTIONS FIRSTmaxpapers.com/wp-content/uploads/2012/11/9702_s17… ·  · 2017-10-17PHYSICS 9702/11 Paper 1 Multiple Choice May/June 2017 1 hour 15

17

© UCLES 2017 9702/13/M/J/17 [Turn over

33 A metal cube with sides of length a has electrical resistance R between opposite faces.

a

What is the resistance between the opposite faces of a cube of the same metal with sides of length 3a?

A 9R B 3R C 3

R D 9

R

34 A graph of potential difference (p.d.) V across a cell against current I in the cell is shown.

V

I0

0

As the cell reaches the end of its useful life, its internal resistance increases and its electromotive force (e.m.f.) decreases.

Which diagram shows a graph of V against I for the cell nearing the end of its useful life?

V

I

A

00

V

I

B

00

V

I

C

00

V

I

D

00

line oforiginalgraph

Page 58: PHYSICS 9702/11 READ THESE INSTRUCTIONS FIRSTmaxpapers.com/wp-content/uploads/2012/11/9702_s17… ·  · 2017-10-17PHYSICS 9702/11 Paper 1 Multiple Choice May/June 2017 1 hour 15

18

© UCLES 2017 9702/13/M/J/17

35 A 20 V d.c. supply is connected to a circuit consisting of five resistors L, M, N, P and Q.

QP

ML

N

+ –

20 V rise

7 V drop4 V

drop

There is a potential drop of 7 V across L and a further 4 V potential drop across N.

What are the potential drops across M, P and Q?

potential drop across M / V

potential drop across P / V

potential drop across Q / V

A 9 7 13

B 13 7 13

C 13 11 9

D 17 3 17

36 A potential divider circuit consists of a cell of negligible internal resistance in series with two

variable resistors of resistances R1 and R2. The potential difference (p.d.) across the cell is V0. The p.d. at the output is V.

V0

V

R1

R2

Which statement is correct?

A When R1 increases, it takes a greater proportion of V0, so V decreases.

B When R1 increases, the current through R1 and R2 decreases, so V increases.

C When R2 decreases, it takes a smaller proportion of V0, so V increases.

D When R2 increases, the current through R1 and R2 decreases, so V decreases.

Page 59: PHYSICS 9702/11 READ THESE INSTRUCTIONS FIRSTmaxpapers.com/wp-content/uploads/2012/11/9702_s17… ·  · 2017-10-17PHYSICS 9702/11 Paper 1 Multiple Choice May/June 2017 1 hour 15

19

© UCLES 2017 9702/13/M/J/17

37 A nucleus of uranium-238, U238

92, decays in a series of steps to form a nucleus of lead-206,

Pb206

82, as shown.

U238

92 → ............ → Pb

206

82

An α-particle or a β– particle is emitted during each step.

What is the total number of β– particles that are emitted?

A 6 B 8 C 10 D 16

38 Which statement about α-particles is correct?

A α-particles emitted from a single radioactive isotope have a continuous distribution of energies.

B α-particles have less ionising power than β-particles.

C The charge of an α-particle is +1.60 × 10–19 C.

D The speeds of α-particles can be as high as 1.5 × 107 m s–1.

39 The nuclear equation shown has a term missing.

C14

6 → N

14

7 + β

0

1 + .............

What is represented by the missing term?

A an antineutrino

B an electron

C a neutrino

D a positron 40 Which particle is a fundamental particle?

A electron

B hadron

C neutron

D proton

Page 60: PHYSICS 9702/11 READ THESE INSTRUCTIONS FIRSTmaxpapers.com/wp-content/uploads/2012/11/9702_s17… ·  · 2017-10-17PHYSICS 9702/11 Paper 1 Multiple Choice May/June 2017 1 hour 15

20

Permission to reproduce items where third-party owned material protected by copyright is included has been sought and cleared where possible. Every reasonable effort has been made by the publisher (UCLES) to trace copyright holders, but if any items requiring clearance have unwittingly been included, the publisher will be pleased to make amends at the earliest possible opportunity. To avoid the issue of disclosure of answer-related information to candidates, all copyright acknowledgements are reproduced online in the Cambridge International Examinations Copyright Acknowledgements Booklet. This is produced for each series of examinations and is freely available to download at www.cie.org.uk after the live examination series.

Cambridge International Examinations is part of the Cambridge Assessment Group. Cambridge Assessment is the brand name of University of Cambridge Local Examinations Syndicate (UCLES), which is itself a department of the University of Cambridge.

© UCLES 2017 9702/13/M/J/17

BLANK PAGE

Page 61: PHYSICS 9702/11 READ THESE INSTRUCTIONS FIRSTmaxpapers.com/wp-content/uploads/2012/11/9702_s17… ·  · 2017-10-17PHYSICS 9702/11 Paper 1 Multiple Choice May/June 2017 1 hour 15

This document consists of 15 printed pages and 1 blank page.

DC (NH/SW) 127081/3© UCLES 2017 [Turn over

*7652535072*

PHYSICS 9702/21Paper 2 AS Level Structured Questions May/June 2017 1 hour 15 minutesCandidates answer on the Question Paper.No Additional Materials are required.

READ THESE INSTRUCTIONS FIRST

Write your Centre number, candidate number and name on all the work you hand in.Write in dark blue or black pen.You may use an HB pencil for any diagrams or graphs.Do not use staples, paper clips, glue or correction fluid.DO NOT WRITE IN ANY BARCODES.

Answer all questions.

Electronic calculators may be used.You may lose marks if you do not show your working or if you do not use appropriate units.

At the end of the examination, fasten all your work securely together.The number of marks is given in brackets [ ] at the end of each question or part question.

Cambridge International ExaminationsCambridge International Advanced Subsidiary and Advanced Level

Page 62: PHYSICS 9702/11 READ THESE INSTRUCTIONS FIRSTmaxpapers.com/wp-content/uploads/2012/11/9702_s17… ·  · 2017-10-17PHYSICS 9702/11 Paper 1 Multiple Choice May/June 2017 1 hour 15

2

9702/21/M/J/17© UCLES 2017

Data

speed of light in free space c = 3.00 × 108 m s−1

permeability of free space μ0 = 4π × 10−7 H m−1

permittivity of free space ε0 = 8.85 × 10−12 F m−1

( 14πε0

= 8.99 × 109 m F−1) elementary charge e = 1.60 × 10−19 C

the Planck constant h = 6.63 × 10−34 J s

unified atomic mass unit 1 u = 1.66 × 10−27 kg

rest mass of electron me = 9.11 × 10−31 kg

rest mass of proton mp = 1.67 × 10−27 kg

molar gas constant R = 8.31 J K−1 mol−1

the Avogadro constant NA = 6.02 × 1023 mol−1

the Boltzmann constant k = 1.38 × 10−23 J K−1

gravitational constant G = 6.67 × 10−11 N m2 kg−2

acceleration of free fall g = 9.81 m s−2

Page 63: PHYSICS 9702/11 READ THESE INSTRUCTIONS FIRSTmaxpapers.com/wp-content/uploads/2012/11/9702_s17… ·  · 2017-10-17PHYSICS 9702/11 Paper 1 Multiple Choice May/June 2017 1 hour 15

3

9702/21/M/J/17© UCLES 2017 [Turn over

Formulae

uniformly accelerated motion s = ut + 12 at 2

v2 = u2 + 2as

work done on/by a gas W = pΔV

gravitational potential φ =− Gmr

hydrostatic pressure p = ρgh

pressure of an ideal gas p = 13 NmV ⟨c2⟩

simple harmonic motion a =−ω 2x

velocity of particle in s.h.m. v = v0 cos ωt v = ± ω ( )x x0

2 2-

Doppler effect fo = fsv

v ± vs

electric potential V = Q4πε0r

capacitors in series 1/C = 1/C1 + 1/C2 + . . .

capacitors in parallel C = C1 + C2 + . . .

energy of charged capacitor W = 12 QV

electric current I = Anvq

resistors in series R = R1 + R2 + . . .

resistors in parallel 1/R = 1/R1 + 1/R2 + . . .

Hall voltage VH = BIntq

alternating current/voltage x = x0 sin ωt

radioactive decay x = x0exp(−λt )

decay constant λ = 0.693t 1

2

Page 64: PHYSICS 9702/11 READ THESE INSTRUCTIONS FIRSTmaxpapers.com/wp-content/uploads/2012/11/9702_s17… ·  · 2017-10-17PHYSICS 9702/11 Paper 1 Multiple Choice May/June 2017 1 hour 15

4

9702/21/M/J/17© UCLES 2017

Answer all the questions in the spaces provided.

1 (a) Determine the SI base units of stress. Show your working.

base units ...........................................................[2]

(b) A beam PQ is clamped so that the beam is horizontal. A mass M of 500 g is hung from end Q and the beam bends slightly, as illustrated in Fig. 1.1.

lR

P

clamp

Qhorizontal

M

Fig. 1.1

The length l of the beam from the edge of the clamp R to end Q is 60.0 cm. The width b of the beam is 30.0 mm and the thickness d of the beam is 5.00 mm. The material of the beam has Young modulus E.

The mass M is made to oscillate vertically. The time period T of the oscillations is 0.58 s.

The period T is given by the expression

T = 2π 4Ebd

Ml3

3.

(i) Determine E in GPa.

E = ...................................................GPa [3]

Page 65: PHYSICS 9702/11 READ THESE INSTRUCTIONS FIRSTmaxpapers.com/wp-content/uploads/2012/11/9702_s17… ·  · 2017-10-17PHYSICS 9702/11 Paper 1 Multiple Choice May/June 2017 1 hour 15

5

9702/21/M/J/17© UCLES 2017 [Turn over

(ii) The quantities used to determine E should be measured with accuracy and with precision.

1. Explain the difference between accuracy and precision.

accuracy: ....................................................................................................................

.....................................................................................................................................

precision: ....................................................................................................................

..................................................................................................................................... [2]

2. In a particular experiment, the quantities l and T are measured with the same percentage uncertainty. State and explain which of these two quantities contributes more to the uncertainty in the value of E.

.....................................................................................................................................

.....................................................................................................................................

.................................................................................................................................[1]

[Total: 8]

Page 66: PHYSICS 9702/11 READ THESE INSTRUCTIONS FIRSTmaxpapers.com/wp-content/uploads/2012/11/9702_s17… ·  · 2017-10-17PHYSICS 9702/11 Paper 1 Multiple Choice May/June 2017 1 hour 15

6

9702/21/M/J/17© UCLES 2017

2 (a) State the two conditions for a system to be in equilibrium.

1. ...............................................................................................................................................

...................................................................................................................................................

2. ...............................................................................................................................................

................................................................................................................................................... [2]

(b) A paraglider P of mass 95 kg is pulled by a wire attached to a boat, as shown in Fig. 2.1.

25°

wire

boat

waterhorizontal

paragliderP

parachute

Fig. 2.1

The wire makes an angle of 25° with the horizontal water surface. P moves in a straight line parallel to the surface of the water.

The variation with time t of the velocity v of P is shown in Fig. 2.2.

00

2.0

4.0

6.0

8.0

10.0

2.0 4.0 6.0 8.0t / s

v / m s–1

Fig. 2.2

Page 67: PHYSICS 9702/11 READ THESE INSTRUCTIONS FIRSTmaxpapers.com/wp-content/uploads/2012/11/9702_s17… ·  · 2017-10-17PHYSICS 9702/11 Paper 1 Multiple Choice May/June 2017 1 hour 15

7

9702/21/M/J/17© UCLES 2017 [Turn over

(i) Show that the acceleration of P is 1.4 m s–2 at time t = 5.0 s.

[2]

(ii) Calculate the total distance moved by P from time t = 0 to t = 7.0 s.

distance = .......................................................m [2]

(iii) Calculate the change in kinetic energy of P from time t = 0 to t = 7.0 s.

change in kinetic energy = ........................................................J [2]

(iv) The tension in the wire at time t = 5.0 s is 280 N.

Calculate, for the horizontal motion,

1. the vertical lift force F supporting P,

F = ....................................................... N [3]

2. the force R due to air resistance acting on P in the horizontal direction.

R = ....................................................... N [3]

[Total: 14]

Page 68: PHYSICS 9702/11 READ THESE INSTRUCTIONS FIRSTmaxpapers.com/wp-content/uploads/2012/11/9702_s17… ·  · 2017-10-17PHYSICS 9702/11 Paper 1 Multiple Choice May/June 2017 1 hour 15

8

9702/21/M/J/17© UCLES 2017

3 (a) A cylinder is made from a material of density 2.7 g cm–3. The cylinder has diameter 2.4 cm and length 5.0 cm.

Show that the cylinder has weight 0.60 N.

[3]

(b) The cylinder in (a) is hung from the end A of a non-uniform bar AB, as shown in Fig. 3.1.

12 cmbar

cylinder

BP

A

20 cm

0.25 N

0.60 N

50 cm

X

Fig. 3.1

The bar has length 50 cm and has weight 0.25 N. The centre of gravity of the bar is 20 cm from B. The bar is pivoted at P. The pivot is 12 cm from B.

An object X is hung from end B. The weight of X is adjusted until the bar is horizontal and in equilibrium.

(i) Explain what is meant by centre of gravity.

...........................................................................................................................................

.......................................................................................................................................[1]

Page 69: PHYSICS 9702/11 READ THESE INSTRUCTIONS FIRSTmaxpapers.com/wp-content/uploads/2012/11/9702_s17… ·  · 2017-10-17PHYSICS 9702/11 Paper 1 Multiple Choice May/June 2017 1 hour 15

9

9702/21/M/J/17© UCLES 2017 [Turn over

(ii) Calculate the weight of X.

weight of X = ............................................... N [3]

(c) The cylinder is now immersed in water, as illustrated in Fig. 3.2.

water X

B

P

A

0.25 N

Fig. 3.2

An upthrust acts on the cylinder and the bar is not in equilibrium.

(i) Explain the origin of the upthrust.

...........................................................................................................................................

...........................................................................................................................................

...........................................................................................................................................

.......................................................................................................................................[2]

(ii) Explain why the weight of X must be reduced in order to obtain equilibrium for AB.

...........................................................................................................................................

...........................................................................................................................................

.......................................................................................................................................[1]

[Total: 10]

Page 70: PHYSICS 9702/11 READ THESE INSTRUCTIONS FIRSTmaxpapers.com/wp-content/uploads/2012/11/9702_s17… ·  · 2017-10-17PHYSICS 9702/11 Paper 1 Multiple Choice May/June 2017 1 hour 15

10

9702/21/M/J/17© UCLES 2017

4 (a) State the conditions required for the formation of stationary waves.

...................................................................................................................................................

...................................................................................................................................................

...............................................................................................................................................[2]

(b) One end of a string is attached to a vibrator. The string is stretched by passing the other end over a pulley and attaching a load, as illustrated in Fig. 4.1.

string

A Bvibrator

pulley

support forpulley

load

Fig. 4.1

The frequency of vibration of the vibrator is adjusted to 250 Hz and a transverse wave travels along the string with a speed of 12 m s–1. The wave is reflected at the pulley and a stationary wave forms on the string.

Fig. 4.2 shows the string between points A and B at time t = t1.

A B

string

Fig. 4.2

At time t = t1 the string has maximum displacement.

(i) Calculate the distance AB.

distance = .......................................................m [2]

Page 71: PHYSICS 9702/11 READ THESE INSTRUCTIONS FIRSTmaxpapers.com/wp-content/uploads/2012/11/9702_s17… ·  · 2017-10-17PHYSICS 9702/11 Paper 1 Multiple Choice May/June 2017 1 hour 15

11

9702/21/M/J/17© UCLES 2017 [Turn over

(ii) On Fig. 4.2, sketch the position of the string between A and B at times

1. t = t1 + 2.0 ms (label this line P),

2. t = t1 + 5.0 ms (label this line Q).

[3]

[Total: 7]

Page 72: PHYSICS 9702/11 READ THESE INSTRUCTIONS FIRSTmaxpapers.com/wp-content/uploads/2012/11/9702_s17… ·  · 2017-10-17PHYSICS 9702/11 Paper 1 Multiple Choice May/June 2017 1 hour 15

12

9702/21/M/J/17© UCLES 2017

BLANK PAGE

Page 73: PHYSICS 9702/11 READ THESE INSTRUCTIONS FIRSTmaxpapers.com/wp-content/uploads/2012/11/9702_s17… ·  · 2017-10-17PHYSICS 9702/11 Paper 1 Multiple Choice May/June 2017 1 hour 15

13

9702/21/M/J/17© UCLES 2017 [Turn over

5 (a) Describe the Doppler effect.

...................................................................................................................................................

...................................................................................................................................................

...............................................................................................................................................[1]

(b) A car travels with a constant velocity along a straight road. The car horn with a frequency of 400 Hz is sounded continuously. A stationary observer on the roadside hears the sound from the horn at a frequency of 360 Hz.

The speed of sound is 340 m s–1.

Determine the magnitude v, and the direction, of the velocity of the car relative to the observer.

v = .......................................................m s–1

direction ............................................................... [3]

[Total: 4]

Page 74: PHYSICS 9702/11 READ THESE INSTRUCTIONS FIRSTmaxpapers.com/wp-content/uploads/2012/11/9702_s17… ·  · 2017-10-17PHYSICS 9702/11 Paper 1 Multiple Choice May/June 2017 1 hour 15

14

9702/21/M/J/17© UCLES 2017

6 (a) Define the ohm.

...................................................................................................................................................

...............................................................................................................................................[1]

(b) A cell X of electromotive force (e.m.f.) 1.5 V and negligible internal resistance is connected in series to three resistors A, B and C, as shown in Fig. 6.1.

6.0 Ω

A

3.0 Ω

B 4.0 Ω

1.5 VX

C

Fig. 6.1

Resistors A and B have resistances 6.0 Ω and 3.0 Ω respectively and are connected in parallel. Resistor C has resistance 4.0 Ω and is connected in series with the parallel combination.

Calculate

(i) the current in the circuit,

current = ........................................................A [3]

(ii) the current in resistor B,

current = ........................................................A [1]

Page 75: PHYSICS 9702/11 READ THESE INSTRUCTIONS FIRSTmaxpapers.com/wp-content/uploads/2012/11/9702_s17… ·  · 2017-10-17PHYSICS 9702/11 Paper 1 Multiple Choice May/June 2017 1 hour 15

15

9702/21/M/J/17© UCLES 2017 [Turn over

(iii) the ratio

power dissipated in resistor Bpower dissipated in resistor C

.

ratio = ...........................................................[2]

(c) The resistors A, B and C in (b) are wires of the same material and have the same length.

(i) Explain how the resistors may be made with different resistance values.

.......................................................................................................................................[1]

(ii) Calculate the ratio

average drift speed of the charge carriers in resistor B average drift speed of the charge carriers in resistor C

.

ratio = ...........................................................[2]

(d) A cell of e.m.f. 1.5 V and negligible internal resistance is connected in parallel with cell X in Fig. 6.1 with their positive terminals together.

State the change, if any, to the current in

(i) cell X,

.......................................................................................................................................[1]

(ii) resistor C.

.......................................................................................................................................[1]

[Total: 12]

Page 76: PHYSICS 9702/11 READ THESE INSTRUCTIONS FIRSTmaxpapers.com/wp-content/uploads/2012/11/9702_s17… ·  · 2017-10-17PHYSICS 9702/11 Paper 1 Multiple Choice May/June 2017 1 hour 15

16

9702/21/M/J/17© UCLES 2017

7 (a) Use the quark model to show that

(i) the charge on a proton is +e,

.......................................................................................................................................[1]

(ii) the charge on a neutron is zero.

.......................................................................................................................................[1]

(b) A nucleus of 93

08Sr decays by the emission of a β– particle. A nucleus of 6

249Cu decays by the

emission of a β+ particle.

(i) In Fig. 7.1, state the nucleon number and proton number for the nucleus produced in each of these decay processes.

nucleus formed by β– decay nucleus formed by β+ decay

nucleon number

proton number

Fig. 7.1 [1]

(ii) State the name of the force responsible for β decay.

.......................................................................................................................................[1]

(iii) State the names of the leptons produced in each of the decay processes.

β– decay: ...........................................................................................................................

β+ decay: ............................................................................................................................ [1]

[Total: 5]

Permission to reproduce items where third-party owned material protected by copyright is included has been sought and cleared where possible. Every reasonable effort has been made by the publisher (UCLES) to trace copyright holders, but if any items requiring clearance have unwittingly been included, the publisher will be pleased to make amends at the earliest possible opportunity.

To avoid the issue of disclosure of answer-related information to candidates, all copyright acknowledgements are reproduced online in the Cambridge International Examinations Copyright Acknowledgements Booklet. This is produced for each series of examinations and is freely available to download at www.cie.org.uk after the live examination series.

Cambridge International Examinations is part of the Cambridge Assessment Group. Cambridge Assessment is the brand name of University of Cambridge Local Examinations Syndicate (UCLES), which is itself a department of the University of Cambridge.

Page 77: PHYSICS 9702/11 READ THESE INSTRUCTIONS FIRSTmaxpapers.com/wp-content/uploads/2012/11/9702_s17… ·  · 2017-10-17PHYSICS 9702/11 Paper 1 Multiple Choice May/June 2017 1 hour 15

This document consists of 16 printed pages.

DC (NH/FD) 127433/3© UCLES 2017 [Turn over

*5643781577*

PHYSICS 9702/22Paper 2 AS Level Structured Questions May/June 2017 1 hour 15 minutesCandidates answer on the Question Paper.No Additional Materials are required.

READ THESE INSTRUCTIONS FIRST

Write your Centre number, candidate number and name on all the work you hand in.Write in dark blue or black pen.You may use an HB pencil for any diagrams or graphs.Do not use staples, paper clips, glue or correction fluid.DO NOT WRITE IN ANY BARCODES.

Answer all questions.

Electronic calculators may be used.You may lose marks if you do not show your working or if you do not use appropriate units.

At the end of the examination, fasten all your work securely together.The number of marks is given in brackets [ ] at the end of each question or part question.

Cambridge International ExaminationsCambridge International Advanced Subsidiary and Advanced Level

Page 78: PHYSICS 9702/11 READ THESE INSTRUCTIONS FIRSTmaxpapers.com/wp-content/uploads/2012/11/9702_s17… ·  · 2017-10-17PHYSICS 9702/11 Paper 1 Multiple Choice May/June 2017 1 hour 15

2

9702/22/M/J/17© UCLES 2017

Data

speed of light in free space c = 3.00 × 108 m s−1

permeability of free space μ0 = 4π × 10−7 H m−1

permittivity of free space ε0 = 8.85 × 10−12 F m−1

( 14πε0

= 8.99 × 109 m F−1) elementary charge e = 1.60 × 10−19 C

the Planck constant h = 6.63 × 10−34 J s

unified atomic mass unit 1 u = 1.66 × 10−27 kg

rest mass of electron me = 9.11 × 10−31 kg

rest mass of proton mp = 1.67 × 10−27 kg

molar gas constant R = 8.31 J K−1 mol−1

the Avogadro constant NA = 6.02 × 1023 mol−1

the Boltzmann constant k = 1.38 × 10−23 J K−1

gravitational constant G = 6.67 × 10−11 N m2 kg−2

acceleration of free fall g = 9.81 m s−2

Page 79: PHYSICS 9702/11 READ THESE INSTRUCTIONS FIRSTmaxpapers.com/wp-content/uploads/2012/11/9702_s17… ·  · 2017-10-17PHYSICS 9702/11 Paper 1 Multiple Choice May/June 2017 1 hour 15

3

9702/22/M/J/17© UCLES 2017 [Turn over

Formulae

uniformly accelerated motion s = ut + 12 at 2

v2 = u2 + 2as

work done on/by a gas W = pΔV

gravitational potential φ =− Gmr

hydrostatic pressure p = ρgh

pressure of an ideal gas p = 13 NmV ⟨c2⟩

simple harmonic motion a =−ω 2x

velocity of particle in s.h.m. v = v0 cos ωt v = ± ω ( )x x0

2 2-

Doppler effect fo = fsv

v ± vs

electric potential V = Q4πε0r

capacitors in series 1/C = 1/C1 + 1/C2 + . . .

capacitors in parallel C = C1 + C2 + . . .

energy of charged capacitor W = 12 QV

electric current I = Anvq

resistors in series R = R1 + R2 + . . .

resistors in parallel 1/R = 1/R1 + 1/R2 + . . .

Hall voltage VH = BIntq

alternating current/voltage x = x0 sin ωt

radioactive decay x = x0exp(−λt )

decay constant λ = 0.693t 1

2

Page 80: PHYSICS 9702/11 READ THESE INSTRUCTIONS FIRSTmaxpapers.com/wp-content/uploads/2012/11/9702_s17… ·  · 2017-10-17PHYSICS 9702/11 Paper 1 Multiple Choice May/June 2017 1 hour 15

4

9702/22/M/J/17© UCLES 2017

Answer all the questions in the spaces provided.

1 (a) State two SI base units other than kilogram, metre and second.

1. ...............................................................................................................................................

2. ............................................................................................................................................... [1]

(b) Determine the SI base units of resistivity.

base units ...........................................................[3]

Page 81: PHYSICS 9702/11 READ THESE INSTRUCTIONS FIRSTmaxpapers.com/wp-content/uploads/2012/11/9702_s17… ·  · 2017-10-17PHYSICS 9702/11 Paper 1 Multiple Choice May/June 2017 1 hour 15

5

9702/22/M/J/17© UCLES 2017 [Turn over

(c) (i) A wire of cross-sectional area 1.5 mm2 and length 2.5 m has a resistance of 0.030 Ω. Calculate the resistivity of the material of the wire in nΩ m.

resistivity = ..................................................nΩ m [3]

(ii) 1. State what is meant by precision.

....................................................................................................................................

....................................................................................................................................

2. Explain why the precision in the value of the resistivity is improved by using a micrometer screw gauge rather than a metre rule to measure the diameter of the wire.

....................................................................................................................................

....................................................................................................................................

.................................................................................................................................... [2]

[Total: 9]

Page 82: PHYSICS 9702/11 READ THESE INSTRUCTIONS FIRSTmaxpapers.com/wp-content/uploads/2012/11/9702_s17… ·  · 2017-10-17PHYSICS 9702/11 Paper 1 Multiple Choice May/June 2017 1 hour 15

6

9702/22/M/J/17© UCLES 2017

2 (a) Define velocity.

...................................................................................................................................................

...............................................................................................................................................[1]

(b) A ball of mass 0.45 kg leaves the edge of a table with a horizontal velocity v, as shown in Fig. 2.1.

ball

v

table1.25 m

path of ball

floorhorizontal

1.50 m

Fig. 2.1

The height of the table is 1.25 m. The ball travels a distance of 1.50 m horizontally before hitting the floor.

Air resistance is negligible.

Calculate, for the ball,

(i) the horizontal velocity v as it leaves the table,

v = ..................................................m s–1 [3]

Page 83: PHYSICS 9702/11 READ THESE INSTRUCTIONS FIRSTmaxpapers.com/wp-content/uploads/2012/11/9702_s17… ·  · 2017-10-17PHYSICS 9702/11 Paper 1 Multiple Choice May/June 2017 1 hour 15

7

9702/22/M/J/17© UCLES 2017 [Turn over

(ii) the velocity just as it hits the floor,

magnitude of velocity = .......................................................m s–1

angle to the horizontal = ............................................................. ° [4]

(iii) the kinetic energy just as it hits the floor,

kinetic energy = ........................................................J [2]

(iv) the loss in gravitational potential energy as it falls from the table to the floor.

loss in potential energy = ........................................................J [2]

(c) Explain why the kinetic energy of the ball in (b)(iii) does not equal the loss of gravitational potential energy in (b)(iv).

...................................................................................................................................................

...............................................................................................................................................[1]

[Total: 13]

Page 84: PHYSICS 9702/11 READ THESE INSTRUCTIONS FIRSTmaxpapers.com/wp-content/uploads/2012/11/9702_s17… ·  · 2017-10-17PHYSICS 9702/11 Paper 1 Multiple Choice May/June 2017 1 hour 15

8

9702/22/M/J/17© UCLES 2017

3 The Young modulus of the material of a wire can be determined using the apparatus shown in Fig. 3.1.

benchscale S

wire marker on wire

pulley

masses

F

XC

clamp

Fig. 3.1

One end of the wire is clamped at C and a marker is attached to the wire above a scale S. A force to extend the wire is applied by attaching masses to the other end of the wire.

The reading X of the marker on the scale S is determined for different forces F applied to the end of the wire. The variation with X of F is shown in Fig. 3.2.

2.00

10

20

30

40

4.0 6.0 8.0 10.0X / mm

F / N

12.0

Fig. 3.2

Page 85: PHYSICS 9702/11 READ THESE INSTRUCTIONS FIRSTmaxpapers.com/wp-content/uploads/2012/11/9702_s17… ·  · 2017-10-17PHYSICS 9702/11 Paper 1 Multiple Choice May/June 2017 1 hour 15

9

9702/22/M/J/17© UCLES 2017 [Turn over

(a) The length of the wire from C to the marker for F = 0 is 3.50 m. The diameter of the wire is 0.38 mm.

Use the gradient of the line in Fig. 3.2 to determine the Young modulus E of the material of the wire in TPa.

E = ................................................... TPa [3]

(b) The experiment is repeated with a thicker wire of the same material and length.

State how the range of the force F must be changed to obtain the same range of scale readings as in Fig. 3.2.

...................................................................................................................................................

...................................................................................................................................................

...............................................................................................................................................[1]

[Total: 4]

Page 86: PHYSICS 9702/11 READ THESE INSTRUCTIONS FIRSTmaxpapers.com/wp-content/uploads/2012/11/9702_s17… ·  · 2017-10-17PHYSICS 9702/11 Paper 1 Multiple Choice May/June 2017 1 hour 15

10

9702/22/M/J/17© UCLES 2017

4 (a) StateNewton’sfirstlawofmotion.

...................................................................................................................................................

...............................................................................................................................................[1]

(b) An object A of mass 100 g is moving in a straight line with a velocity of 0.60 m s–1 to the right. An object B of mass 200 g is moving in the same straight line as object A with a velocity of 0.80 m s–1 to the left, as shown in Fig. 4.1.

100 g0.60 m s–1 0.80 m s–1

200 g

A B

Fig. 4.1

Objects A and B collide. Object A then moves with a velocity of 0.40 m s–1 to the left.

(i) Calculate the magnitude of the velocity of B after the collision.

magnitude of velocity = ..................................................m s–1 [2]

(ii) The collision between A and B is inelastic.

Explain how the collision is inelastic and still obeys the law of conservation of energy.

...........................................................................................................................................

...........................................................................................................................................

.......................................................................................................................................[1]

[Total: 4]

5 (a) Define the frequency of a sound wave.

...................................................................................................................................................

...............................................................................................................................................[1]

(b) A sound wave travels through air. Describe the motion of the air particles relative to the direction of travel of the sound wave.

...................................................................................................................................................

...............................................................................................................................................[1]

Page 87: PHYSICS 9702/11 READ THESE INSTRUCTIONS FIRSTmaxpapers.com/wp-content/uploads/2012/11/9702_s17… ·  · 2017-10-17PHYSICS 9702/11 Paper 1 Multiple Choice May/June 2017 1 hour 15

11

9702/22/M/J/17© UCLES 2017 [Turn over

(c) The sound wave emitted from the horn of a stationary car is detected with a microphone and displayed on a cathode-ray oscilloscope (c.r.o.), as shown in Fig. 5.1.

1.0 cm

1.0 cm

Fig. 5.1

The y-axis setting is 5.0 mV cm–1. The time-base setting is 0.50 ms cm–1.

(i) Use Fig. 5.1 to determine the frequency of the sound wave.

frequency = ..................................................... Hz [2]

(ii) The horn of the car sounds continuously. Describe the changes to the trace seen on the c.r.o. as the car travels at constant speed

1. directly towards the stationary microphone,

....................................................................................................................................

....................................................................................................................................

2. directly away from the stationary microphone.

....................................................................................................................................

.................................................................................................................................... [3]

[Total: 7]

Page 88: PHYSICS 9702/11 READ THESE INSTRUCTIONS FIRSTmaxpapers.com/wp-content/uploads/2012/11/9702_s17… ·  · 2017-10-17PHYSICS 9702/11 Paper 1 Multiple Choice May/June 2017 1 hour 15

12

9702/22/M/J/17© UCLES 2017

6 (a) Interference fringes may be observed using a light-emitting laser to illuminate a double slit. The double slit acts as two sources of light.

Explain

(i) the part played by diffraction in the production of the fringes,

...........................................................................................................................................

...........................................................................................................................................

.......................................................................................................................................[2]

(ii) the reason why a double slit is used rather than two separate sources of light.

...........................................................................................................................................

...........................................................................................................................................

.......................................................................................................................................[1]

Page 89: PHYSICS 9702/11 READ THESE INSTRUCTIONS FIRSTmaxpapers.com/wp-content/uploads/2012/11/9702_s17… ·  · 2017-10-17PHYSICS 9702/11 Paper 1 Multiple Choice May/June 2017 1 hour 15

13

9702/22/M/J/17© UCLES 2017 [Turn over

(b) A laser emitting light of a single wavelength is used to illuminate slits S1 and S2, as shown in Fig. 6.1.

laser

screenlight

2.4 m

B

A

0.48 mm

S1

S2

Fig. 6.1 (not to scale)

An interference pattern is observed on the screen AB. The separation of the slits is 0.48 mm. The slits are 2.4 m from AB. The distance on the screen across 16 fringes is 36 mm, as illustrated in Fig. 6.2.

16 fringes

36 mm

Fig. 6.2

Calculate the wavelength of the light emitted by the laser.

wavelength = .......................................................m [3]

Page 90: PHYSICS 9702/11 READ THESE INSTRUCTIONS FIRSTmaxpapers.com/wp-content/uploads/2012/11/9702_s17… ·  · 2017-10-17PHYSICS 9702/11 Paper 1 Multiple Choice May/June 2017 1 hour 15

14

9702/22/M/J/17© UCLES 2017

(c) Two dippers D1 and D2 are used to produce identical waves on the surface of water, as illustrated in Fig. 6.3.

D1

P

D2

11.2 cm water

7.2 cm7.2 cm

Fig. 6.3 (not to scale)

Point P is 7.2 cm from D1 and 11.2 cm from D2.

The wavelength of the waves is 1.6 cm. The phase difference between the waves produced at D1 and D2 is zero.

(i) State and explain what is observed at P.

...........................................................................................................................................

...........................................................................................................................................

.......................................................................................................................................[2]

(ii) State and explain the effect on the answer to (c)(i) if the apparatus is changed so that, separately,

1. the phase difference between the waves at D1 and at D2 is 180°,

....................................................................................................................................

....................................................................................................................................

....................................................................................................................................

2. the intensity of the wave from D1 is less than the intensity of that from D2.

....................................................................................................................................

....................................................................................................................................

.................................................................................................................................... [2]

[Total: 10]

Page 91: PHYSICS 9702/11 READ THESE INSTRUCTIONS FIRSTmaxpapers.com/wp-content/uploads/2012/11/9702_s17… ·  · 2017-10-17PHYSICS 9702/11 Paper 1 Multiple Choice May/June 2017 1 hour 15

15

9702/22/M/J/17© UCLES 2017 [Turn over

7 (a) Define electromotive force (e.m.f.) of a cell.

...................................................................................................................................................

...............................................................................................................................................[1]

(b) A cell C of e.m.f. 1.50 V and internal resistance 0.200 Ω is connected in series with resistors X and Y, as shown in Fig. 7.1.

X

A B

C

1.50 V

0.200 Ω

Y

Fig. 7.1

The resistance of X is constant and the resistance of Y can be varied.

(i) The resistance of Y is varied from 0 to 8.00 Ω.

State and explain the variation in the potential difference (p.d.) between points A and B (terminal p.d. across C). Numerical values are not required.

...........................................................................................................................................

...........................................................................................................................................

...........................................................................................................................................

.......................................................................................................................................[3]

(ii) The resistance of Y is set at 6.00 Ω. The current in the circuit is 0.180 A.

Calculate

1. the resistance of X,

resistance = ....................................................... Ω [2]

Page 92: PHYSICS 9702/11 READ THESE INSTRUCTIONS FIRSTmaxpapers.com/wp-content/uploads/2012/11/9702_s17… ·  · 2017-10-17PHYSICS 9702/11 Paper 1 Multiple Choice May/June 2017 1 hour 15

16

9702/22/M/J/17© UCLES 2017

2. the p.d. between points A and B,

p.d. = ....................................................... V [2]

3. the efficiency of the cell.

efficiency = ...........................................................[2]

[Total: 10]

8 (a) Describe two differences between the decay of a nucleus that emits a β– particle and the decay of a nucleus that emits a β+ particle.

1. ...............................................................................................................................................

...................................................................................................................................................

2. ...............................................................................................................................................

...................................................................................................................................................[2]

(b) In a simple quark model there are three types of quark. State the composition of the proton and of the neutron in terms of these three quarks.

proton: ......................................................................................................................................

neutron: .................................................................................................................................... [1]

[Total: 3]

Permission to reproduce items where third-party owned material protected by copyright is included has been sought and cleared where possible. Every reasonable effort has been made by the publisher (UCLES) to trace copyright holders, but if any items requiring clearance have unwittingly been included, the publisher will be pleased to make amends at the earliest possible opportunity.

To avoid the issue of disclosure of answer-related information to candidates, all copyright acknowledgements are reproduced online in the Cambridge International Examinations Copyright Acknowledgements Booklet. This is produced for each series of examinations and is freely available to download at www.cie.org.uk after the live examination series.

Cambridge International Examinations is part of the Cambridge Assessment Group. Cambridge Assessment is the brand name of University of Cambridge Local Examinations Syndicate (UCLES), which is itself a department of the University of Cambridge.

Page 93: PHYSICS 9702/11 READ THESE INSTRUCTIONS FIRSTmaxpapers.com/wp-content/uploads/2012/11/9702_s17… ·  · 2017-10-17PHYSICS 9702/11 Paper 1 Multiple Choice May/June 2017 1 hour 15

This document consists of 16 printed pages.

DC (NH/FD) 127438/3© UCLES 2017 [Turn over

*6797582506*

PHYSICS 9702/23Paper 2 AS Level Structured Questions May/June 2017 1 hour 15 minutesCandidates answer on the Question Paper.No Additional Materials are required.

READ THESE INSTRUCTIONS FIRST

Write your Centre number, candidate number and name on all the work you hand in.Write in dark blue or black pen.You may use an HB pencil for any diagrams or graphs.Do not use staples, paper clips, glue or correction fluid.DO NOT WRITE IN ANY BARCODES.

Answer all questions.

Electronic calculators may be used.You may lose marks if you do not show your working or if you do not use appropriate units.

At the end of the examination, fasten all your work securely together.The number of marks is given in brackets [ ] at the end of each question or part question.

Cambridge International ExaminationsCambridge International Advanced Subsidiary and Advanced Level

Page 94: PHYSICS 9702/11 READ THESE INSTRUCTIONS FIRSTmaxpapers.com/wp-content/uploads/2012/11/9702_s17… ·  · 2017-10-17PHYSICS 9702/11 Paper 1 Multiple Choice May/June 2017 1 hour 15

2

9702/23/M/J/17© UCLES 2017

Data

speed of light in free space c = 3.00 × 108 m s−1

permeability of free space μ0 = 4π × 10−7 H m−1

permittivity of free space ε0 = 8.85 × 10−12 F m−1

( 14πε0

= 8.99 × 109 m F−1) elementary charge e = 1.60 × 10−19 C

the Planck constant h = 6.63 × 10−34 J s

unified atomic mass unit 1 u = 1.66 × 10−27 kg

rest mass of electron me = 9.11 × 10−31 kg

rest mass of proton mp = 1.67 × 10−27 kg

molar gas constant R = 8.31 J K−1 mol−1

the Avogadro constant NA = 6.02 × 1023 mol−1

the Boltzmann constant k = 1.38 × 10−23 J K−1

gravitational constant G = 6.67 × 10−11 N m2 kg−2

acceleration of free fall g = 9.81 m s−2

Page 95: PHYSICS 9702/11 READ THESE INSTRUCTIONS FIRSTmaxpapers.com/wp-content/uploads/2012/11/9702_s17… ·  · 2017-10-17PHYSICS 9702/11 Paper 1 Multiple Choice May/June 2017 1 hour 15

3

9702/23/M/J/17© UCLES 2017 [Turn over

Formulae

uniformly accelerated motion s = ut + 12 at 2

v2 = u2 + 2as

work done on/by a gas W = pΔV

gravitational potential φ =− Gmr

hydrostatic pressure p = ρgh

pressure of an ideal gas p = 13 NmV ⟨c2⟩

simple harmonic motion a =−ω 2x

velocity of particle in s.h.m. v = v0 cos ωt v = ± ω ( )x x0

2 2-

Doppler effect fo = fsv

v ± vs

electric potential V = Q4πε0r

capacitors in series 1/C = 1/C1 + 1/C2 + . . .

capacitors in parallel C = C1 + C2 + . . .

energy of charged capacitor W = 12 QV

electric current I = Anvq

resistors in series R = R1 + R2 + . . .

resistors in parallel 1/R = 1/R1 + 1/R2 + . . .

Hall voltage VH = BIntq

alternating current/voltage x = x0 sin ωt

radioactive decay x = x0exp(−λt )

decay constant λ = 0.693t 1

2

Page 96: PHYSICS 9702/11 READ THESE INSTRUCTIONS FIRSTmaxpapers.com/wp-content/uploads/2012/11/9702_s17… ·  · 2017-10-17PHYSICS 9702/11 Paper 1 Multiple Choice May/June 2017 1 hour 15

4

9702/23/M/J/17© UCLES 2017

Answer all the questions in the spaces provided.

1 (a) Two forces, with magnitudes 5.0 N and 12 N, act from the same point on an object. Calculate the magnitude of the resultant force R for the forces acting

(i) in opposite directions,

R = ....................................................... N [1]

(ii) at right angles to each other.

R = ....................................................... N [1]

(b) An object X rests on a smooth horizontal surface. Two horizontal forces act on X as shown in Fig. 1.1.

18 N

55 N

115°

X

Fig. 1.1 (not to scale)

A force of 55 N is applied to the right. A force of 18 N is applied at an angle of 115° to the direction of the 55 N force.

Page 97: PHYSICS 9702/11 READ THESE INSTRUCTIONS FIRSTmaxpapers.com/wp-content/uploads/2012/11/9702_s17… ·  · 2017-10-17PHYSICS 9702/11 Paper 1 Multiple Choice May/June 2017 1 hour 15

5

9702/23/M/J/17© UCLES 2017 [Turn over

(i) Use the resolution of forces or a scale diagram to show that the magnitude of the resultant force acting on X is 65 N.

[2]

(ii) Determine the angle between the resultant force and the 55 N force.

angle = ........................................................ ° [2]

(c) A third force of 80 N is now applied to X in the opposite direction to the resultant force in (b).

The mass of X is 2.7 kg.

Calculate the magnitude of the acceleration of X.

acceleration = ..................................................m s–2 [3]

[Total: 9]

Page 98: PHYSICS 9702/11 READ THESE INSTRUCTIONS FIRSTmaxpapers.com/wp-content/uploads/2012/11/9702_s17… ·  · 2017-10-17PHYSICS 9702/11 Paper 1 Multiple Choice May/June 2017 1 hour 15

6

9702/23/M/J/17© UCLES 2017

2 (a) StateNewton’ssecondlawofmotion.

...................................................................................................................................................

...............................................................................................................................................[1]

(b) A constant resultant force F acts on an object A. The variation with time t of the velocity v for the motion of A is shown in Fig. 2.1.

04.0

5.0

6.0

7.0

8.0

v / m s–1

9.0

1.0 2.0 3.0 4.0t / s

Fig. 2.1

The mass of A is 840 g.

Calculate, for the time t = 0 to t = 4.0 s,

(i) the change in momentum of A,

change in momentum = ............................................. kg m s–1 [2]

(ii) the force F.

F = ....................................................... N [1]

Page 99: PHYSICS 9702/11 READ THESE INSTRUCTIONS FIRSTmaxpapers.com/wp-content/uploads/2012/11/9702_s17… ·  · 2017-10-17PHYSICS 9702/11 Paper 1 Multiple Choice May/June 2017 1 hour 15

7

9702/23/M/J/17© UCLES 2017 [Turn over

(c) The force F is removed at t = 4.0 s. Object A continues at constant velocity before colliding with an object B, as illustrated in Fig. 2.2.

840 g

A

730 g at rest

B

Fig. 2.2

Object B is initially at rest. The mass of B is 730 g. The objects A and B join together and have a velocity of 4.7 m s–1.

(i) By calculation, show that the changes in momentum of A and of B during the collision are equal and opposite.

[2]

(ii) Explain how the answers obtained in (i) supportNewton’sthirdlaw.

...........................................................................................................................................

...........................................................................................................................................

...........................................................................................................................................

.......................................................................................................................................[2]

(iii) By reference to the speeds of A and B, explain whether the collision is elastic.

...........................................................................................................................................

.......................................................................................................................................[1]

[Total: 9]

Page 100: PHYSICS 9702/11 READ THESE INSTRUCTIONS FIRSTmaxpapers.com/wp-content/uploads/2012/11/9702_s17… ·  · 2017-10-17PHYSICS 9702/11 Paper 1 Multiple Choice May/June 2017 1 hour 15

8

9702/23/M/J/17© UCLES 2017

3 (a) Define electric field strength.

...................................................................................................................................................

...............................................................................................................................................[1]

(b) An electron is accelerated from point A to point B by a uniform electric field, as illustrated in Fig. 3.1.

A electron

electric field

B

Fig. 3.1

The distance between A and B is 12 mm. The velocity of the electron at A is 2.5 km s–1 and at B is 18 Mm s–1.

Calculate

(i) the acceleration of the electron,

acceleration = ..................................................m s–2 [2]

(ii) the change in kinetic energy of the electron,

change in kinetic energy = ........................................................J [3]

Page 101: PHYSICS 9702/11 READ THESE INSTRUCTIONS FIRSTmaxpapers.com/wp-content/uploads/2012/11/9702_s17… ·  · 2017-10-17PHYSICS 9702/11 Paper 1 Multiple Choice May/June 2017 1 hour 15

9

9702/23/M/J/17© UCLES 2017 [Turn over

(iii) the electric field strength.

electric field strength = .................................................V m–1 [3]

(c) An α-particle moves from A to B in the electric field in (b).

Describe and explain how the change in the kinetic energy of the α-particle compares with that of the electron. Numerical values are not required.

...................................................................................................................................................

...................................................................................................................................................

...................................................................................................................................................

...................................................................................................................................................

...............................................................................................................................................[3]

[Total: 12]

Page 102: PHYSICS 9702/11 READ THESE INSTRUCTIONS FIRSTmaxpapers.com/wp-content/uploads/2012/11/9702_s17… ·  · 2017-10-17PHYSICS 9702/11 Paper 1 Multiple Choice May/June 2017 1 hour 15

10

9702/23/M/J/17© UCLES 2017

4 A spring is supported so that it hangs vertically, as shown in Fig. 4.1.

spring

mass M

Fig. 4.1

Different masses are attached to the lower end of the spring. The extension x of the spring is measured for each mass M. The variation with x of M is shown in Fig. 4.2.

00

50

100

150

40 80 120 160x / mm

M / g

200

Fig. 4.2

(a) StateandexplainwhetherthespringobeysHooke’slaw.

...................................................................................................................................................

...............................................................................................................................................[1]

(b) State the form of energy stored in the spring due to the addition of the masses.

...............................................................................................................................................[1]

(c) Describe how to determine whether the extension of the spring is elastic.

...................................................................................................................................................

...............................................................................................................................................[1]

Page 103: PHYSICS 9702/11 READ THESE INSTRUCTIONS FIRSTmaxpapers.com/wp-content/uploads/2012/11/9702_s17… ·  · 2017-10-17PHYSICS 9702/11 Paper 1 Multiple Choice May/June 2017 1 hour 15

11

9702/23/M/J/17© UCLES 2017 [Turn over

(d) Calculate the work done on the spring as it is extended from x = 40.0 mm to x = 160 mm.

work done = ........................................................J [3]

[Total: 6]

Page 104: PHYSICS 9702/11 READ THESE INSTRUCTIONS FIRSTmaxpapers.com/wp-content/uploads/2012/11/9702_s17… ·  · 2017-10-17PHYSICS 9702/11 Paper 1 Multiple Choice May/June 2017 1 hour 15

12

9702/23/M/J/17© UCLES 2017

5 (a) A diffraction grating is used to determine the wavelength of light.

(i) Describe the diffraction of light at a diffraction grating.

...........................................................................................................................................

...........................................................................................................................................

.......................................................................................................................................[2]

(ii) By reference to interference, explain

1. the zero order maximum,

....................................................................................................................................

....................................................................................................................................

....................................................................................................................................

2. the first order maximum.

....................................................................................................................................

.................................................................................................................................... [3]

(b) A diffraction grating is used with different wavelengths of light. The angle θ of the second order maximum is measured for each wavelength. The variation with wavelength λ of sin θ is shown in Fig. 5.1.

3000.10

0.20

0.30

0.40

0.50

0.60

350 400 450 500 / nmλ

sinθ

550

Fig. 5.1

Page 105: PHYSICS 9702/11 READ THESE INSTRUCTIONS FIRSTmaxpapers.com/wp-content/uploads/2012/11/9702_s17… ·  · 2017-10-17PHYSICS 9702/11 Paper 1 Multiple Choice May/June 2017 1 hour 15

13

9702/23/M/J/17© UCLES 2017 [Turn over

(i) Determine the gradient of the line shown in Fig. 5.1.

gradient = ...........................................................[2]

(ii) Use the gradient determined in (i) to calculate the slit separation d of the diffraction grating.

d = .......................................................m [2]

(iii) On Fig. 5.1, sketch a line to show the results that would be obtained for the first order maxima. [1]

[Total: 10]

Page 106: PHYSICS 9702/11 READ THESE INSTRUCTIONS FIRSTmaxpapers.com/wp-content/uploads/2012/11/9702_s17… ·  · 2017-10-17PHYSICS 9702/11 Paper 1 Multiple Choice May/June 2017 1 hour 15

14

9702/23/M/J/17© UCLES 2017

6 (a) Describe the I–V characteristic of

(i) a metallic conductor at constant temperature,

...........................................................................................................................................

.......................................................................................................................................[1]

(ii) a semiconductor diode.

...........................................................................................................................................

...........................................................................................................................................

.......................................................................................................................................[2]

(b) Two identical filament lamps are connected in series and then in parallel to a battery of electromotive force (e.m.f.) 12 V and negligible internal resistance, as shown in Fig. 6.1a and Fig. 6.1b.

12 V 12 V

Fig. 6.1a Fig. 6.1b

The I–V characteristic of each lamp is shown in Fig. 6.2.

00

2.0

4.0

6.0

2.0 4.0 6.0 8.0V / V

I / A

10.0 12.0

Fig. 6.2

Page 107: PHYSICS 9702/11 READ THESE INSTRUCTIONS FIRSTmaxpapers.com/wp-content/uploads/2012/11/9702_s17… ·  · 2017-10-17PHYSICS 9702/11 Paper 1 Multiple Choice May/June 2017 1 hour 15

15

9702/23/M/J/17© UCLES 2017 [Turn over

(i) Use the information shown in Fig. 6.2 to determine the current through the battery in

1. the circuit of Fig. 6.1a,

current = .............................................................A

2. the circuit of Fig. 6.1b.

current = .............................................................A[3]

(ii) Calculate the total resistance in

1. the circuit of Fig. 6.1a,

resistance = ............................................................Ω

2. the circuit of Fig. 6.1b.

resistance = ............................................................Ω[3]

(iii) Calculate the ratio

power dissipated in a lamp in the circuit of Fig. 6.1apower dissipated in a lamp in the circuit of Fig. 6.1b .

ratio = ...........................................................[2]

[Total: 11]

Page 108: PHYSICS 9702/11 READ THESE INSTRUCTIONS FIRSTmaxpapers.com/wp-content/uploads/2012/11/9702_s17… ·  · 2017-10-17PHYSICS 9702/11 Paper 1 Multiple Choice May/June 2017 1 hour 15

16

9702/23/M/J/17© UCLES 2017

7 (a) The following particles are used to describe the structure of an atom.

electron neutron proton quark

Underline the fundamental particles in the above list. [1]

(b) The following equation represents the decay of a nucleus of 62

07Co to form nucleus Q by

β– emission.

62

07Co → A

BQ + β– + x

(i) Complete Fig. 7.1.

value

A

B

Fig. 7.1 [1]

(ii) State the name of the particle x.

.......................................................................................................................................[1]

[Total: 3]

Permission to reproduce items where third-party owned material protected by copyright is included has been sought and cleared where possible. Every reasonable effort has been made by the publisher (UCLES) to trace copyright holders, but if any items requiring clearance have unwittingly been included, the publisher will be pleased to make amends at the earliest possible opportunity.

To avoid the issue of disclosure of answer-related information to candidates, all copyright acknowledgements are reproduced online in the Cambridge International Examinations Copyright Acknowledgements Booklet. This is produced for each series of examinations and is freely available to download at www.cie.org.uk after the live examination series.

Cambridge International Examinations is part of the Cambridge Assessment Group. Cambridge Assessment is the brand name of University of Cambridge Local Examinations Syndicate (UCLES), which is itself a department of the University of Cambridge.

Page 109: PHYSICS 9702/11 READ THESE INSTRUCTIONS FIRSTmaxpapers.com/wp-content/uploads/2012/11/9702_s17… ·  · 2017-10-17PHYSICS 9702/11 Paper 1 Multiple Choice May/June 2017 1 hour 15

This document consists of 12 printed pages.

DC (ST/SW) 127161/2© UCLES 2017 [Turn over

Cambridge International ExaminationsCambridge International Advanced Subsidiary and Advanced Level

*8585515756*

PHYSICS 9702/31Paper 3 Advanced Practical Skills 1 May/June 2017 2 hoursCandidates answer on the Question Paper.Additional Materials: As listed in the Confidential Instructions.

READ THESE INSTRUCTIONS FIRST

Write your Centre number, candidate number and name on all the work you hand in.Write in dark blue or black pen.You may use an HB pencil for any diagrams or graphs.Do not use staples, paper clips, glue or correction fluid.DO NOT WRITE IN ANY BARCODES.

Answer both questions.You will be allowed to work with the apparatus for a maximum of one hour for each question.You are expected to record all your observations as soon as these observations are made, and to plan the presentation of the records so that it is not necessary to make a fair copy of them. You are reminded of the need for good English and clear presentation in your answers.

Electronic calculators may be used.You may lose marks if you do not show your working or if you do not use appropriate units.

Additional answer paper and graph paper should be used only if it becomes necessary to do so.

At the end of the examination, fasten all your work securely together.The number of marks is given in brackets [ ] at the end of each question or part question.

For Examiner’s Use

1

2

Total

Page 110: PHYSICS 9702/11 READ THESE INSTRUCTIONS FIRSTmaxpapers.com/wp-content/uploads/2012/11/9702_s17… ·  · 2017-10-17PHYSICS 9702/11 Paper 1 Multiple Choice May/June 2017 1 hour 15

2

9702/31/M/J/17© UCLES 2017

You may not need to use all of the materials provided.

1 In this experiment, you will investigate an electrical circuit.

(a) Set up the circuit shown in Fig. 1.1.

A

crocodileclip

wiremetre rule

power supply

crocodileclip

x

Fig. 1.1

The distance x between the crocodile clips should be approximately 40 cm.

(b) (i) Measure and record x.

x = ...................................................... (ii) Close the switch. (iii) Record the ammeter reading I1.

I1 = ..................................................[1]

(iv) Open the switch.

Page 111: PHYSICS 9702/11 READ THESE INSTRUCTIONS FIRSTmaxpapers.com/wp-content/uploads/2012/11/9702_s17… ·  · 2017-10-17PHYSICS 9702/11 Paper 1 Multiple Choice May/June 2017 1 hour 15

3

9702/31/M/J/17© UCLES 2017 [Turn over

(c) (i) Connect an additional lead L to the circuit as shown in Fig. 1.2.

A

L

x

Fig. 1.2 (ii) Close the switch.

(iii) Record the ammeter reading I2.

I2 = ..................................................[1]

(iv) Open the switch.

(v) Remove L. The circuit is now as shown in Fig. 1.1.

Page 112: PHYSICS 9702/11 READ THESE INSTRUCTIONS FIRSTmaxpapers.com/wp-content/uploads/2012/11/9702_s17… ·  · 2017-10-17PHYSICS 9702/11 Paper 1 Multiple Choice May/June 2017 1 hour 15

4

9702/31/M/J/17© UCLES 2017

(d) Increase x and repeat (b) and (c) until you have six sets of readings of x, I1 and I2.

Record your values in a table. Include values of I2I1

in your table.

[10] (e) (i) Plot a graph of

I2I1

on the y-axis against x on the x-axis. [3]

(ii) Draw the straight line of best fit. [1]

(iii) Determine the gradient and y-intercept of this line.

gradient = ......................................................

y-intercept = ...................................................... [2]

Page 113: PHYSICS 9702/11 READ THESE INSTRUCTIONS FIRSTmaxpapers.com/wp-content/uploads/2012/11/9702_s17… ·  · 2017-10-17PHYSICS 9702/11 Paper 1 Multiple Choice May/June 2017 1 hour 15

5

9702/31/M/J/17© UCLES 2017 [Turn over

Page 114: PHYSICS 9702/11 READ THESE INSTRUCTIONS FIRSTmaxpapers.com/wp-content/uploads/2012/11/9702_s17… ·  · 2017-10-17PHYSICS 9702/11 Paper 1 Multiple Choice May/June 2017 1 hour 15

6

9702/31/M/J/17© UCLES 2017

(f) It is suggested that the quantities I1, I2 and x are related by the equationI2I1

= Px + Q

where P and Q are constants.

Using your answers in (e)(iii), determine values for P and Q. Give appropriate units.

P = ......................................................

Q = ......................................................[2]

[Total: 20]

Page 115: PHYSICS 9702/11 READ THESE INSTRUCTIONS FIRSTmaxpapers.com/wp-content/uploads/2012/11/9702_s17… ·  · 2017-10-17PHYSICS 9702/11 Paper 1 Multiple Choice May/June 2017 1 hour 15

7

9702/31/M/J/17© UCLES 2017 [Turn over

You may not need to use all of the materials provided.

2 In this experiment, you will investigate the motion of oscillating table tennis balls.

(a) Tape each ball to a length of string. Ensure the total length of the string and ball is 35.0 cm, as shown in Fig. 2.1.

35.0 cm

stringtape

ball

Fig. 2.1

(b) (i) Set up the apparatus as shown in Fig. 2.2.

bench

balls

stand

strings

stand

boss boss

wooden rod

Fig. 2.2

Page 116: PHYSICS 9702/11 READ THESE INSTRUCTIONS FIRSTmaxpapers.com/wp-content/uploads/2012/11/9702_s17… ·  · 2017-10-17PHYSICS 9702/11 Paper 1 Multiple Choice May/June 2017 1 hour 15

8

9702/31/M/J/17© UCLES 2017

(ii) Pull one of the balls towards you through a short distance.

Release the ball and determine the time for five complete oscillations.

time = ................................................... s

Repeat for the other ball.

time = ................................................... s[1]

(iii) Remove the balls and strings from the wooden rod.

(c) (i) Tape the shorter wooden block to one of the balls as shown in Fig. 2.3. Tape should be used on opposite sides of the block and the ball.

x

marktape

wooden block

Fig. 2.3

The distance between the end of the string loop and the mark around the wooden block is x.

(ii) Measure and record x.

x = ..................................................[1]

Page 117: PHYSICS 9702/11 READ THESE INSTRUCTIONS FIRSTmaxpapers.com/wp-content/uploads/2012/11/9702_s17… ·  · 2017-10-17PHYSICS 9702/11 Paper 1 Multiple Choice May/June 2017 1 hour 15

9

9702/31/M/J/17© UCLES 2017 [Turn over

(iii) Estimate the percentage uncertainty in your value of x.

percentage uncertainty = ..................................................[1]

(d) (i) Set up the apparatus as shown in Fig. 2.4.

Fig. 2.4 (ii) Pull both balls towards you.

Release the balls at the same time and watch the movement. The two balls will move backwards and forwards becoming out of phase. After a time they will be back in phase so that they move towards you together. The ball with the block attached completes n oscillations in this time.

Page 118: PHYSICS 9702/11 READ THESE INSTRUCTIONS FIRSTmaxpapers.com/wp-content/uploads/2012/11/9702_s17… ·  · 2017-10-17PHYSICS 9702/11 Paper 1 Multiple Choice May/June 2017 1 hour 15

10

9702/31/M/J/17© UCLES 2017

(e) (i) Repeat (d)(ii) and record n.

n = ..................................................[2]

(ii) Calculate ( )n

n 12

2+ .

( )n

n 12

2+ = ..................................................[1]

(f) Using the longer wooden block, repeat (c)(i), (c)(ii), (d) and (e).

x = ......................................................

n = ......................................................

( )n

n 12

2+ = ......................................................[3]

Page 119: PHYSICS 9702/11 READ THESE INSTRUCTIONS FIRSTmaxpapers.com/wp-content/uploads/2012/11/9702_s17… ·  · 2017-10-17PHYSICS 9702/11 Paper 1 Multiple Choice May/June 2017 1 hour 15

11

9702/31/M/J/17© UCLES 2017 [Turn over

(g) It is suggested that the relationship between n and x is

( )n

n 12

2+ = k x

where k is a constant. (i) Using your data, calculate two values of k.

first value of k = ......................................................

second value of k = ......................................................[1]

(ii) Explain whether your results in (g)(i) support the suggested relationship.

.................................................................................................................................

.................................................................................................................................

.................................................................................................................................

............................................................................................................................. [1]

(h) The effective length of the pendulum formed by the ball and string is L. Use your second value of k to calculate L using the relationship

k = L1 .

Give your answer to three significant figures.

L = ..................................................[1]

Page 120: PHYSICS 9702/11 READ THESE INSTRUCTIONS FIRSTmaxpapers.com/wp-content/uploads/2012/11/9702_s17… ·  · 2017-10-17PHYSICS 9702/11 Paper 1 Multiple Choice May/June 2017 1 hour 15

12

9702/31/M/J/17© UCLES 2017

(i) (i) Describe four sources of uncertainty or limitations of the procedure for this experiment.

1. .............................................................................................................................

.................................................................................................................................

2. .............................................................................................................................

.................................................................................................................................

3. .............................................................................................................................

.................................................................................................................................

4. .............................................................................................................................

.................................................................................................................................[4]

(ii) Describe four improvements that could be made to this experiment. You may suggest the use of other apparatus or different procedures.

1. .............................................................................................................................

.................................................................................................................................

2. .............................................................................................................................

.................................................................................................................................

3. .............................................................................................................................

.................................................................................................................................

4. .............................................................................................................................

.................................................................................................................................[4]

[Total: 20]

Permission to reproduce items where third-party owned material protected by copyright is included has been sought and cleared where possible. Every reasonable effort has been made by the publisher (UCLES) to trace copyright holders, but if any items requiring clearance have unwittingly been included, the publisher will be pleased to make amends at the earliest possible opportunity.

To avoid the issue of disclosure of answer-related information to candidates, all copyright acknowledgements are reproduced online in the Cambridge International Examinations Copyright Acknowledgements Booklet. This is produced for each series of examinations and is freely available to download at www.cie.org.uk after the live examination series.

Cambridge International Examinations is part of the Cambridge Assessment Group. Cambridge Assessment is the brand name of University of Cambridge Local Examinations Syndicate (UCLES), which is itself a department of the University of Cambridge.

Page 121: PHYSICS 9702/11 READ THESE INSTRUCTIONS FIRSTmaxpapers.com/wp-content/uploads/2012/11/9702_s17… ·  · 2017-10-17PHYSICS 9702/11 Paper 1 Multiple Choice May/June 2017 1 hour 15

This document consists of 11 printed pages and 1 blank page.

DC (LK/JG) 127169/4© UCLES 2017 [Turn over

*7650054384*

PHYSICS 9702/32Paper 3 Advanced Practical Skills 2 May/June 2017 2 hoursCandidates answer on the Question Paper.Additional Materials: As listed in the Confidential Instructions.

READ THESE INSTRUCTIONS FIRST

Write your Centre number, candidate number and name on all the work you hand in.Write in dark blue or black pen.You may use an HB pencil for any diagrams or graphs.Do not use staples, paper clips, glue or correction fluid.DO NOT WRITE IN ANY BARCODES.

Answer both questions.You will be allowed to work with the apparatus for a maximum of one hour for each question.You are expected to record all your observations as soon as these observations are made, and to plan the presentation of the records so that it is not necessary to make a fair copy of them.You are reminded of the need for good English and clear presentation in your answers.

Electronic calculators may be used.You may lose marks if you do not show your working or if you do not use appropriate units.

Additional answer paper and graph paper should be used only if it becomes necessary to do so.

At the end of the examination, fasten all your work securely together.The number of marks is given in brackets [ ] at the end of each question or part question.

Cambridge International ExaminationsCambridge International Advanced Subsidiary and Advanced Level

For Examiner’s Use

1

2

Total

Page 122: PHYSICS 9702/11 READ THESE INSTRUCTIONS FIRSTmaxpapers.com/wp-content/uploads/2012/11/9702_s17… ·  · 2017-10-17PHYSICS 9702/11 Paper 1 Multiple Choice May/June 2017 1 hour 15

2

9702/32/M/J/17© UCLES 2017

You may not need to use all of the materials provided.

1 In this experiment, you will investigate an electrical circuit.

(a) (i) Assemble the circuit shown in Fig. 1.1.

VV

3 V

AB

C

screw

wire

tapewooden strip

Fig. 1.1

The two resistors are identical. A, B and C are crocodile clips. Connect C to the screw.

(ii) Connect A to the wire at a distance p of approximately 25 cm from the screw, as shown in Fig. 1.2.

(iii) Close the switch.

Page 123: PHYSICS 9702/11 READ THESE INSTRUCTIONS FIRSTmaxpapers.com/wp-content/uploads/2012/11/9702_s17… ·  · 2017-10-17PHYSICS 9702/11 Paper 1 Multiple Choice May/June 2017 1 hour 15

3

9702/32/M/J/17© UCLES 2017 [Turn over

(iv) Position B on the other side of the screw so that the two voltmeter readings have the same value V.

The distance between the screw and B is q, as shown in Fig. 1.2.

VV

BA

p q

Fig. 1.2

(v) Measure and record the distances p and q. Record V.

p = .......................................................

q = .......................................................

V = ....................................................... [2] (vi) Open the switch.

Page 124: PHYSICS 9702/11 READ THESE INSTRUCTIONS FIRSTmaxpapers.com/wp-content/uploads/2012/11/9702_s17… ·  · 2017-10-17PHYSICS 9702/11 Paper 1 Multiple Choice May/June 2017 1 hour 15

4

9702/32/M/J/17© UCLES 2017

(b) Change p and repeat (a)(iii), (a)(iv), (a)(v) and (a)(vi) until you have six sets of values of p, q and V.

Record your results in a table. Include values of p

1 and q1 in your table.

[10]

(c) (i) Plot a graph of q1 on the y-axis against p

1 on the x-axis. [3]

(ii) Draw the straight line of best fit. [1]

(iii) Determine the gradient and y-intercept of this line.

gradient = .......................................................

y-intercept = ....................................................... [2]

Page 125: PHYSICS 9702/11 READ THESE INSTRUCTIONS FIRSTmaxpapers.com/wp-content/uploads/2012/11/9702_s17… ·  · 2017-10-17PHYSICS 9702/11 Paper 1 Multiple Choice May/June 2017 1 hour 15

5

9702/32/M/J/17© UCLES 2017 [Turn over

Page 126: PHYSICS 9702/11 READ THESE INSTRUCTIONS FIRSTmaxpapers.com/wp-content/uploads/2012/11/9702_s17… ·  · 2017-10-17PHYSICS 9702/11 Paper 1 Multiple Choice May/June 2017 1 hour 15

6

9702/32/M/J/17© UCLES 2017

(d) It is suggested that the quantities q and p are related by the equation

q pa b1 = +

where a and b are constants.

Use your answers in (c)(iii) to determine the values of a and b. Give appropriate units.

a = .......................................................

b = ....................................................... [2]

[Total: 20]

Page 127: PHYSICS 9702/11 READ THESE INSTRUCTIONS FIRSTmaxpapers.com/wp-content/uploads/2012/11/9702_s17… ·  · 2017-10-17PHYSICS 9702/11 Paper 1 Multiple Choice May/June 2017 1 hour 15

7

9702/32/M/J/17© UCLES 2017 [Turn over

BLANK PAGE

Page 128: PHYSICS 9702/11 READ THESE INSTRUCTIONS FIRSTmaxpapers.com/wp-content/uploads/2012/11/9702_s17… ·  · 2017-10-17PHYSICS 9702/11 Paper 1 Multiple Choice May/June 2017 1 hour 15

8

9702/32/M/J/17© UCLES 2017

You may not need to use all of the materials provided.

2 In this experiment, you will investigate the rotational oscillation of a combination of springs.

(a) (i) You are provided with two joined springs and three joined springs. Using the two joined springs, set up the apparatus as shown in Fig. 2.1.

h1

clamp

springs

stand

boss

bench

mass hanger

Fig. 2.1

(ii) Measure and record the height h1 of the bottom of the mass hanger above the bench, as shown in Fig. 2.1.

h1 = .............................................. m [1]

(iii) Add the 100 g mass to the mass hanger. Measure and record the height h2 of the bottom of the mass hanger above the bench.

h2 = .............................................. m [1]

(iv) Estimate the percentage uncertainty in your value of h2.

percentage uncertainty = .................................................. [1]

Page 129: PHYSICS 9702/11 READ THESE INSTRUCTIONS FIRSTmaxpapers.com/wp-content/uploads/2012/11/9702_s17… ·  · 2017-10-17PHYSICS 9702/11 Paper 1 Multiple Choice May/June 2017 1 hour 15

9

9702/32/M/J/17© UCLES 2017 [Turn over

(b) (i) Calculate the spring constant k for the combination, using the expression

( )k h hmg–1 2

=

where m = 0.100 kg and g = 9.81N kg–1.

k = ............................................... N m–1 [1]

(ii) Justify the number of significant figures you have given for your value of k.

..................................................................................................................................

..................................................................................................................................

.............................................................................................................................. [1]

(c) (i) Use small pieces of adhesive tape to reduce movement at the joints between components, as shown in Fig. 2.2.

clamp

tape

springs

mass hanger

100 g mass

Fig. 2.2

Page 130: PHYSICS 9702/11 READ THESE INSTRUCTIONS FIRSTmaxpapers.com/wp-content/uploads/2012/11/9702_s17… ·  · 2017-10-17PHYSICS 9702/11 Paper 1 Multiple Choice May/June 2017 1 hour 15

10

9702/32/M/J/17© UCLES 2017

(ii) Rotate the mass hanger and mass through one turn and release them. The masses make rotational oscillations, as shown in Fig. 2.3.

Fig. 2.3

(iii) Take measurements to find the period T of the rotational oscillations.

T = ............................................... s [2]

(d) Repeat (a)(ii), (a)(iii), (b)(i) and (c) using the three joined springs.

h1 = ................................................... m

h2 = ................................................... m

k = ............................................. N m–1

T = .................................................... s [3]

Page 131: PHYSICS 9702/11 READ THESE INSTRUCTIONS FIRSTmaxpapers.com/wp-content/uploads/2012/11/9702_s17… ·  · 2017-10-17PHYSICS 9702/11 Paper 1 Multiple Choice May/June 2017 1 hour 15

11

9702/32/M/J/17© UCLES 2017 [Turn over

(e) It is suggested that the relationship between T and k is

TkC3

2=

where C is a constant.

(i) Using your data, calculate two values of C.

first value of C = .......................................................

second value of C = ....................................................... [1]

(ii) Explain whether your results in (e)(i) support the suggested relationship.

..................................................................................................................................

..................................................................................................................................

..................................................................................................................................

.............................................................................................................................. [1]

Page 132: PHYSICS 9702/11 READ THESE INSTRUCTIONS FIRSTmaxpapers.com/wp-content/uploads/2012/11/9702_s17… ·  · 2017-10-17PHYSICS 9702/11 Paper 1 Multiple Choice May/June 2017 1 hour 15

12

9702/32/M/J/17© UCLES 2017

Permission to reproduce items where third-party owned material protected by copyright is included has been sought and cleared where possible. Every reasonable effort has been made by the publisher (UCLES) to trace copyright holders, but if any items requiring clearance have unwittingly been included, the publisher will be pleased to make amends at the earliest possible opportunity.

To avoid the issue of disclosure of answer-related information to candidates, all copyright acknowledgements are reproduced online in the Cambridge International Examinations Copyright Acknowledgements Booklet. This is produced for each series of examinations and is freely available to download at www.cie.org.uk after the live examination series.

Cambridge International Examinations is part of the Cambridge Assessment Group. Cambridge Assessment is the brand name of University of Cambridge Local Examinations Syndicate (UCLES), which is itself a department of the University of Cambridge.

(f) (i) Describe four sources of uncertainty or limitations of the procedure for this experiment.

1. ..............................................................................................................................

..................................................................................................................................

2. ..............................................................................................................................

..................................................................................................................................

3. ..............................................................................................................................

..................................................................................................................................

4. ..............................................................................................................................

.................................................................................................................................. [4]

(ii) Describe four improvements that could be made to this experiment. You may suggest the use of other apparatus or different procedures.

1. ..............................................................................................................................

..................................................................................................................................

2. ..............................................................................................................................

..................................................................................................................................

3. ..............................................................................................................................

..................................................................................................................................

4. ..............................................................................................................................

.................................................................................................................................. [4]

[Total: 20]

Page 133: PHYSICS 9702/11 READ THESE INSTRUCTIONS FIRSTmaxpapers.com/wp-content/uploads/2012/11/9702_s17… ·  · 2017-10-17PHYSICS 9702/11 Paper 1 Multiple Choice May/June 2017 1 hour 15

This document consists of 11 printed pages and 1 blank page.

DC (LK/SW) 127170/3© UCLES 2017 [Turn over

Cambridge International ExaminationsCambridge International Advanced Subsidiary and Advanced Level

*4791138932*

PHYSICS 9702/33Paper 3 Advanced Practical Skills 1 May/June 2017 2 hoursCandidates answer on the Question Paper.Additional Materials: As listed in the Confidential Instructions.

READ THESE INSTRUCTIONS FIRST

Write your Centre number, candidate number and name on all the work you hand in.Write in dark blue or black pen.You may use an HB pencil for any diagrams or graphs.Do not use staples, paper clips, glue or correction fluid.DO NOT WRITE IN ANY BARCODES.

Answer both questions.You will be allowed to work with the apparatus for a maximum of one hour for each question.You are expected to record all your observations as soon as these observations are made, and to plan the presentation of the records so that it is not necessary to make a fair copy of them.You are reminded of the need for good English and clear presentation in your answers.

Electronic calculators may be used.You may lose marks if you do not show your working or if you do not use appropriate units.

Additional answer paper and graph paper should be used only if it becomes necessary to do so.

At the end of the examination, fasten all your work securely together.The number of marks is given in brackets [ ] at the end of each question or part question.

For Examiner’s Use

1

2

Total

Page 134: PHYSICS 9702/11 READ THESE INSTRUCTIONS FIRSTmaxpapers.com/wp-content/uploads/2012/11/9702_s17… ·  · 2017-10-17PHYSICS 9702/11 Paper 1 Multiple Choice May/June 2017 1 hour 15

2

9702/33/M/J/17© UCLES 2017

You may not need to use all of the materials provided.

1 In this experiment, you will investigate the motion of a chain of paper clips.

(a) You have been provided with a chain of fifteen paper clips with a sphere of modelling clay attached to one end of the chain.

Measure and record the length L of one paper clip as shown in Fig. 1.1.

L

Fig. 1.1

L = .................................................. [1]

Page 135: PHYSICS 9702/11 READ THESE INSTRUCTIONS FIRSTmaxpapers.com/wp-content/uploads/2012/11/9702_s17… ·  · 2017-10-17PHYSICS 9702/11 Paper 1 Multiple Choice May/June 2017 1 hour 15

3

9702/33/M/J/17© UCLES 2017 [Turn over

(b) (i) Set up the apparatus as shown in Fig. 1.2.

chain

wooden rod

n paper clips

sphere

boss

stand

bench

boss

nail

Fig. 1.2

The chain should be suspended from the nail.

The wooden rod should be positioned so that the chain, when hanging vertically, just touches the rod with 6 paper clips below the rod.

(ii) Record the number n of paper clips below the rod.

n = ......................................................

(c) Move the sphere towards you through a distance of approximately 10 cm. Release the sphere. The chain will oscillate and hit the rod during these oscillations.

Determine the period T of the oscillations.

T = .................................................. [2]

Page 136: PHYSICS 9702/11 READ THESE INSTRUCTIONS FIRSTmaxpapers.com/wp-content/uploads/2012/11/9702_s17… ·  · 2017-10-17PHYSICS 9702/11 Paper 1 Multiple Choice May/June 2017 1 hour 15

4

9702/33/M/J/17© UCLES 2017

(d) Change n by moving the wooden rod vertically and repeat (b)(ii) and (c) until you have six sets of values of n and T.

Record your results in a table. Include values of n to three significant figures in your table.

[8]

(e) (i) Plot a graph of T on the y-axis against n on the x-axis. [3]

(ii) Draw the straight line of best fit. [1]

(iii) Determine the gradient and y-intercept of this line.

gradient = ......................................................

y-intercept = ...................................................... [2]

Page 137: PHYSICS 9702/11 READ THESE INSTRUCTIONS FIRSTmaxpapers.com/wp-content/uploads/2012/11/9702_s17… ·  · 2017-10-17PHYSICS 9702/11 Paper 1 Multiple Choice May/June 2017 1 hour 15

5

9702/33/M/J/17© UCLES 2017 [Turn over

Page 138: PHYSICS 9702/11 READ THESE INSTRUCTIONS FIRSTmaxpapers.com/wp-content/uploads/2012/11/9702_s17… ·  · 2017-10-17PHYSICS 9702/11 Paper 1 Multiple Choice May/June 2017 1 hour 15

6

9702/33/M/J/17© UCLES 2017

(f) It is suggested that the quantities T and n are related by the equation

T P n Q= +

where P and Q are constants.

Using your answers in (e)(iii), determine the values of P and Q. Give appropriate units.

P = ......................................................

Q = ...................................................... [2]

(g) Theory suggests that

P = π gL

where g is the acceleration of free fall.

Use your values in (a) and (f) to determine a value for g. Give an appropriate unit.

g = .................................................. [1]

[Total: 20]

Page 139: PHYSICS 9702/11 READ THESE INSTRUCTIONS FIRSTmaxpapers.com/wp-content/uploads/2012/11/9702_s17… ·  · 2017-10-17PHYSICS 9702/11 Paper 1 Multiple Choice May/June 2017 1 hour 15

7

9702/33/M/J/17© UCLES 2017 [Turn over

You may not need to use all of the materials provided.

2 In this experiment, you will investigate the equilibrium of a balanced metre rule and determine its mass.

(a) You have been provided with a metre rule with a hole close to each end as shown in Fig. 2.1.

A

Fig. 2.1

The distance between the centres of the holes is A.

Measure and record A.

A = .................................................. [1]

(b) (i) Set up the apparatus as shown in Fig. 2.2.

x

5 cm

100 g mass hanger stand

50 g mass hanger

bench

metre rule

boss

string loop

rod of clamp

Fig. 2.2

Adjust the position of the rule until it is balanced.

The distance between the centre of the hole from which the 100 g mass hanger is supported and the position of the central string loop is x.

(ii) Measure and record x.

x = .................................................. [1]

Page 140: PHYSICS 9702/11 READ THESE INSTRUCTIONS FIRSTmaxpapers.com/wp-content/uploads/2012/11/9702_s17… ·  · 2017-10-17PHYSICS 9702/11 Paper 1 Multiple Choice May/June 2017 1 hour 15

8

9702/33/M/J/17© UCLES 2017

(c) (i) Add a 10 g slotted mass to each mass hanger.

(ii) Adjust the position of the rule until it is balanced. The distance between the centre of the hole from which the 100 g mass hanger is

supported and the position of the central string loop is now y as shown in Fig. 2.3.

y

Fig. 2.3

(iii) Measure and record y.

y = .................................................. [1]

(iv) Calculate (y – x).

(y – x) = .................................................. [1]

(v) Estimate the percentage uncertainty in your value of (y – x).

percentage uncertainty = .................................................. [1]

Page 141: PHYSICS 9702/11 READ THESE INSTRUCTIONS FIRSTmaxpapers.com/wp-content/uploads/2012/11/9702_s17… ·  · 2017-10-17PHYSICS 9702/11 Paper 1 Multiple Choice May/June 2017 1 hour 15

9

9702/33/M/J/17© UCLES 2017 [Turn over

(d) (i) Calculate m (A – 2y) where m = 10.0 g.

m (A – 2y) = .................................................. [1]

(ii) Justify the number of significant figures that you have given for your value of m (A – 2y).

..................................................................................................................................

..................................................................................................................................

.............................................................................................................................. [1]

(e) (i) Add another 10 g mass to each of the mass hangers and repeat (c)(ii), (c)(iii) and (c)(iv).

y = ......................................................

(y – x) = ...................................................... [2]

(ii) Calculate m (A – 2y) where m = 20.0 g.

m (A – 2y) = ......................................................

Page 142: PHYSICS 9702/11 READ THESE INSTRUCTIONS FIRSTmaxpapers.com/wp-content/uploads/2012/11/9702_s17… ·  · 2017-10-17PHYSICS 9702/11 Paper 1 Multiple Choice May/June 2017 1 hour 15

10

9702/33/M/J/17© UCLES 2017

(f) It is suggested that the relationship between y, x, m and A is

(y – x) = km (A – 2y)

where k is a constant.

(i) Using your data, calculate two values of k.

first value of k = ......................................................

second value of k = ...................................................... [1]

(ii) Explain whether your results in (f)(i) support the suggested relationship.

..................................................................................................................................

..................................................................................................................................

..................................................................................................................................

.............................................................................................................................. [1]

(g) Using your second value of k, calculate the mass M of the metre rule using the relationship

k B M1= +

where B = 150 g.

M = .............................................. g [1]

Page 143: PHYSICS 9702/11 READ THESE INSTRUCTIONS FIRSTmaxpapers.com/wp-content/uploads/2012/11/9702_s17… ·  · 2017-10-17PHYSICS 9702/11 Paper 1 Multiple Choice May/June 2017 1 hour 15

11

9702/33/M/J/17© UCLES 2017

(h) (i) Describe four sources of uncertainty or limitations of the procedure for this experiment.

1. ..............................................................................................................................

..................................................................................................................................

2. ..............................................................................................................................

..................................................................................................................................

3. ..............................................................................................................................

..................................................................................................................................

4. ..............................................................................................................................

.................................................................................................................................. [4]

(ii) Describe four improvements that could be made to this experiment. You may suggest the use of other apparatus or different procedures.

1. ..............................................................................................................................

..................................................................................................................................

2. ..............................................................................................................................

..................................................................................................................................

3. ..............................................................................................................................

..................................................................................................................................

4. ..............................................................................................................................

.................................................................................................................................. [4]

[Total: 20]

Page 144: PHYSICS 9702/11 READ THESE INSTRUCTIONS FIRSTmaxpapers.com/wp-content/uploads/2012/11/9702_s17… ·  · 2017-10-17PHYSICS 9702/11 Paper 1 Multiple Choice May/June 2017 1 hour 15

12

9702/33/M/J/17© UCLES 2017

Permission to reproduce items where third-party owned material protected by copyright is included has been sought and cleared where possible. Every reasonable effort has been made by the publisher (UCLES) to trace copyright holders, but if any items requiring clearance have unwittingly been included, the publisher will be pleased to make amends at the earliest possible opportunity.

To avoid the issue of disclosure of answer-related information to candidates, all copyright acknowledgements are reproduced online in the Cambridge International Examinations Copyright Acknowledgements Booklet. This is produced for each series of examinations and is freely available to download at www.cie.org.uk after the live examination series.

Cambridge International Examinations is part of the Cambridge Assessment Group. Cambridge Assessment is the brand name of University of Cambridge Local Examinations Syndicate (UCLES), which is itself a department of the University of Cambridge.

BLANK PAGE

Page 145: PHYSICS 9702/11 READ THESE INSTRUCTIONS FIRSTmaxpapers.com/wp-content/uploads/2012/11/9702_s17… ·  · 2017-10-17PHYSICS 9702/11 Paper 1 Multiple Choice May/June 2017 1 hour 15

This document consists of 12 printed pages.

DC (ST/FC) 127312/3© UCLES 2017 [Turn over

Cambridge International ExaminationsCambridge International Advanced Subsidiary and Advanced Level

*7337664815*

PHYSICS 9702/34Paper 3 Advanced Practical Skills 2 May/June 2017 2 hoursCandidates answer on the Question Paper.Additional Materials: As listed in the Confidential Instructions.

READ THESE INSTRUCTIONS FIRST

Write your Centre number, candidate number and name on all the work you hand in.Write in dark blue or black pen.You may use an HB pencil for any diagrams or graphs.Do not use staples, paper clips, glue or correction fluid.DO NOT WRITE IN ANY BARCODES.

Answer both questions.You will be allowed to work with the apparatus for a maximum of one hour for each question.You are expected to record all your observations as soon as these observations are made, and to plan the presentation of the records so that it is not necessary to make a fair copy of them.You are reminded of the need for good English and clear presentation in your answers.

Electronic calculators may be used.You may lose marks if you do not show your working or if you do not use appropriate units.

Additional answer paper and graph paper should be used only if it becomes necessary to do so.

At the end of the examination, fasten all your work securely together.The number of marks is given in brackets [ ] at the end of each question or part question.

For Examiner’s Use

1

2

Total

Page 146: PHYSICS 9702/11 READ THESE INSTRUCTIONS FIRSTmaxpapers.com/wp-content/uploads/2012/11/9702_s17… ·  · 2017-10-17PHYSICS 9702/11 Paper 1 Multiple Choice May/June 2017 1 hour 15

2

9702/34/M/J/17© UCLES 2017

You may not need to use all of the materials provided.

1 In this experiment, you will investigate the oscillation of a mass-spring system.

(a) Assemble the apparatus as shown in Fig. 1.1, with the rods of both clamps at an equal height of approximately 50 cm above the bench.

string

spring

θ

50 cm

h

standmasshanger

loop

rod ofclamp

bench

boss

Fig. 1.1

(i) The angle between the two lengths of string is θ, as shown in Fig. 1.1. Adjust the positions of the stands so that θ is approximately 165°.

Use the two G-clamps to secure the stands to the bench. The stands should remain in these positions for the rest of the experiment.

Page 147: PHYSICS 9702/11 READ THESE INSTRUCTIONS FIRSTmaxpapers.com/wp-content/uploads/2012/11/9702_s17… ·  · 2017-10-17PHYSICS 9702/11 Paper 1 Multiple Choice May/June 2017 1 hour 15

3

9702/34/M/J/17© UCLES 2017 [Turn over

(ii) Add 200 g to the mass hanger.

(iii) Record the total mass M of the mass hanger and added mass.

M = ......................................................

(iv) Measure and record the height h of the bottom of the mass hanger above the bench.

h = ................................................ cm (v) Pull down the mass hanger through a distance of approximately 2 cm. Release the

mass hanger so that it oscillates vertically.

Determine the period T of the vertical oscillations.

T = ...............................................s [2]

Page 148: PHYSICS 9702/11 READ THESE INSTRUCTIONS FIRSTmaxpapers.com/wp-content/uploads/2012/11/9702_s17… ·  · 2017-10-17PHYSICS 9702/11 Paper 1 Multiple Choice May/June 2017 1 hour 15

4

9702/34/M/J/17© UCLES 2017

(b) Change M and repeat (a)(iii), (a)(iv) and (a)(v) until you have six sets of values of M, h and T.

Record your results in a table. Include values of T 3 in your table.

[10]

(c) (i) Plot a graph of T 3 on the y-axis against h on the x-axis. [3]

(ii) Draw the straight line of best fit. [1]

(iii) Determine the gradient and y-intercept of this line.

gradient = ......................................................

y-intercept = ......................................................[2]

Page 149: PHYSICS 9702/11 READ THESE INSTRUCTIONS FIRSTmaxpapers.com/wp-content/uploads/2012/11/9702_s17… ·  · 2017-10-17PHYSICS 9702/11 Paper 1 Multiple Choice May/June 2017 1 hour 15

5

9702/34/M/J/17© UCLES 2017 [Turn over

Page 150: PHYSICS 9702/11 READ THESE INSTRUCTIONS FIRSTmaxpapers.com/wp-content/uploads/2012/11/9702_s17… ·  · 2017-10-17PHYSICS 9702/11 Paper 1 Multiple Choice May/June 2017 1 hour 15

6

9702/34/M/J/17© UCLES 2017

(d) It is suggested that the quantities T and h are related by the equation

T 3 = a h + b

where a and b are constants.

Use your answers in (c)(iii) to determine the values of a and b. Give appropriate units.

a = ......................................................

b = ......................................................[2]

[Total: 20]

Page 151: PHYSICS 9702/11 READ THESE INSTRUCTIONS FIRSTmaxpapers.com/wp-content/uploads/2012/11/9702_s17… ·  · 2017-10-17PHYSICS 9702/11 Paper 1 Multiple Choice May/June 2017 1 hour 15

7

9702/34/M/J/17© UCLES 2017 [Turn over

You may not need to use all of the materials provided.

2 In this experiment, you will investigate the equilibrium position of a pivoted wooden strip and determine the density of water.

(a) Assemble the apparatus as shown in Fig. 2.1. The nail should pass through the hole in the wooden strip and be held in the boss. The bottom of the plastic pipe should be approximately 2 cm above the bottom of the container.

Position the string loop with paper clips so that the wooden strip is parallel to the bench.

string loopstand

plumb-line

2 cmcontainer

water

plastic pipe

stringloop

paper clips

bench

bossx0

nail

wooden strip

Fig. 2.1 (not to scale)

The distance from the nail to the string loop holding the paper clips is x0, as shown in Fig. 2.1.

Measure and record x0.

x0 = ........................................... cm [1]

Page 152: PHYSICS 9702/11 READ THESE INSTRUCTIONS FIRSTmaxpapers.com/wp-content/uploads/2012/11/9702_s17… ·  · 2017-10-17PHYSICS 9702/11 Paper 1 Multiple Choice May/June 2017 1 hour 15

8

9702/34/M/J/17© UCLES 2017

(b) (i) Move the string loop holding the paper clips approximately 4 cm further from the nail. Let the wooden strip settle at an angle, as shown in Fig. 2.2.

x

φ

Fig. 2.2 (not to scale)

(ii) Measure and record the new distance x from the nail to the string loop holding the paper clips, as shown in Fig. 2.2.

x = ........................................... cm [1]

(iii) Measure and record the larger angle φ between the wooden strip and the plumb-line, as shown in Fig. 2.2.

φ = .............................................. ° [1]

(iv) Calculate (φ – 90°).

(φ – 90°) = ................................................... °

(v) Estimate the percentage uncertainty in your value of (φ – 90°).

percentage uncertainty = ..................................................[1]

Page 153: PHYSICS 9702/11 READ THESE INSTRUCTIONS FIRSTmaxpapers.com/wp-content/uploads/2012/11/9702_s17… ·  · 2017-10-17PHYSICS 9702/11 Paper 1 Multiple Choice May/June 2017 1 hour 15

9

9702/34/M/J/17© UCLES 2017 [Turn over

(c) (i) Move the string loop holding the paper clips approximately 3 cm further from the nail. Let the wooden strip settle at a new angle.

(ii) Repeat (b)(ii), (b)(iii) and (b)(iv).

x = ................................................ cm

φ = ................................................... °

(φ – 90°) = ................................................... °[3]

Question 2 continues on page 10.

Page 154: PHYSICS 9702/11 READ THESE INSTRUCTIONS FIRSTmaxpapers.com/wp-content/uploads/2012/11/9702_s17… ·  · 2017-10-17PHYSICS 9702/11 Paper 1 Multiple Choice May/June 2017 1 hour 15

10

9702/34/M/J/17© UCLES 2017

(d) It is suggested that the relationship between φ, x0 and x is

sin(φ – 90°) = k(x – x0)

where k is a constant.

(i) Using your data, calculate two values of k.

first value of k = ......................................................

second value of k = ......................................................[1]

(ii) Justify the number of significant figures you have given for your values of k.

.................................................................................................................................

.................................................................................................................................

............................................................................................................................. [1]

(iii) Explain whether your results in (d)(i) support the suggested relationship.

.................................................................................................................................

.................................................................................................................................

.................................................................................................................................

............................................................................................................................. [1]

Page 155: PHYSICS 9702/11 READ THESE INSTRUCTIONS FIRSTmaxpapers.com/wp-content/uploads/2012/11/9702_s17… ·  · 2017-10-17PHYSICS 9702/11 Paper 1 Multiple Choice May/June 2017 1 hour 15

11

9702/34/M/J/17© UCLES 2017 [Turn over

(e) (i) Remove the plastic pipe from the water.

(ii) Measure and record the outside diameter D and the inside diameter d of the pipe, as shown in Fig. 2.3.

D

d

Fig. 2.3

D = ................................................ cm

d = .................................................cm[1]

(iii) The mass m of the paper clips and string loop is written on the card. Record m.

m = ................................................... g

(iv) Using your second value of k, calculate the density ρ of the water using the relationship

ρ = mCk(D 2 – d 2)

where C = 201 cm2.

ρ = ..................................... g cm–3 [1]

Page 156: PHYSICS 9702/11 READ THESE INSTRUCTIONS FIRSTmaxpapers.com/wp-content/uploads/2012/11/9702_s17… ·  · 2017-10-17PHYSICS 9702/11 Paper 1 Multiple Choice May/June 2017 1 hour 15

12

9702/34/M/J/17© UCLES 2017

Permission to reproduce items where third-party owned material protected by copyright is included has been sought and cleared where possible. Every reasonable effort has been made by the publisher (UCLES) to trace copyright holders, but if any items requiring clearance have unwittingly been included, the publisher will be pleased to make amends at the earliest possible opportunity.

To avoid the issue of disclosure of answer-related information to candidates, all copyright acknowledgements are reproduced online in the Cambridge International Examinations Copyright Acknowledgements Booklet. This is produced for each series of examinations and is freely available to download at www.cie.org.uk after the live examination series.

Cambridge International Examinations is part of the Cambridge Assessment Group. Cambridge Assessment is the brand name of University of Cambridge Local Examinations Syndicate (UCLES), which is itself a department of the University of Cambridge.

(f) (i) Describe four sources of uncertainty or limitations of the procedure for this experiment.

1. .............................................................................................................................

.................................................................................................................................

2. .............................................................................................................................

.................................................................................................................................

3. .............................................................................................................................

.................................................................................................................................

4. .............................................................................................................................

.................................................................................................................................[4]

(ii) Describe four improvements that could be made to this experiment. You may suggest the use of other apparatus or different procedures.

1. .............................................................................................................................

.................................................................................................................................

2. .............................................................................................................................

.................................................................................................................................

3. .............................................................................................................................

.................................................................................................................................

4. .............................................................................................................................

.................................................................................................................................[4]

[Total: 20]

Page 157: PHYSICS 9702/11 READ THESE INSTRUCTIONS FIRSTmaxpapers.com/wp-content/uploads/2012/11/9702_s17… ·  · 2017-10-17PHYSICS 9702/11 Paper 1 Multiple Choice May/June 2017 1 hour 15

This document consists of 11 printed pages and 1 blank page.

DC (ST/SW) 127174/3© UCLES 2017 [Turn over

Cambridge International ExaminationsCambridge International Advanced Subsidiary and Advanced Level

*8973624404*

PHYSICS 9702/35Paper 3 Advanced Practical Skills 1 May/June 2017 2 hoursCandidates answer on the Question Paper.Additional Materials: As listed in the Confidential Instructions.

READ THESE INSTRUCTIONS FIRST

Write your Centre number, candidate number and name on all the work you hand in.Write in dark blue or black pen.You may use an HB pencil for any diagrams or graphs.Do not use staples, paper clips, glue or correction fluid.DO NOT WRITE IN ANY BARCODES.

Answer both questions.You will be allowed to work with the apparatus for a maximum of one hour for each question.You are expected to record all your observations as soon as these observations are made, and to plan the presentation of the records so that it is not necessary to make a fair copy of them.You are reminded of the need for good English and clear presentation in your answers.

Electronic calculators may be used.You may lose marks if you do not show your working or if you do not use appropriate units.

Additional answer paper and graph paper should be used only if it becomes necessary to do so.

At the end of the examination, fasten all your work securely together.The number of marks is given in brackets [ ] at the end of each question or part question.

For Examiner’s Use

1

2

Total

Page 158: PHYSICS 9702/11 READ THESE INSTRUCTIONS FIRSTmaxpapers.com/wp-content/uploads/2012/11/9702_s17… ·  · 2017-10-17PHYSICS 9702/11 Paper 1 Multiple Choice May/June 2017 1 hour 15

2

9702/35/M/J/17© UCLES 2017

You may not need to use all of the materials provided.

1 In this experiment, you will investigate the motion of a supported copper wire. (a) You have been provided with a copper wire and two spheres of modelling clay. (i) Bend the wire about its midpoint so that the two lengths are perpendicular to each

other, as shown in Fig. 1.1.

25 cm

wire

25 cm90°

Fig. 1.1

(ii) Push each end of the wire through the centre of a sphere of modelling clay. Place the spheres approximately half-way along each length of the wire as shown

in Fig. 1.2.

x x

sphere of modelling clay

Fig. 1.2

The centres of the spheres should each be the same distance from the midpoint of the wire.

Gently press the modelling clay onto the wire to ensure the spheres stay in position. (iii) Measure and record the distance x from the midpoint of the wire to the centre of a

sphere.

x = .................................................. [1]

Page 159: PHYSICS 9702/11 READ THESE INSTRUCTIONS FIRSTmaxpapers.com/wp-content/uploads/2012/11/9702_s17… ·  · 2017-10-17PHYSICS 9702/11 Paper 1 Multiple Choice May/June 2017 1 hour 15

3

9702/35/M/J/17© UCLES 2017 [Turn over

(b) (i) Set up the apparatus as shown in Fig. 1.3.

bench

bosswooden rod

wire

stand

Fig. 1.3 (ii) Move one end of the wire down through a short distance. Release the wire.

The wire will oscillate.

Determine the period T of these oscillations.

T = ..................................................[1]

Page 160: PHYSICS 9702/11 READ THESE INSTRUCTIONS FIRSTmaxpapers.com/wp-content/uploads/2012/11/9702_s17… ·  · 2017-10-17PHYSICS 9702/11 Paper 1 Multiple Choice May/June 2017 1 hour 15

4

9702/35/M/J/17© UCLES 2017

(c) Vary x and repeat (a)(iii) and (b) until you have six sets of values of x and T.

Record your results in a table. Include values of x 2 in your table.

[9]

(d) (i) Plot a graph of T on the y-axis against x 2 on the x-axis. [3] (ii) Draw the straight line of best fit. [1] (iii) Determine the gradient and y-intercept of this line.

gradient = ......................................................

y-intercept = ......................................................[2]

Page 161: PHYSICS 9702/11 READ THESE INSTRUCTIONS FIRSTmaxpapers.com/wp-content/uploads/2012/11/9702_s17… ·  · 2017-10-17PHYSICS 9702/11 Paper 1 Multiple Choice May/June 2017 1 hour 15

5

9702/35/M/J/17© UCLES 2017 [Turn over

Page 162: PHYSICS 9702/11 READ THESE INSTRUCTIONS FIRSTmaxpapers.com/wp-content/uploads/2012/11/9702_s17… ·  · 2017-10-17PHYSICS 9702/11 Paper 1 Multiple Choice May/June 2017 1 hour 15

6

9702/35/M/J/17© UCLES 2017

(e) It is suggested that the quantities T and x are related by the equation

T = Px 2 + Q

where P and Q are constants. Using your answers in (d)(iii), determine the values of P and Q. Give appropriate units.

P = ......................................................

Q = ......................................................[2]

(f) (i) Remove the spheres from the wire.

(ii) Repeat (b)(ii).

T = ...................................................... (iii) Calculate the value of x that corresponds to the value of T in (f)(ii).

x = ..................................................[1]

[Total: 20]

Page 163: PHYSICS 9702/11 READ THESE INSTRUCTIONS FIRSTmaxpapers.com/wp-content/uploads/2012/11/9702_s17… ·  · 2017-10-17PHYSICS 9702/11 Paper 1 Multiple Choice May/June 2017 1 hour 15

7

9702/35/M/J/17© UCLES 2017 [Turn over

You may not need to use all of the materials provided.

2 In this experiment, you will investigate the equilibrium of a system of three identical springs.

(a) Measure the length L of the unstretched coiled section of one of the springs as shown in Fig. 2.1.

L

spring

Fig. 2.1

L = ..................................................[1]

(b) Set up the apparatus as shown in Fig. 2.2.

bench

d 5 cm

G-clamp

tapespring B

spring C

stand

spring A

Fig. 2.2 (not to scale)

The springs are identical. Hold spring A as shown in Fig. 2.2.

The distance d  between the loops of springs B and C on the stand should be approximately 5 cm as shown in Fig. 2.2. Do not mark the tape.

Page 164: PHYSICS 9702/11 READ THESE INSTRUCTIONS FIRSTmaxpapers.com/wp-content/uploads/2012/11/9702_s17… ·  · 2017-10-17PHYSICS 9702/11 Paper 1 Multiple Choice May/June 2017 1 hour 15

8

9702/35/M/J/17© UCLES 2017

(c) (i) Pull spring A until the length of its coiled section is 15 cm as shown in Fig. 2.3.

15 cm

y

θ

Fig. 2.3 (not to scale)

The length of the coiled section of spring B is y. The angle between springs B and C is θ.

(ii) Measure and record y.

y = ..................................................[1]

(iii) Measure and record θ.

θ = ............................................... ° [1]

(d) Estimate the percentage uncertainty in your value of θ.

percentage uncertainty = ................................................. [1]

Page 165: PHYSICS 9702/11 READ THESE INSTRUCTIONS FIRSTmaxpapers.com/wp-content/uploads/2012/11/9702_s17… ·  · 2017-10-17PHYSICS 9702/11 Paper 1 Multiple Choice May/June 2017 1 hour 15

9

9702/35/M/J/17© UCLES 2017 [Turn over

(e) (i) Calculate (y – L).

(y – L) = ..................................................[1]

(ii) Calculate cos (θ2).

cos (θ2) = ..................................................[1]

(iii) Justify the number of significant figures that you have given for your value of cos (θ2).

.................................................................................................................................

.................................................................................................................................

............................................................................................................................. [1]

(f) Increase d to approximately 10 cm and repeat (c), (e)(i) and (e)(ii).

y = ......................................................

θ = .................................................... °

(y – L) = ......................................................

cos (θ2) = ......................................................[3]

Page 166: PHYSICS 9702/11 READ THESE INSTRUCTIONS FIRSTmaxpapers.com/wp-content/uploads/2012/11/9702_s17… ·  · 2017-10-17PHYSICS 9702/11 Paper 1 Multiple Choice May/June 2017 1 hour 15

10

9702/35/M/J/17© UCLES 2017

(g) It is suggested that the relationship between θ, y and L is

cos (θ2) = k(y – L)

where k is a constant.

(i) Using your data, calculate two values of k.

first value of k = ......................................................

second value of k = ......................................................[1]

(ii) Explain whether your results support the suggested relationship.

.................................................................................................................................

.................................................................................................................................

.................................................................................................................................

............................................................................................................................. [1]

Page 167: PHYSICS 9702/11 READ THESE INSTRUCTIONS FIRSTmaxpapers.com/wp-content/uploads/2012/11/9702_s17… ·  · 2017-10-17PHYSICS 9702/11 Paper 1 Multiple Choice May/June 2017 1 hour 15

11

9702/35/M/J/17© UCLES 2017

(h) (i) Describe four sources of uncertainty or limitations of the procedure for this experiment.

1. .............................................................................................................................

.................................................................................................................................

2. .............................................................................................................................

.................................................................................................................................

3. .............................................................................................................................

.................................................................................................................................

4. .............................................................................................................................

.................................................................................................................................[4]

(ii) Describe four improvements that could be made to this experiment. You may

suggest the use of other apparatus or different procedures.

1. .............................................................................................................................

.................................................................................................................................

2. .............................................................................................................................

.................................................................................................................................

3. .............................................................................................................................

.................................................................................................................................

4. .............................................................................................................................

.................................................................................................................................[4]

[Total: 20]

Page 168: PHYSICS 9702/11 READ THESE INSTRUCTIONS FIRSTmaxpapers.com/wp-content/uploads/2012/11/9702_s17… ·  · 2017-10-17PHYSICS 9702/11 Paper 1 Multiple Choice May/June 2017 1 hour 15

12

9702/35/M/J/17© UCLES 2017

Permission to reproduce items where third-party owned material protected by copyright is included has been sought and cleared where possible. Every reasonable effort has been made by the publisher (UCLES) to trace copyright holders, but if any items requiring clearance have unwittingly been included, the publisher will be pleased to make amends at the earliest possible opportunity.

To avoid the issue of disclosure of answer-related information to candidates, all copyright acknowledgements are reproduced online in the Cambridge International Examinations Copyright Acknowledgements Booklet. This is produced for each series of examinations and is freely available to download at www.cie.org.uk after the live examination series.

Cambridge International Examinations is part of the Cambridge Assessment Group. Cambridge Assessment is the brand name of University of Cambridge Local Examinations Syndicate (UCLES), which is itself a department of the University of Cambridge.

BLANK PAGE

Page 169: PHYSICS 9702/11 READ THESE INSTRUCTIONS FIRSTmaxpapers.com/wp-content/uploads/2012/11/9702_s17… ·  · 2017-10-17PHYSICS 9702/11 Paper 1 Multiple Choice May/June 2017 1 hour 15

This document consists of 22 printed pages and 2 blank pages.

DC (LK/FD) 127087/2© UCLES 2017 [Turn over

Cambridge International ExaminationsCambridge International Advanced Subsidiary and Advanced Level

*7280320988*

PHYSICS 9702/41Paper 4 A Level Structured Questions May/June 2017 2 hoursCandidates answer on the Question Paper.No Additional Materials are required.

READ THESE INSTRUCTIONS FIRST

Write your Centre number, candidate number and name on all the work you hand in.Write in dark blue or black pen.You may use an HB pencil for any diagrams or graphs.Do not use staples, paper clips, glue or correction fluid.DO NOT WRITE IN ANY BARCODES.

Answer all questions.

Electronic calculators may be used.You may lose marks if you do not show your working or if you do not use appropriate units.

At the end of the examination, fasten all your work securely together.The number of marks is given in brackets [ ] at the end of each question or part question.

Page 170: PHYSICS 9702/11 READ THESE INSTRUCTIONS FIRSTmaxpapers.com/wp-content/uploads/2012/11/9702_s17… ·  · 2017-10-17PHYSICS 9702/11 Paper 1 Multiple Choice May/June 2017 1 hour 15

2

9702/41/M/J/17© UCLES 2017

Data

speed of light in free space c = 3.00 × 108 m s−1

permeability of free space μ0 = 4π × 10−7 H m−1

permittivity of free space ε0 = 8.85 × 10−12 F m−1

( 14πε0

= 8.99 × 109 m F−1) elementary charge e = 1.60 × 10−19 C

the Planck constant h = 6.63 × 10−34 J s

unified atomic mass unit 1 u = 1.66 × 10−27 kg

rest mass of electron me = 9.11 × 10−31 kg

rest mass of proton mp = 1.67 × 10−27 kg

molar gas constant R = 8.31 J K−1 mol−1

the Avogadro constant NA = 6.02 × 1023 mol−1

the Boltzmann constant k = 1.38 × 10−23 J K−1

gravitational constant G = 6.67 × 10−11 N m2 kg−2

acceleration of free fall g = 9.81 m s−2

Page 171: PHYSICS 9702/11 READ THESE INSTRUCTIONS FIRSTmaxpapers.com/wp-content/uploads/2012/11/9702_s17… ·  · 2017-10-17PHYSICS 9702/11 Paper 1 Multiple Choice May/June 2017 1 hour 15

3

9702/41/M/J/17© UCLES 2017 [Turn over

Formulae

uniformly accelerated motion s = ut + 12 at 2

v2 = u2 + 2as

work done on/by a gas W = pΔV

gravitational potential φ =− Gmr

hydrostatic pressure p = ρgh

pressure of an ideal gas p = 13 NmV 〈c2〉

simple harmonic motion a =−ω 2x

velocity of particle in s.h.m. v = v0 cos ωt v = ± ω ( )x x0

2 2-

Doppler effect fo = fsv

v ± vs

electric potential V = Q4πε0r

capacitors in series 1/C = 1/C1 + 1/C2 + . . .

capacitors in parallel C = C1 + C2 + . . .

energy of charged capacitor W = 12 QV

electric current I = Anvq

resistors in series R = R1 + R2 + . . .

resistors in parallel 1/R = 1/R1 + 1/R2 + . . .

Hall voltage VH = BIntq

alternating current/voltage x = x0 sin ω  t

radioactive decay x = x0exp(−λt )

decay constant λ = 0.693t 1

2

Page 172: PHYSICS 9702/11 READ THESE INSTRUCTIONS FIRSTmaxpapers.com/wp-content/uploads/2012/11/9702_s17… ·  · 2017-10-17PHYSICS 9702/11 Paper 1 Multiple Choice May/June 2017 1 hour 15

4

9702/41/M/J/17© UCLES 2017

Answer all the questions in the spaces provided.

1 (a) Explain how a satellite may be in a circular orbit around a planet.

...................................................................................................................................................

...................................................................................................................................................

...............................................................................................................................................[2]

(b) The Earth and the Moon may be considered to be uniform spheres that are isolated in space. The Earth has radius R and mean density ρ. The Moon, mass m, is in a circular orbit about the Earth with radius nR, as illustrated in Fig. 1.1.

Moon

Earthradius R

nR

Fig. 1.1

The Moon makes one complete orbit of the Earth in time T. Show that the mean density ρ of the Earth is given by the expression

ρ = 3πn3

GT 2 .

[4]

Page 173: PHYSICS 9702/11 READ THESE INSTRUCTIONS FIRSTmaxpapers.com/wp-content/uploads/2012/11/9702_s17… ·  · 2017-10-17PHYSICS 9702/11 Paper 1 Multiple Choice May/June 2017 1 hour 15

5

9702/41/M/J/17© UCLES 2017 [Turn over

(c) The radius R of the Earth is 6.38 × 103 km and the distance between the centre of the Earth and the centre of the Moon is 3.84 × 105 km.

The period T of the orbit of the Moon about the Earth is 27.3 days. Use the expression in (b) to calculate ρ.

ρ = ............................................... kg m–3 [3]

[Total: 9]

Page 174: PHYSICS 9702/11 READ THESE INSTRUCTIONS FIRSTmaxpapers.com/wp-content/uploads/2012/11/9702_s17… ·  · 2017-10-17PHYSICS 9702/11 Paper 1 Multiple Choice May/June 2017 1 hour 15

6

9702/41/M/J/17© UCLES 2017

2 A bar magnet of mass 180 g is suspended from the free end of a spring, as illustrated in Fig. 2.1.

spring

magnet

coil

Fig. 2.1

The magnet hangs so that one pole is near the centre of a coil of wire.

The coil is connected in series with a resistor and a switch. The switch is open.

The magnet is displaced vertically and then allowed to oscillate with one pole remaining inside the coil. The other pole remains outside the coil.

At time t = 0, the magnet is oscillating freely as it passes through its equilibrium position. At time t = 3.0 s, the switch in the circuit is closed.

The variation with time t of the vertical displacement y of the magnet is shown in Fig. 2.2.

–2.0

–1.5

–1.0

–0.5

0

0.5

1.0y / cm

1.5

2.0

0 1 2 3 4 5 6 7 8 9t / s

Fig. 2.2

Page 175: PHYSICS 9702/11 READ THESE INSTRUCTIONS FIRSTmaxpapers.com/wp-content/uploads/2012/11/9702_s17… ·  · 2017-10-17PHYSICS 9702/11 Paper 1 Multiple Choice May/June 2017 1 hour 15

7

9702/41/M/J/17© UCLES 2017 [Turn over

(a) Determine, to two significant figures, the frequency of oscillation of the magnet.

frequency = .................................................... Hz [2]

(b) State whether the closing of the switch gives rise to light, heavy or critical damping.

...............................................................................................................................................[1]

(c) Calculate the change in the energy ΔE of oscillation of the magnet between time t = 2.7 s and time t = 7.5 s. Explain your working.

ΔE = ....................................................... J [6]

[Total: 9]

Page 176: PHYSICS 9702/11 READ THESE INSTRUCTIONS FIRSTmaxpapers.com/wp-content/uploads/2012/11/9702_s17… ·  · 2017-10-17PHYSICS 9702/11 Paper 1 Multiple Choice May/June 2017 1 hour 15

8

9702/41/M/J/17© UCLES 2017

3 The digital transmission of speech may be illustrated using the block diagram of Fig. 3.1.

ADCserial -to -parallel

converteroptic fibreX Y

Fig. 3.1

(a) (i) State what is meant by a digital signal.

...........................................................................................................................................

.......................................................................................................................................[1]

(ii) State the names of the components labelled X and Y on Fig. 3.1.

X: ......................................................................................................................................

Y: ...................................................................................................................................... [2]

(iii) Describe the function of the ADC.

...........................................................................................................................................

...........................................................................................................................................

.......................................................................................................................................[2]

(b) The optic fibre has length 84 km and the attenuation per unit length in the fibre is 0.19 dB km–1.

The input power to the optic fibre is 9.7 mW. At the output from the optic fibre, the signal-to-noise ratio is 28 dB.

Calculate

(i) in dB, the ratio

input power to optic fibrenoise power at output of optic fibre,

ratio = .................................................... dB [2]

Page 177: PHYSICS 9702/11 READ THESE INSTRUCTIONS FIRSTmaxpapers.com/wp-content/uploads/2012/11/9702_s17… ·  · 2017-10-17PHYSICS 9702/11 Paper 1 Multiple Choice May/June 2017 1 hour 15

9

9702/41/M/J/17© UCLES 2017 [Turn over

(ii) the noise power at the output of the optic fibre.

noise power = ..................................................... W [3]

[Total: 10]

Page 178: PHYSICS 9702/11 READ THESE INSTRUCTIONS FIRSTmaxpapers.com/wp-content/uploads/2012/11/9702_s17… ·  · 2017-10-17PHYSICS 9702/11 Paper 1 Multiple Choice May/June 2017 1 hour 15

10

9702/41/M/J/17© UCLES 2017

4 (a) Describe the motion of molecules in a gas, according to the kinetic theory of gases.

...................................................................................................................................................

...................................................................................................................................................

...............................................................................................................................................[2]

(b) Describe what is observed when viewing Brownian motion that provides evidence for your answer in (a).

...................................................................................................................................................

...................................................................................................................................................

...............................................................................................................................................[2]

(c) At a pressure of 1.05 × 105 Pa and a temperature of 27 °C, 1.00 mol of helium gas has a volume of 0.0240 m3.

The mass of 1.00 mol of helium gas, assumed to be an ideal gas, is 4.00 g.

(i) Calculate the root-mean-square (r.m.s.) speed of an atom of helium gas for a temperature of 27 °C.

r.m.s. speed = ................................................. m s–1 [3]

(ii) Using your answer in (i), calculate the r.m.s. speed of the atoms at 177 °C.

r.m.s. speed = ................................................. m s–1 [3]

[Total: 10]

Page 179: PHYSICS 9702/11 READ THESE INSTRUCTIONS FIRSTmaxpapers.com/wp-content/uploads/2012/11/9702_s17… ·  · 2017-10-17PHYSICS 9702/11 Paper 1 Multiple Choice May/June 2017 1 hour 15

11

9702/41/M/J/17© UCLES 2017 [Turn over

5 An α-particle is travelling in a vacuum towards the centre of a gold nucleus, as illustrated in Fig. 5.1.

gold nucleus α-particle

charge 79e energy 7.7 10–13 J×

Fig. 5.1

The gold nucleus has charge 79e. The gold nucleus and the α-particle may be assumed to behave as point charges. At a large distance from the gold nucleus, the α-particle has energy 7.7 × 10–13 J.

(a) The α-particle does not collide with the gold nucleus. Show that the radius of the gold nucleus must be less than 4.7 × 10–14 m.

[3]

(b) Determine the acceleration of the α-particle for a separation of 4.7 × 10–14 m between the centres of the gold nucleus and of the α-particle.

acceleration = ................................................. m s–2 [3]

(c) In an α-particle scattering experiment, the beam of α-particles is incident on a very thin gold foil.

Suggest why the gold foil must be very thin.

...................................................................................................................................................

...............................................................................................................................................[1]

[Total: 7]

Page 180: PHYSICS 9702/11 READ THESE INSTRUCTIONS FIRSTmaxpapers.com/wp-content/uploads/2012/11/9702_s17… ·  · 2017-10-17PHYSICS 9702/11 Paper 1 Multiple Choice May/June 2017 1 hour 15

12

9702/41/M/J/17© UCLES 2017

6 A comparator circuit is designed to switch on a mains lamp when the ambient light level reaches a set value.

An incomplete diagram of the circuit is shown in Fig. 6.1.

+

+5 V

–5 V

D

RV

6 V

Fig. 6.1

(a) (i) A relay is required as part of the output device. This is not shown in Fig. 6.1. Explain why a relay is required.

...........................................................................................................................................

...........................................................................................................................................

.......................................................................................................................................[2]

(ii) On Fig. 6.1, draw the symbol for a relay connected in the circuit as part of the output device. [2]

(b) Describe the function of

(i) the variable resistor RV,

...........................................................................................................................................

.......................................................................................................................................[1]

(ii) the diode D.

...........................................................................................................................................

.......................................................................................................................................[1]

Page 181: PHYSICS 9702/11 READ THESE INSTRUCTIONS FIRSTmaxpapers.com/wp-content/uploads/2012/11/9702_s17… ·  · 2017-10-17PHYSICS 9702/11 Paper 1 Multiple Choice May/June 2017 1 hour 15

13

9702/41/M/J/17© UCLES 2017 [Turn over

(c) State whether the lamp will switch on as the light level increases or as it decreases. Explain your answer.

...................................................................................................................................................

...................................................................................................................................................

...................................................................................................................................................

...................................................................................................................................................

...............................................................................................................................................[3]

[Total: 9]

Page 182: PHYSICS 9702/11 READ THESE INSTRUCTIONS FIRSTmaxpapers.com/wp-content/uploads/2012/11/9702_s17… ·  · 2017-10-17PHYSICS 9702/11 Paper 1 Multiple Choice May/June 2017 1 hour 15

14

9702/41/M/J/17© UCLES 2017

7 An electron having charge –q and mass m is accelerated from rest in a vacuum through a potential difference V.

The electron then enters a region of uniform magnetic field of magnetic flux density B, as shown in Fig. 7.1.

path ofelectron

uniform magneticfield into planeof paper

Fig. 7.1

The direction of the uniform magnetic field is into the plane of the paper. The velocity of the electron as it enters the magnetic field is normal to the magnetic field. The radius of the circular path of the electron in the magnetic field is r.

(a) Explain why the path of the electron in the magnetic field is the arc of a circle.

...................................................................................................................................................

...................................................................................................................................................

...................................................................................................................................................

...............................................................................................................................................[3]

(b) Show that the magnitude p of the momentum of the electron as it enters the magnetic field is given by

p = (2mqV ).

[2]

Page 183: PHYSICS 9702/11 READ THESE INSTRUCTIONS FIRSTmaxpapers.com/wp-content/uploads/2012/11/9702_s17… ·  · 2017-10-17PHYSICS 9702/11 Paper 1 Multiple Choice May/June 2017 1 hour 15

15

9702/41/M/J/17© UCLES 2017 [Turn over

(c) The potential difference V is 120 V. The radius r of the circular arc is 7.4 cm.

Determine the magnitude B of the magnetic flux density.

B = ....................................................... T [3]

(d) The potential difference V in (c) is increased. The magnetic flux density B remains unchanged.

By reference to the momentum of the electron, explain the effect of this increase on the radius r of the path of the electron in the magnetic field.

...................................................................................................................................................

...................................................................................................................................................

...............................................................................................................................................[2]

[Total: 10]

Page 184: PHYSICS 9702/11 READ THESE INSTRUCTIONS FIRSTmaxpapers.com/wp-content/uploads/2012/11/9702_s17… ·  · 2017-10-17PHYSICS 9702/11 Paper 1 Multiple Choice May/June 2017 1 hour 15

16

9702/41/M/J/17© UCLES 2017

8 Explain the main principles behind the use of nuclear magnetic resonance imaging (NMRI) to obtain information about internal body structures.

..........................................................................................................................................................

..........................................................................................................................................................

..........................................................................................................................................................

..........................................................................................................................................................

..........................................................................................................................................................

..........................................................................................................................................................

..........................................................................................................................................................

..........................................................................................................................................................

..........................................................................................................................................................

..........................................................................................................................................................

..........................................................................................................................................................

..........................................................................................................................................................

..........................................................................................................................................................

......................................................................................................................................................[8]

[Total: 8]

Page 185: PHYSICS 9702/11 READ THESE INSTRUCTIONS FIRSTmaxpapers.com/wp-content/uploads/2012/11/9702_s17… ·  · 2017-10-17PHYSICS 9702/11 Paper 1 Multiple Choice May/June 2017 1 hour 15

17

9702/41/M/J/17© UCLES 2017 [Turn over

9 A simple transformer is illustrated in Fig. 9.1.

output

laminatediron core

input

Fig. 9.1

(a) (i) State why the transformer has an iron core, rather than having no core.

...........................................................................................................................................

.......................................................................................................................................[1]

(ii) Explain why the core is laminated.

...........................................................................................................................................

...........................................................................................................................................

.......................................................................................................................................[2]

(b) By reference to the action of a transformer, explain why the input to the transformer is an alternating voltage, rather than a constant voltage.

...................................................................................................................................................

...................................................................................................................................................

...................................................................................................................................................

...................................................................................................................................................

...............................................................................................................................................[3]

[Total: 6]

Page 186: PHYSICS 9702/11 READ THESE INSTRUCTIONS FIRSTmaxpapers.com/wp-content/uploads/2012/11/9702_s17… ·  · 2017-10-17PHYSICS 9702/11 Paper 1 Multiple Choice May/June 2017 1 hour 15

18

9702/41/M/J/17© UCLES 2017

BLANK PAGE

Page 187: PHYSICS 9702/11 READ THESE INSTRUCTIONS FIRSTmaxpapers.com/wp-content/uploads/2012/11/9702_s17… ·  · 2017-10-17PHYSICS 9702/11 Paper 1 Multiple Choice May/June 2017 1 hour 15

19

9702/41/M/J/17© UCLES 2017 [Turn over

10 (a) State

(i) what is meant by the hardness of an X-ray beam,

...........................................................................................................................................

...........................................................................................................................................

.......................................................................................................................................[2]

(ii) how the hardness of an X-ray beam from an X-ray tube is increased.

...........................................................................................................................................

.......................................................................................................................................[1]

(b) The same parallel beam of X-ray radiation is incident, separately, on samples of bone and of muscle.

Data for the thickness x of the samples of bone and of muscle, together with the linear attenuation (absorption) coefficients μ of the radiation in bone and in muscle, are given in Fig. 10.1.

x / cm μ / cm–1

bone 1.5 2.9

muscle 4.0 0.95

Fig. 10.1

Determine the ratio

intensity transmitted through boneintensity transmitted through muscle

.

ratio = .......................................................... [2]

[Total: 5]

Page 188: PHYSICS 9702/11 READ THESE INSTRUCTIONS FIRSTmaxpapers.com/wp-content/uploads/2012/11/9702_s17… ·  · 2017-10-17PHYSICS 9702/11 Paper 1 Multiple Choice May/June 2017 1 hour 15

20

9702/41/M/J/17© UCLES 2017

11 A beam of light consists of a continuous range of wavelengths from 420 nm to 740 nm. The light passes through a cloud of cool gas, as shown in Fig. 11.1.

incident lightcool gas

emergent light

wavelengths 420 nm – 740 nm

Fig. 11.1

(a) The spectrum of the light emerging from the cloud of cool gas is viewed using a diffraction grating.

Explain why this spectrum contains a number of dark lines.

...................................................................................................................................................

...................................................................................................................................................

...................................................................................................................................................

...................................................................................................................................................

...................................................................................................................................................

...............................................................................................................................................[4]

(b) Some of the electron energy levels of the atoms in the cloud of gas are represented in Fig. 11.2.

energy

– 0.38 eV– 0.54 eV– 0.85 eV

– 1.5 eV

– 3.4 eV

– 13.6 eV

Fig. 11.2 (not to scale)

Page 189: PHYSICS 9702/11 READ THESE INSTRUCTIONS FIRSTmaxpapers.com/wp-content/uploads/2012/11/9702_s17… ·  · 2017-10-17PHYSICS 9702/11 Paper 1 Multiple Choice May/June 2017 1 hour 15

21

9702/41/M/J/17© UCLES 2017 [Turn over

(i) Light of wavelength 420 nm has a photon energy of 2.96 eV. Calculate the photon energy, in eV, of light of wavelength 740 nm.

photon energy = .................................................... eV [2]

(ii) Use data from (i) and your answer in (i) to show, on Fig. 11.2, the changes in energy levels giving rise to the dark lines in (a). [2]

[Total: 8]

Page 190: PHYSICS 9702/11 READ THESE INSTRUCTIONS FIRSTmaxpapers.com/wp-content/uploads/2012/11/9702_s17… ·  · 2017-10-17PHYSICS 9702/11 Paper 1 Multiple Choice May/June 2017 1 hour 15

22

9702/41/M/J/17© UCLES 2017

12 One possible nuclear reaction that takes place in a nuclear reactor is given by the equation

23592U + 10n 95

42Mo + 13957La + 21

0n + x 0–1e

Data for the nuclei and particles are given in Fig. 12.1.

nucleus or particle mass / u235

92U 235.1239542Mo 94.945139

57La 138.95510n 1.008630

–1e 5.49 × 10–4

Fig. 12.1

(a) Determine, for this nuclear reaction, the value of x.

x = ...........................................................[1]

Page 191: PHYSICS 9702/11 READ THESE INSTRUCTIONS FIRSTmaxpapers.com/wp-content/uploads/2012/11/9702_s17… ·  · 2017-10-17PHYSICS 9702/11 Paper 1 Multiple Choice May/June 2017 1 hour 15

23

9702/41/M/J/17© UCLES 2017

(b) (i) Show that the energy equivalent to 1.00 u is 934 MeV.

[3]

(ii) Calculate the energy, in MeV, released in this reaction. Give your answer to three significant figures.

energy = ................................................. MeV [3]

(c) Suggest the forms of energy into which the energy calculated in (b)(ii) is transformed.

...................................................................................................................................................

...................................................................................................................................................

...................................................................................................................................................

...............................................................................................................................................[2]

[Total: 9]

Page 192: PHYSICS 9702/11 READ THESE INSTRUCTIONS FIRSTmaxpapers.com/wp-content/uploads/2012/11/9702_s17… ·  · 2017-10-17PHYSICS 9702/11 Paper 1 Multiple Choice May/June 2017 1 hour 15

24

9702/41/M/J/17© UCLES 2017

Permission to reproduce items where third-party owned material protected by copyright is included has been sought and cleared where possible. Every reasonable effort has been made by the publisher (UCLES) to trace copyright holders, but if any items requiring clearance have unwittingly been included, the publisher will be pleased to make amends at the earliest possible opportunity.

To avoid the issue of disclosure of answer-related information to candidates, all copyright acknowledgements are reproduced online in the Cambridge International Examinations Copyright Acknowledgements Booklet. This is produced for each series of examinations and is freely available to download at www.cie.org.uk after the live examination series.

Cambridge International Examinations is part of the Cambridge Assessment Group. Cambridge Assessment is the brand name of University of Cambridge Local Examinations Syndicate (UCLES), which is itself a department of the University of Cambridge.

BLANK PAGE

Page 193: PHYSICS 9702/11 READ THESE INSTRUCTIONS FIRSTmaxpapers.com/wp-content/uploads/2012/11/9702_s17… ·  · 2017-10-17PHYSICS 9702/11 Paper 1 Multiple Choice May/June 2017 1 hour 15

*3828804905*

This document consists of 24 printed pages.

DC (CW/JG) 127310/3© UCLES 2017 [Turn over

Cambridge International ExaminationsCambridge International Advanced Subsidiary and Advanced Level

PHYSICS 9702/42Paper 4 A Level Structured Questions May/June 2017 2 hoursCandidates answer on the Question Paper.No Additional Materials are required.

READ THESE INSTRUCTIONS FIRST

Write your Centre number, candidate number and name on all the work you hand in.Write in dark blue or black pen.You may use an HB pencil for any diagrams or graphs.Do not use staples, paper clips, glue or correction fluid.DO NOT WRITE IN ANY BARCODES.

Answer all questions.

Electronic calculators may be used.You may lose marks if you do not show your working or if you do not use appropriate units.

At the end of the examination, fasten all your work securely together.The number of marks is given in brackets [ ] at the end of each question or part question.

Page 194: PHYSICS 9702/11 READ THESE INSTRUCTIONS FIRSTmaxpapers.com/wp-content/uploads/2012/11/9702_s17… ·  · 2017-10-17PHYSICS 9702/11 Paper 1 Multiple Choice May/June 2017 1 hour 15

2

9702/42/M/J/17© UCLES 2017

Data

speed of light in free space c = 3.00 × 108 m s−1

permeability of free space μ0 = 4π × 10−7 H m−1

permittivity of free space ε0 = 8.85 × 10−12 F m−1

( 14πε0

= 8.99 × 109 m F−1) elementary charge e = 1.60 × 10−19 C

the Planck constant h = 6.63 × 10−34 J s

unified atomic mass unit 1 u = 1.66 × 10−27 kg

rest mass of electron me = 9.11 × 10−31 kg

rest mass of proton mp = 1.67 × 10−27 kg

molar gas constant R = 8.31 J K−1 mol−1

the Avogadro constant NA = 6.02 × 1023 mol−1

the Boltzmann constant k = 1.38 × 10−23 J K−1

gravitational constant G = 6.67 × 10−11 N m2 kg−2

acceleration of free fall g = 9.81 m s−2

Page 195: PHYSICS 9702/11 READ THESE INSTRUCTIONS FIRSTmaxpapers.com/wp-content/uploads/2012/11/9702_s17… ·  · 2017-10-17PHYSICS 9702/11 Paper 1 Multiple Choice May/June 2017 1 hour 15

3

9702/42/M/J/17© UCLES 2017 [Turn over

Formulae

uniformly accelerated motion s = ut + 12 at 2

v2 = u2 + 2as

work done on/by a gas W = pΔV

gravitational potential φ =− Gmr

hydrostatic pressure p = ρgh

pressure of an ideal gas p = 13 NmV 〈c2〉

simple harmonic motion a =−ω 2x

velocity of particle in s.h.m. v = v0 cos ωt v = ± ω ( )x x0

2 2-

Doppler effect fo = fsv

v ± vs

electric potential V = Q4πε0r

capacitors in series 1/C = 1/C1 + 1/C2 + . . .

capacitors in parallel C = C1 + C2 + . . .

energy of charged capacitor W = 12 QV

electric current I = Anvq

resistors in series R = R1 + R2 + . . .

resistors in parallel 1/R = 1/R1 + 1/R2 + . . .

Hall voltage VH = BIntq

alternating current/voltage x = x0 sin ω  t

radioactive decay x = x0exp(−λt )

decay constant λ = 0.693t 1

2

Page 196: PHYSICS 9702/11 READ THESE INSTRUCTIONS FIRSTmaxpapers.com/wp-content/uploads/2012/11/9702_s17… ·  · 2017-10-17PHYSICS 9702/11 Paper 1 Multiple Choice May/June 2017 1 hour 15

4

9702/42/M/J/17© UCLES 2017

Answer all the questions in the spaces provided.

1 (a) Define gravitational field strength.

...................................................................................................................................................

...............................................................................................................................................[1]

(b) The mass of a spherical comet of radius 3.6 km is approximately 1.0 × 1013 kg.

(i) Assuming that the comet has constant density, calculate the gravitational field strength on the surface of the comet.

field strength = ............................................... N kg–1 [2] (ii) A probe having a weight of 960 N on Earth lands on the comet. Using your answer in (i), determine the weight of the probe on the surface of the comet.

weight = ...................................................... N [2]

Page 197: PHYSICS 9702/11 READ THESE INSTRUCTIONS FIRSTmaxpapers.com/wp-content/uploads/2012/11/9702_s17… ·  · 2017-10-17PHYSICS 9702/11 Paper 1 Multiple Choice May/June 2017 1 hour 15

5

9702/42/M/J/17© UCLES 2017 [Turn over

(c) A second comet has a length of approximately 4.5 km and a width of approximately 2.6 km. Its outline is illustrated in Fig. 1.1.

Fig. 1.1

Suggest one similarity and one difference between the gravitational fields at the surface of this comet and at the surface of the comet in (b).

similarity: ...................................................................................................................................

...................................................................................................................................................

difference: .................................................................................................................................

................................................................................................................................................... [2]

[Total: 7]

Page 198: PHYSICS 9702/11 READ THESE INSTRUCTIONS FIRSTmaxpapers.com/wp-content/uploads/2012/11/9702_s17… ·  · 2017-10-17PHYSICS 9702/11 Paper 1 Multiple Choice May/June 2017 1 hour 15

6

9702/42/M/J/17© UCLES 2017

2 (a) The pressure p and volume V of an ideal gas are related to the density ρ of the gas by the expression

p = 31ρ  〈c   2〉.

(i) State what is meant by the symbol 〈c   2〉.

...........................................................................................................................................

.......................................................................................................................................[1]

(ii) Use the expression to show that the mean kinetic energy EK of a gas molecule is given by

EK = 23 kT

where k is the Boltzmann constant and T is the thermodynamic temperature.

[3]

(b) (i) An ideal gas containing 1.0 mol of molecules is heated at constant volume. Use the expression in (a)(ii) to show that the thermal energy required to raise the temperature of the gas by 1.0 K has a value of 2

3 R, where R is the molar gas constant.

[3]

Page 199: PHYSICS 9702/11 READ THESE INSTRUCTIONS FIRSTmaxpapers.com/wp-content/uploads/2012/11/9702_s17… ·  · 2017-10-17PHYSICS 9702/11 Paper 1 Multiple Choice May/June 2017 1 hour 15

7

9702/42/M/J/17© UCLES 2017 [Turn over

(ii) Nitrogen may be assumed to be an ideal gas. The molar mass of nitrogen gas is 28 g mol–1. Use the answer in (b)(i) to calculate a value for the specific heat capacity, in J kg–1 K–1, at

constant volume for nitrogen.

specific heat capacity = .......................................... J kg–1 K–1 [2]

[Total: 9]

Page 200: PHYSICS 9702/11 READ THESE INSTRUCTIONS FIRSTmaxpapers.com/wp-content/uploads/2012/11/9702_s17… ·  · 2017-10-17PHYSICS 9702/11 Paper 1 Multiple Choice May/June 2017 1 hour 15

8

9702/42/M/J/17© UCLES 2017

3 A bar magnet of mass 250 g is suspended from the free end of a spring, as illustrated in Fig. 3.1.

spring

magnet

coil

Fig. 3.1

The magnet hangs so that one pole is near the centre of a coil of wire.

The coil is connected in series with a resistor and a switch. The switch is open.

The magnet is displaced vertically and then allowed to oscillate with one pole remaining inside the coil. The other pole remains outside the coil.

At time t = 0, the magnet is oscillating freely as it passes through its equilibrium position. At time t = 6.0 s, the switch in the circuit is closed.

Page 201: PHYSICS 9702/11 READ THESE INSTRUCTIONS FIRSTmaxpapers.com/wp-content/uploads/2012/11/9702_s17… ·  · 2017-10-17PHYSICS 9702/11 Paper 1 Multiple Choice May/June 2017 1 hour 15

9

9702/42/M/J/17© UCLES 2017 [Turn over

The variation with time t of the vertical displacement y of the magnet is shown in Fig. 3.2.

–2.0

–1.5

–1.0

–0.5

0

0.5

1.0

y / cm1.5

2.0

0 2 4 6 8 10 12 14 16t / s

Fig. 3.2

(a) For the oscillating magnet, use data from Fig. 3.2 to calculate, to two significant figures,

(i) the frequency f,

f = .................................................... Hz [2]

(ii) the energy of the oscillations during the time t = 0 to time t = 6.0 s.

energy = ....................................................... J [3]

Page 202: PHYSICS 9702/11 READ THESE INSTRUCTIONS FIRSTmaxpapers.com/wp-content/uploads/2012/11/9702_s17… ·  · 2017-10-17PHYSICS 9702/11 Paper 1 Multiple Choice May/June 2017 1 hour 15

10

9702/42/M/J/17© UCLES 2017

(b) (i) StateFaraday’slawofelectromagneticinduction.

...........................................................................................................................................

...........................................................................................................................................

...........................................................................................................................................

.......................................................................................................................................[2]

(ii) Use Faraday’s law and energy conservation to explain why the amplitude of theoscillations of the magnet reduces after time t = 6.0 s.

...........................................................................................................................................

...........................................................................................................................................

...........................................................................................................................................

...........................................................................................................................................

...........................................................................................................................................

.......................................................................................................................................[3]

[Total: 10]

Page 203: PHYSICS 9702/11 READ THESE INSTRUCTIONS FIRSTmaxpapers.com/wp-content/uploads/2012/11/9702_s17… ·  · 2017-10-17PHYSICS 9702/11 Paper 1 Multiple Choice May/June 2017 1 hour 15

11

9702/42/M/J/17© UCLES 2017 [Turn over

4 (a) Explain the main principles behind the use of ultrasound to obtain diagnostic information about internal body structures.

...................................................................................................................................................

...................................................................................................................................................

...................................................................................................................................................

...................................................................................................................................................

...................................................................................................................................................

...................................................................................................................................................

...................................................................................................................................................

...................................................................................................................................................

...................................................................................................................................................

...................................................................................................................................................

...................................................................................................................................................

...................................................................................................................................................

...................................................................................................................................................

...................................................................................................................................................

...............................................................................................................................................[6]

Page 204: PHYSICS 9702/11 READ THESE INSTRUCTIONS FIRSTmaxpapers.com/wp-content/uploads/2012/11/9702_s17… ·  · 2017-10-17PHYSICS 9702/11 Paper 1 Multiple Choice May/June 2017 1 hour 15

12

9702/42/M/J/17© UCLES 2017

(b) A parallel beam of ultrasound has intensity I0 as it enters a muscle of thickness 4.6 cm, as illustrated in Fig. 4.1.

4.6 cmmuscle

beam ofultrasound

I0 IT

Fig. 4.1

The intensity of the beam just before it leaves the muscle is IT. The ratio I0 / IT is found to be 2.9.

Calculate the linear attenuation (absorption) coefficient μ of the ultrasound in the layer of muscle.

μ = ................................................. cm–1 [3]

[Total: 9]

Page 205: PHYSICS 9702/11 READ THESE INSTRUCTIONS FIRSTmaxpapers.com/wp-content/uploads/2012/11/9702_s17… ·  · 2017-10-17PHYSICS 9702/11 Paper 1 Multiple Choice May/June 2017 1 hour 15

13

9702/42/M/J/17© UCLES 2017 [Turn over

5 (a) State two advantages of the transmission of data in digital form rather than in analogue form.

1. ...............................................................................................................................................

...................................................................................................................................................

2. ...............................................................................................................................................

................................................................................................................................................... [2]

(b) An analogue signal SI is converted into a digital signal D using an analogue-to-digital converter (ADC). After transmission of the digital signal, it is converted back to an analogue signal ST using a digital-to-analogue converter (DAC), as illustrated in Fig. 5.1.

ADC DACanalogue signal analogue signaldigital signal

SI D ST

Fig. 5.1

(i) Outline the process by which the ADC converts the analogue signal SI into the digital signal D.

...........................................................................................................................................

...........................................................................................................................................

.......................................................................................................................................[2]

(ii) The ADC and the DAC operate with the same sampling rate and the same number of bits in each digital number.

State the effect on the transmitted analogue signal ST when, for the ADC and the DAC,

1. the sampling rate is increased,

....................................................................................................................................

....................................................................................................................................

2. the number of bits in each digital number is increased.

....................................................................................................................................

.................................................................................................................................... [2]

[Total: 6]

Page 206: PHYSICS 9702/11 READ THESE INSTRUCTIONS FIRSTmaxpapers.com/wp-content/uploads/2012/11/9702_s17… ·  · 2017-10-17PHYSICS 9702/11 Paper 1 Multiple Choice May/June 2017 1 hour 15

14

9702/42/M/J/17© UCLES 2017

6 (a) StateCoulomb’slaw.

...................................................................................................................................................

...................................................................................................................................................

...............................................................................................................................................[2]

(b) Two charged metal spheres A and B are situated in a vacuum, as illustrated in Fig. 6.1.

sphere Bsphere AP

x

6.0 cm

Fig. 6.1

The shortest distance between the surfaces of the spheres is 6.0 cm.

A movable point P lies along the line joining the centres of the two spheres, a distance x from the surface of sphere A.

The variation with distance x of the electric field strength E at point P is shown in Fig. 6.2.

10 2 3 4 5 6

10

5

0

–5

–10

–15

x / cm

E / 103 V m–1

Fig. 6.2

Page 207: PHYSICS 9702/11 READ THESE INSTRUCTIONS FIRSTmaxpapers.com/wp-content/uploads/2012/11/9702_s17… ·  · 2017-10-17PHYSICS 9702/11 Paper 1 Multiple Choice May/June 2017 1 hour 15

15

9702/42/M/J/17© UCLES 2017 [Turn over

(i) Use Fig. 6.2 to explain whether the two spheres have charges of the same, or opposite, sign.

...........................................................................................................................................

...........................................................................................................................................

...........................................................................................................................................

.......................................................................................................................................[2]

(ii) A proton is at point P where x = 5.0 cm. Use data from Fig. 6.2 to determine the acceleration of the proton.

acceleration = ................................................. m s–2 [3]

(c) Use data from Fig. 6.2 to state the value of x at which the rate of change of electric potential is maximum. Give the reason for the value you have chosen.

...................................................................................................................................................

...................................................................................................................................................

...............................................................................................................................................[2]

[Total: 9]

Page 208: PHYSICS 9702/11 READ THESE INSTRUCTIONS FIRSTmaxpapers.com/wp-content/uploads/2012/11/9702_s17… ·  · 2017-10-17PHYSICS 9702/11 Paper 1 Multiple Choice May/June 2017 1 hour 15

16

9702/42/M/J/17© UCLES 2017

7 A capacitor consists of two parallel metal plates, separated by an insulator, as shown in Fig. 7.1.

metalplates

insulator

Fig. 7.1

(a) Suggest why, when the capacitor is connected across the terminals of a battery, the capacitor stores energy, not charge.

...................................................................................................................................................

...................................................................................................................................................

...............................................................................................................................................[2]

(b) Define the capacitance of the capacitor.

...................................................................................................................................................

...................................................................................................................................................

...............................................................................................................................................[2]

(c) The capacitor is charged so that the potential difference between its plates is V0. The capacitor is then connected across a resistor for a short time. It is then disconnected. The energy stored in the capacitor is reduced to 16

1 of its initial value.

Determine, in terms of V0, the potential difference across the capacitor.

potential difference = ...........................................................[2]

[Total: 6]

Page 209: PHYSICS 9702/11 READ THESE INSTRUCTIONS FIRSTmaxpapers.com/wp-content/uploads/2012/11/9702_s17… ·  · 2017-10-17PHYSICS 9702/11 Paper 1 Multiple Choice May/June 2017 1 hour 15

17

9702/42/M/J/17© UCLES 2017 [Turn over

8 A student designs a circuit incorporating an operational amplifier (op-amp) as shown in Fig. 8.1.

+

+5 V

+6 V

0 V

–5 VY

X

B GRV

R

R

component C

Fig. 8.1

(a) (i) On Fig. 8.1, draw a circle around the output device. [1]

(ii) State the purpose of this circuit.

...........................................................................................................................................

...........................................................................................................................................

.......................................................................................................................................[2]

(b) The resistors X and Y each have resistance R. When conducting, the LED labelled B emits blue light and the LED labelled G emits green

light.

(i) State whether blue light or green light is emitted when the resistance of component C is greater than the resistance RV of the variable resistor. Explain your answer.

...........................................................................................................................................

...........................................................................................................................................

...........................................................................................................................................

...........................................................................................................................................

.......................................................................................................................................[3]

Page 210: PHYSICS 9702/11 READ THESE INSTRUCTIONS FIRSTmaxpapers.com/wp-content/uploads/2012/11/9702_s17… ·  · 2017-10-17PHYSICS 9702/11 Paper 1 Multiple Choice May/June 2017 1 hour 15

18

9702/42/M/J/17© UCLES 2017

(ii) State and explain what is observed as the resistance of component C is reduced.

...........................................................................................................................................

...........................................................................................................................................

...........................................................................................................................................

...........................................................................................................................................

.......................................................................................................................................[3]

(c) Suggest the function of the variable resistor.

...................................................................................................................................................

...............................................................................................................................................[1]

[Total: 10]

Page 211: PHYSICS 9702/11 READ THESE INSTRUCTIONS FIRSTmaxpapers.com/wp-content/uploads/2012/11/9702_s17… ·  · 2017-10-17PHYSICS 9702/11 Paper 1 Multiple Choice May/June 2017 1 hour 15

19

9702/42/M/J/17© UCLES 2017 [Turn over

9 A Hall probe is placed near to one end of a current-carrying solenoid, as shown in Fig. 9.1.

solenoid

X

Y

Hall probe

Fig. 9.1

The probe is rotated about the axis XY and is then held in a position so that the Hall voltage is maximum.

(a) Explain why

(i) a Hall probe is made from a thin slice of material,

...........................................................................................................................................

...........................................................................................................................................

.......................................................................................................................................[2]

(ii) in order for consistent measurements of magnetic flux density to be made, the current in the probe must be constant.

...........................................................................................................................................

.......................................................................................................................................[1]

Page 212: PHYSICS 9702/11 READ THESE INSTRUCTIONS FIRSTmaxpapers.com/wp-content/uploads/2012/11/9702_s17… ·  · 2017-10-17PHYSICS 9702/11 Paper 1 Multiple Choice May/June 2017 1 hour 15

20

9702/42/M/J/17© UCLES 2017

(b) The probe is now rotated through an angle of 360° about the axis XY. At angle θ = 0, the Hall voltage VH has maximum value VMAX .

On Fig. 9.2, sketch the variation with angle θ of the Hall voltage VH for one complete revolution of the probe about axis XY.

90 180 270 36000

+

V HV MAX

/ °θ

Fig. 9.2 [3]

[Total: 6]

10 (a) Briefly describe two phenomena associated with the photoelectric effect that cannot be explained using a wave theory of light.

1. ...............................................................................................................................................

...................................................................................................................................................

2. ...............................................................................................................................................

................................................................................................................................................... [2]

(b) The maximum energy EMAX of electrons emitted from a metal surface when illuminated by light of wavelength λ is given by the expression

EMAX = hc  (1λ – 1λ0

) where h is the Planck constant and c is the speed of light.

(i) Identify the symbol λ0.

.......................................................................................................................................[1]

Page 213: PHYSICS 9702/11 READ THESE INSTRUCTIONS FIRSTmaxpapers.com/wp-content/uploads/2012/11/9702_s17… ·  · 2017-10-17PHYSICS 9702/11 Paper 1 Multiple Choice May/June 2017 1 hour 15

21

9702/42/M/J/17© UCLES 2017 [Turn over

(ii) The variation with 1λ of EMAX for the metal surface is shown in Fig. 10.1.

/ 106 m–1λ1

1.5 2.0 2.5 3.0 3.5 4.00

1

2

3

4

EMAX / 10–19 J

Fig. 10.1

1. Use Fig. 10.1 to determine the magnitude of λ0.

λ0 = ...................................................... m [1]

2. Use the gradient of Fig. 10.1 to determine a value for the Planck constant h.

h = ..................................................... J s [3]

(c) The metal surface in (b) becomes oxidised. Photoelectric emission is still observed but the work function energy is increased.

On Fig. 10.1, draw a line to show the variation with 1λ of EMAX for the oxidised surface. [2]

[Total: 9]

Page 214: PHYSICS 9702/11 READ THESE INSTRUCTIONS FIRSTmaxpapers.com/wp-content/uploads/2012/11/9702_s17… ·  · 2017-10-17PHYSICS 9702/11 Paper 1 Multiple Choice May/June 2017 1 hour 15

22

9702/42/M/J/17© UCLES 2017

11 An electron has charge –q and mass m. It is accelerated from rest in a vacuum through a potential difference V.

(a) Show that the momentum p of the accelerated electron is given by

p = ( )mqV2 .

[2]

(b) The potential difference V through which the electron is accelerated is 120 V.

(i) State what is meant by the de Broglie wavelength.

...........................................................................................................................................

...........................................................................................................................................

.......................................................................................................................................[2]

(ii) Calculate the de Broglie wavelength of the electron.

wavelength = ...................................................... m [3]

(c) The separation of copper atoms in a copper crystal is approximately 2 × 10–10 m.

By reference to your answer in (b)(ii), suggest whether electron diffraction could be observed using a beam of electrons that have been accelerated through a potential difference of 120 V and are then incident on a thin copper crystal.

...................................................................................................................................................

...................................................................................................................................................

...............................................................................................................................................[2]

[Total: 9]

Page 215: PHYSICS 9702/11 READ THESE INSTRUCTIONS FIRSTmaxpapers.com/wp-content/uploads/2012/11/9702_s17… ·  · 2017-10-17PHYSICS 9702/11 Paper 1 Multiple Choice May/June 2017 1 hour 15

23

9702/42/M/J/17© UCLES 2017 [Turn over

12 One nuclear reaction that can take place in a nuclear reactor may be represented, in part, by the equation

23592 U + 10  n 95

42 Mo + 13957 La + 21

0  n + …………. + energy

Data for a nucleus and some particles are given in Fig. 12.1.

nucleus or particle mass / u13957 La 138.955

10  n 1.00863

11  p 1.00728

0–1  e 5.49 × 10–4

Fig. 12.1

(a) Complete the nuclear reaction shown above. [1]

(b) (i) Show that the energy equivalent to 1.00 u is 934 MeV.

[3]

(ii) Calculate the binding energy per nucleon, in MeV, of lanthanum-139 (13957 La).

binding energy per nucleon = ................................................ MeV [3]

Question 12 continues on the next page.

Page 216: PHYSICS 9702/11 READ THESE INSTRUCTIONS FIRSTmaxpapers.com/wp-content/uploads/2012/11/9702_s17… ·  · 2017-10-17PHYSICS 9702/11 Paper 1 Multiple Choice May/June 2017 1 hour 15

24

9702/42/M/J/17© UCLES 2017

(c) State and explain whether the binding energy per nucleon of uranium-235 (23592 U) will be

greater, equal to or less than your answer in (b)(ii).

...................................................................................................................................................

...................................................................................................................................................

...................................................................................................................................................

...............................................................................................................................................[3]

[Total: 10]

Permission to reproduce items where third-party owned material protected by copyright is included has been sought and cleared where possible. Every reasonable effort has been made by the publisher (UCLES) to trace copyright holders, but if any items requiring clearance have unwittingly been included, the publisher will be pleased to make amends at the earliest possible opportunity.

To avoid the issue of disclosure of answer-related information to candidates, all copyright acknowledgements are reproduced online in the Cambridge International Examinations Copyright Acknowledgements Booklet. This is produced for each series of examinations and is freely available to download at www.cie.org.uk after the live examination series.

Cambridge International Examinations is part of the Cambridge Assessment Group. Cambridge Assessment is the brand name of University of Cambridge Local Examinations Syndicate (UCLES), which is itself a department of the University of Cambridge.

Page 217: PHYSICS 9702/11 READ THESE INSTRUCTIONS FIRSTmaxpapers.com/wp-content/uploads/2012/11/9702_s17… ·  · 2017-10-17PHYSICS 9702/11 Paper 1 Multiple Choice May/June 2017 1 hour 15

This document consists of 22 printed pages and 2 blank pages.

DC (LK) 143917© UCLES 2017 [Turn over

Cambridge International ExaminationsCambridge International Advanced Subsidiary and Advanced Level

*3781213391*

PHYSICS 9702/43Paper 4 A Level Structured Questions May/June 2017 2 hoursCandidates answer on the Question Paper.No Additional Materials are required.

READ THESE INSTRUCTIONS FIRST

Write your Centre number, candidate number and name on all the work you hand in.Write in dark blue or black pen.You may use an HB pencil for any diagrams or graphs.Do not use staples, paper clips, glue or correction fluid.DO NOT WRITE IN ANY BARCODES.

Answer all questions.

Electronic calculators may be used.You may lose marks if you do not show your working or if you do not use appropriate units.

At the end of the examination, fasten all your work securely together.The number of marks is given in brackets [ ] at the end of each question or part question.

Page 218: PHYSICS 9702/11 READ THESE INSTRUCTIONS FIRSTmaxpapers.com/wp-content/uploads/2012/11/9702_s17… ·  · 2017-10-17PHYSICS 9702/11 Paper 1 Multiple Choice May/June 2017 1 hour 15

2

9702/43/M/J/17© UCLES 2017

Data

speed of light in free space c = 3.00 × 108 m s−1

permeability of free space μ0 = 4π × 10−7 H m−1

permittivity of free space ε0 = 8.85 × 10−12 F m−1

( 14πε0

= 8.99 × 109 m F−1) elementary charge e = 1.60 × 10−19 C

the Planck constant h = 6.63 × 10−34 J s

unified atomic mass unit 1 u = 1.66 × 10−27 kg

rest mass of electron me = 9.11 × 10−31 kg

rest mass of proton mp = 1.67 × 10−27 kg

molar gas constant R = 8.31 J K−1 mol−1

the Avogadro constant NA = 6.02 × 1023 mol−1

the Boltzmann constant k = 1.38 × 10−23 J K−1

gravitational constant G = 6.67 × 10−11 N m2 kg−2

acceleration of free fall g = 9.81 m s−2

Page 219: PHYSICS 9702/11 READ THESE INSTRUCTIONS FIRSTmaxpapers.com/wp-content/uploads/2012/11/9702_s17… ·  · 2017-10-17PHYSICS 9702/11 Paper 1 Multiple Choice May/June 2017 1 hour 15

3

9702/43/M/J/17© UCLES 2017 [Turn over

Formulae

uniformly accelerated motion s = ut + 12 at 2

v2 = u2 + 2as

work done on/by a gas W = pΔV

gravitational potential φ =− Gmr

hydrostatic pressure p = ρgh

pressure of an ideal gas p = 13 NmV 〈c2〉

simple harmonic motion a =−ω 2x

velocity of particle in s.h.m. v = v0 cos ωt v = ± ω ( )x x0

2 2-

Doppler effect fo = fsv

v ± vs

electric potential V = Q4πε0r

capacitors in series 1/C = 1/C1 + 1/C2 + . . .

capacitors in parallel C = C1 + C2 + . . .

energy of charged capacitor W = 12 QV

electric current I = Anvq

resistors in series R = R1 + R2 + . . .

resistors in parallel 1/R = 1/R1 + 1/R2 + . . .

Hall voltage VH = BIntq

alternating current/voltage x = x0 sin ω  t

radioactive decay x = x0exp(−λt )

decay constant λ = 0.693t 1

2

Page 220: PHYSICS 9702/11 READ THESE INSTRUCTIONS FIRSTmaxpapers.com/wp-content/uploads/2012/11/9702_s17… ·  · 2017-10-17PHYSICS 9702/11 Paper 1 Multiple Choice May/June 2017 1 hour 15

4

9702/43/M/J/17© UCLES 2017

Answer all the questions in the spaces provided.

1 (a) Explain how a satellite may be in a circular orbit around a planet.

...................................................................................................................................................

...................................................................................................................................................

...............................................................................................................................................[2]

(b) The Earth and the Moon may be considered to be uniform spheres that are isolated in space. The Earth has radius R and mean density ρ. The Moon, mass m, is in a circular orbit about the Earth with radius nR, as illustrated in Fig. 1.1.

Moon

Earthradius R

nR

Fig. 1.1

The Moon makes one complete orbit of the Earth in time T. Show that the mean density ρ of the Earth is given by the expression

ρ = 3πn3

GT 2 .

[4]

Page 221: PHYSICS 9702/11 READ THESE INSTRUCTIONS FIRSTmaxpapers.com/wp-content/uploads/2012/11/9702_s17… ·  · 2017-10-17PHYSICS 9702/11 Paper 1 Multiple Choice May/June 2017 1 hour 15

5

9702/43/M/J/17© UCLES 2017 [Turn over

(c) The radius R of the Earth is 6.38 × 103 km and the distance between the centre of the Earth and the centre of the Moon is 3.84 × 105 km.

The period T of the orbit of the Moon about the Earth is 27.3 days. Use the expression in (b) to calculate ρ.

ρ = ............................................... kg m–3 [3]

[Total: 9]

Page 222: PHYSICS 9702/11 READ THESE INSTRUCTIONS FIRSTmaxpapers.com/wp-content/uploads/2012/11/9702_s17… ·  · 2017-10-17PHYSICS 9702/11 Paper 1 Multiple Choice May/June 2017 1 hour 15

6

9702/43/M/J/17© UCLES 2017

2 A bar magnet of mass 180 g is suspended from the free end of a spring, as illustrated in Fig. 2.1.

spring

magnet

coil

Fig. 2.1

The magnet hangs so that one pole is near the centre of a coil of wire.

The coil is connected in series with a resistor and a switch. The switch is open.

The magnet is displaced vertically and then allowed to oscillate with one pole remaining inside the coil. The other pole remains outside the coil.

At time t = 0, the magnet is oscillating freely as it passes through its equilibrium position. At time t = 3.0 s, the switch in the circuit is closed.

The variation with time t of the vertical displacement y of the magnet is shown in Fig. 2.2.

–2.0

–1.5

–1.0

–0.5

0

0.5

1.0y / cm

1.5

2.0

0 1 2 3 4 5 6 7 8 9t / s

Fig. 2.2

Page 223: PHYSICS 9702/11 READ THESE INSTRUCTIONS FIRSTmaxpapers.com/wp-content/uploads/2012/11/9702_s17… ·  · 2017-10-17PHYSICS 9702/11 Paper 1 Multiple Choice May/June 2017 1 hour 15

7

9702/43/M/J/17© UCLES 2017 [Turn over

(a) Determine, to two significant figures, the frequency of oscillation of the magnet.

frequency = .................................................... Hz [2]

(b) State whether the closing of the switch gives rise to light, heavy or critical damping.

...............................................................................................................................................[1]

(c) Calculate the change in the energy ΔE of oscillation of the magnet between time t = 2.7 s and time t = 7.5 s. Explain your working.

ΔE = ....................................................... J [6]

[Total: 9]

Page 224: PHYSICS 9702/11 READ THESE INSTRUCTIONS FIRSTmaxpapers.com/wp-content/uploads/2012/11/9702_s17… ·  · 2017-10-17PHYSICS 9702/11 Paper 1 Multiple Choice May/June 2017 1 hour 15

8

9702/43/M/J/17© UCLES 2017

3 The digital transmission of speech may be illustrated using the block diagram of Fig. 3.1.

ADCserial -to -parallel

converteroptic fibreX Y

Fig. 3.1

(a) (i) State what is meant by a digital signal.

...........................................................................................................................................

.......................................................................................................................................[1]

(ii) State the names of the components labelled X and Y on Fig. 3.1.

X: ......................................................................................................................................

Y: ...................................................................................................................................... [2]

(iii) Describe the function of the ADC.

...........................................................................................................................................

...........................................................................................................................................

.......................................................................................................................................[2]

(b) The optic fibre has length 84 km and the attenuation per unit length in the fibre is 0.19 dB km–1.

The input power to the optic fibre is 9.7 mW. At the output from the optic fibre, the signal-to-noise ratio is 28 dB.

Calculate

(i) in dB, the ratio

input power to optic fibrenoise power at output of optic fibre,

ratio = .................................................... dB [2]

Page 225: PHYSICS 9702/11 READ THESE INSTRUCTIONS FIRSTmaxpapers.com/wp-content/uploads/2012/11/9702_s17… ·  · 2017-10-17PHYSICS 9702/11 Paper 1 Multiple Choice May/June 2017 1 hour 15

9

9702/43/M/J/17© UCLES 2017 [Turn over

(ii) the noise power at the output of the optic fibre.

noise power = ..................................................... W [3]

[Total: 10]

Page 226: PHYSICS 9702/11 READ THESE INSTRUCTIONS FIRSTmaxpapers.com/wp-content/uploads/2012/11/9702_s17… ·  · 2017-10-17PHYSICS 9702/11 Paper 1 Multiple Choice May/June 2017 1 hour 15

10

9702/43/M/J/17© UCLES 2017

4 (a) Describe the motion of molecules in a gas, according to the kinetic theory of gases.

...................................................................................................................................................

...................................................................................................................................................

...............................................................................................................................................[2]

(b) Describe what is observed when viewing Brownian motion that provides evidence for your answer in (a).

...................................................................................................................................................

...................................................................................................................................................

...............................................................................................................................................[2]

(c) At a pressure of 1.05 × 105 Pa and a temperature of 27 °C, 1.00 mol of helium gas has a volume of 0.0240 m3.

The mass of 1.00 mol of helium gas, assumed to be an ideal gas, is 4.00 g.

(i) Calculate the root-mean-square (r.m.s.) speed of an atom of helium gas for a temperature of 27 °C.

r.m.s. speed = ................................................. m s–1 [3]

(ii) Using your answer in (i), calculate the r.m.s. speed of the atoms at 177 °C.

r.m.s. speed = ................................................. m s–1 [3]

[Total: 10]

Page 227: PHYSICS 9702/11 READ THESE INSTRUCTIONS FIRSTmaxpapers.com/wp-content/uploads/2012/11/9702_s17… ·  · 2017-10-17PHYSICS 9702/11 Paper 1 Multiple Choice May/June 2017 1 hour 15

11

9702/43/M/J/17© UCLES 2017 [Turn over

5 An α-particle is travelling in a vacuum towards the centre of a gold nucleus, as illustrated in Fig. 5.1.

gold nucleus α-particle

charge 79e energy 7.7 10–13 J×

Fig. 5.1

The gold nucleus has charge 79e. The gold nucleus and the α-particle may be assumed to behave as point charges. At a large distance from the gold nucleus, the α-particle has energy 7.7 × 10–13 J.

(a) The α-particle does not collide with the gold nucleus. Show that the radius of the gold nucleus must be less than 4.7 × 10–14 m.

[3]

(b) Determine the acceleration of the α-particle for a separation of 4.7 × 10–14 m between the centres of the gold nucleus and of the α-particle.

acceleration = ................................................. m s–2 [3]

(c) In an α-particle scattering experiment, the beam of α-particles is incident on a very thin gold foil.

Suggest why the gold foil must be very thin.

...................................................................................................................................................

...............................................................................................................................................[1]

[Total: 7]

Page 228: PHYSICS 9702/11 READ THESE INSTRUCTIONS FIRSTmaxpapers.com/wp-content/uploads/2012/11/9702_s17… ·  · 2017-10-17PHYSICS 9702/11 Paper 1 Multiple Choice May/June 2017 1 hour 15

12

9702/43/M/J/17© UCLES 2017

6 A comparator circuit is designed to switch on a mains lamp when the ambient light level reaches a set value.

An incomplete diagram of the circuit is shown in Fig. 6.1.

+

+5 V

–5 V

D

RV

6 V

Fig. 6.1

(a) (i) A relay is required as part of the output device. This is not shown in Fig. 6.1. Explain why a relay is required.

...........................................................................................................................................

...........................................................................................................................................

.......................................................................................................................................[2]

(ii) On Fig. 6.1, draw the symbol for a relay connected in the circuit as part of the output device. [2]

(b) Describe the function of

(i) the variable resistor RV,

...........................................................................................................................................

.......................................................................................................................................[1]

(ii) the diode D.

...........................................................................................................................................

.......................................................................................................................................[1]

Page 229: PHYSICS 9702/11 READ THESE INSTRUCTIONS FIRSTmaxpapers.com/wp-content/uploads/2012/11/9702_s17… ·  · 2017-10-17PHYSICS 9702/11 Paper 1 Multiple Choice May/June 2017 1 hour 15

13

9702/43/M/J/17© UCLES 2017 [Turn over

(c) State whether the lamp will switch on as the light level increases or as it decreases. Explain your answer.

...................................................................................................................................................

...................................................................................................................................................

...................................................................................................................................................

...................................................................................................................................................

...............................................................................................................................................[3]

[Total: 9]

Page 230: PHYSICS 9702/11 READ THESE INSTRUCTIONS FIRSTmaxpapers.com/wp-content/uploads/2012/11/9702_s17… ·  · 2017-10-17PHYSICS 9702/11 Paper 1 Multiple Choice May/June 2017 1 hour 15

14

9702/43/M/J/17© UCLES 2017

7 An electron having charge –q and mass m is accelerated from rest in a vacuum through a potential difference V.

The electron then enters a region of uniform magnetic field of magnetic flux density B, as shown in Fig. 7.1.

path ofelectron

uniform magneticfield into planeof paper

Fig. 7.1

The direction of the uniform magnetic field is into the plane of the paper. The velocity of the electron as it enters the magnetic field is normal to the magnetic field. The radius of the circular path of the electron in the magnetic field is r.

(a) Explain why the path of the electron in the magnetic field is the arc of a circle.

...................................................................................................................................................

...................................................................................................................................................

...................................................................................................................................................

...............................................................................................................................................[3]

(b) Show that the magnitude p of the momentum of the electron as it enters the magnetic field is given by

p = (2mqV ).

[2]

Page 231: PHYSICS 9702/11 READ THESE INSTRUCTIONS FIRSTmaxpapers.com/wp-content/uploads/2012/11/9702_s17… ·  · 2017-10-17PHYSICS 9702/11 Paper 1 Multiple Choice May/June 2017 1 hour 15

15

9702/43/M/J/17© UCLES 2017 [Turn over

(c) The potential difference V is 120 V. The radius r of the circular arc is 7.4 cm.

Determine the magnitude B of the magnetic flux density.

B = ....................................................... T [3]

(d) The potential difference V in (c) is increased. The magnetic flux density B remains unchanged.

By reference to the momentum of the electron, explain the effect of this increase on the radius r of the path of the electron in the magnetic field.

...................................................................................................................................................

...................................................................................................................................................

...............................................................................................................................................[2]

[Total: 10]

Page 232: PHYSICS 9702/11 READ THESE INSTRUCTIONS FIRSTmaxpapers.com/wp-content/uploads/2012/11/9702_s17… ·  · 2017-10-17PHYSICS 9702/11 Paper 1 Multiple Choice May/June 2017 1 hour 15

16

9702/43/M/J/17© UCLES 2017

8 Explain the main principles behind the use of nuclear magnetic resonance imaging (NMRI) to obtain information about internal body structures.

..........................................................................................................................................................

..........................................................................................................................................................

..........................................................................................................................................................

..........................................................................................................................................................

..........................................................................................................................................................

..........................................................................................................................................................

..........................................................................................................................................................

..........................................................................................................................................................

..........................................................................................................................................................

..........................................................................................................................................................

..........................................................................................................................................................

..........................................................................................................................................................

..........................................................................................................................................................

......................................................................................................................................................[8]

[Total: 8]

Page 233: PHYSICS 9702/11 READ THESE INSTRUCTIONS FIRSTmaxpapers.com/wp-content/uploads/2012/11/9702_s17… ·  · 2017-10-17PHYSICS 9702/11 Paper 1 Multiple Choice May/June 2017 1 hour 15

17

9702/43/M/J/17© UCLES 2017 [Turn over

9 A simple transformer is illustrated in Fig. 9.1.

output

laminatediron core

input

Fig. 9.1

(a) (i) State why the transformer has an iron core, rather than having no core.

...........................................................................................................................................

.......................................................................................................................................[1]

(ii) Explain why the core is laminated.

...........................................................................................................................................

...........................................................................................................................................

.......................................................................................................................................[2]

(b) By reference to the action of a transformer, explain why the input to the transformer is an alternating voltage, rather than a constant voltage.

...................................................................................................................................................

...................................................................................................................................................

...................................................................................................................................................

...................................................................................................................................................

...............................................................................................................................................[3]

[Total: 6]

Page 234: PHYSICS 9702/11 READ THESE INSTRUCTIONS FIRSTmaxpapers.com/wp-content/uploads/2012/11/9702_s17… ·  · 2017-10-17PHYSICS 9702/11 Paper 1 Multiple Choice May/June 2017 1 hour 15

18

9702/43/M/J/17© UCLES 2017

BLANK PAGE

Page 235: PHYSICS 9702/11 READ THESE INSTRUCTIONS FIRSTmaxpapers.com/wp-content/uploads/2012/11/9702_s17… ·  · 2017-10-17PHYSICS 9702/11 Paper 1 Multiple Choice May/June 2017 1 hour 15

19

9702/43/M/J/17© UCLES 2017 [Turn over

10 (a) State

(i) what is meant by the hardness of an X-ray beam,

...........................................................................................................................................

...........................................................................................................................................

.......................................................................................................................................[2]

(ii) how the hardness of an X-ray beam from an X-ray tube is increased.

...........................................................................................................................................

.......................................................................................................................................[1]

(b) The same parallel beam of X-ray radiation is incident, separately, on samples of bone and of muscle.

Data for the thickness x of the samples of bone and of muscle, together with the linear attenuation (absorption) coefficients μ of the radiation in bone and in muscle, are given in Fig. 10.1.

x / cm μ / cm–1

bone 1.5 2.9

muscle 4.0 0.95

Fig. 10.1

Determine the ratio

intensity transmitted through boneintensity transmitted through muscle

.

ratio = .......................................................... [2]

[Total: 5]

Page 236: PHYSICS 9702/11 READ THESE INSTRUCTIONS FIRSTmaxpapers.com/wp-content/uploads/2012/11/9702_s17… ·  · 2017-10-17PHYSICS 9702/11 Paper 1 Multiple Choice May/June 2017 1 hour 15

20

9702/43/M/J/17© UCLES 2017

11 A beam of light consists of a continuous range of wavelengths from 420 nm to 740 nm. The light passes through a cloud of cool gas, as shown in Fig. 11.1.

incident lightcool gas

emergent light

wavelengths 420 nm – 740 nm

Fig. 11.1

(a) The spectrum of the light emerging from the cloud of cool gas is viewed using a diffraction grating.

Explain why this spectrum contains a number of dark lines.

...................................................................................................................................................

...................................................................................................................................................

...................................................................................................................................................

...................................................................................................................................................

...................................................................................................................................................

...............................................................................................................................................[4]

(b) Some of the electron energy levels of the atoms in the cloud of gas are represented in Fig. 11.2.

energy

– 0.38 eV– 0.54 eV– 0.85 eV

– 1.5 eV

– 3.4 eV

– 13.6 eV

Fig. 11.2 (not to scale)

Page 237: PHYSICS 9702/11 READ THESE INSTRUCTIONS FIRSTmaxpapers.com/wp-content/uploads/2012/11/9702_s17… ·  · 2017-10-17PHYSICS 9702/11 Paper 1 Multiple Choice May/June 2017 1 hour 15

21

9702/43/M/J/17© UCLES 2017 [Turn over

(i) Light of wavelength 420 nm has a photon energy of 2.96 eV. Calculate the photon energy, in eV, of light of wavelength 740 nm.

photon energy = .................................................... eV [2]

(ii) Use data from (i) and your answer in (i) to show, on Fig. 11.2, the changes in energy levels giving rise to the dark lines in (a). [2]

[Total: 8]

Page 238: PHYSICS 9702/11 READ THESE INSTRUCTIONS FIRSTmaxpapers.com/wp-content/uploads/2012/11/9702_s17… ·  · 2017-10-17PHYSICS 9702/11 Paper 1 Multiple Choice May/June 2017 1 hour 15

22

9702/43/M/J/17© UCLES 2017

12 One possible nuclear reaction that takes place in a nuclear reactor is given by the equation

23592U + 10n 95

42Mo + 13957La + 21

0n + x 0–1e

Data for the nuclei and particles are given in Fig. 12.1.

nucleus or particle mass / u235

92U 235.1239542Mo 94.945139

57La 138.95510n 1.008630

–1e 5.49 × 10–4

Fig. 12.1

(a) Determine, for this nuclear reaction, the value of x.

x = ...........................................................[1]

Page 239: PHYSICS 9702/11 READ THESE INSTRUCTIONS FIRSTmaxpapers.com/wp-content/uploads/2012/11/9702_s17… ·  · 2017-10-17PHYSICS 9702/11 Paper 1 Multiple Choice May/June 2017 1 hour 15

23

9702/43/M/J/17© UCLES 2017

(b) (i) Show that the energy equivalent to 1.00 u is 934 MeV.

[3]

(ii) Calculate the energy, in MeV, released in this reaction. Give your answer to three significant figures.

energy = ................................................. MeV [3]

(c) Suggest the forms of energy into which the energy calculated in (b)(ii) is transformed.

...................................................................................................................................................

...................................................................................................................................................

...................................................................................................................................................

...............................................................................................................................................[2]

[Total: 9]

Page 240: PHYSICS 9702/11 READ THESE INSTRUCTIONS FIRSTmaxpapers.com/wp-content/uploads/2012/11/9702_s17… ·  · 2017-10-17PHYSICS 9702/11 Paper 1 Multiple Choice May/June 2017 1 hour 15

24

9702/43/M/J/17© UCLES 2017

Permission to reproduce items where third-party owned material protected by copyright is included has been sought and cleared where possible. Every reasonable effort has been made by the publisher (UCLES) to trace copyright holders, but if any items requiring clearance have unwittingly been included, the publisher will be pleased to make amends at the earliest possible opportunity.

To avoid the issue of disclosure of answer-related information to candidates, all copyright acknowledgements are reproduced online in the Cambridge International Examinations Copyright Acknowledgements Booklet. This is produced for each series of examinations and is freely available to download at www.cie.org.uk after the live examination series.

Cambridge International Examinations is part of the Cambridge Assessment Group. Cambridge Assessment is the brand name of University of Cambridge Local Examinations Syndicate (UCLES), which is itself a department of the University of Cambridge.

BLANK PAGE

Page 241: PHYSICS 9702/11 READ THESE INSTRUCTIONS FIRSTmaxpapers.com/wp-content/uploads/2012/11/9702_s17… ·  · 2017-10-17PHYSICS 9702/11 Paper 1 Multiple Choice May/June 2017 1 hour 15

*5126977804*

This document consists of 8 printed pages.

DC (CW/SG) 127830/2© UCLES 2017 [Turn over

Cambridge International ExaminationsCambridge International Advanced Subsidiary and Advanced Level

PHYSICS 9702/51Paper 5 Planning, Analysis and Evaluation May/June 2017 1 hour 15 minutesCandidates answer on the Question Paper.No Additional Materials are required.

READ THESE INSTRUCTIONS FIRST

Write your Centre number, candidate number and name on all the work you hand in.Write in dark blue or black pen.You may use an HB pencil for any diagrams or graphs.Do not use staples, paper clips, glue or correction fluid.DO NOT WRITE IN ANY BARCODES.

Answer all questions.

Electronic calculators may be used. You may lose marks if you do not show your working or if you do not use appropriate units.

At the end of the examination, fasten all your work securely together.The number of marks is given in brackets [ ] at the end of each question or part question.

Page 242: PHYSICS 9702/11 READ THESE INSTRUCTIONS FIRSTmaxpapers.com/wp-content/uploads/2012/11/9702_s17… ·  · 2017-10-17PHYSICS 9702/11 Paper 1 Multiple Choice May/June 2017 1 hour 15

2

9702/51/M/J/17© UCLES 2017

1 A student is investigating the motion of a wooden block on an inclined plane, as shown in Fig. 1.1. A falling body causes the wooden block to accelerate.

θ

P

Q

pulley

falling bodywooden block

string

plane

Fig. 1.1

The wooden block is initially at rest at point P and has velocity v at point Q.

It is suggested that the relationship between v and the angle θ of the plane to the horizontal is

(B + m)v  22s = Bg – mg sin θ

where B is the mass of the falling body, m is the mass of the wooden block, s is the distance between P and Q and g is the acceleration of free fall.

Design a laboratory experiment to test the relationship between v and θ. Explain how your results could be used to determine a value for g. You should draw a diagram, on page 3, showing the arrangement of your equipment. In your account you should pay particular attention to

• the procedure to be followed, • the measurements to be taken, • the control of variables, • the analysis of the data, • any safety precautions to be taken. [15]

Page 243: PHYSICS 9702/11 READ THESE INSTRUCTIONS FIRSTmaxpapers.com/wp-content/uploads/2012/11/9702_s17… ·  · 2017-10-17PHYSICS 9702/11 Paper 1 Multiple Choice May/June 2017 1 hour 15

3

9702/51/M/J/17© UCLES 2017 [Turn over

Diagram

..................................................................................................................................................................

..................................................................................................................................................................

..................................................................................................................................................................

..................................................................................................................................................................

..................................................................................................................................................................

..................................................................................................................................................................

..................................................................................................................................................................

..................................................................................................................................................................

..................................................................................................................................................................

..................................................................................................................................................................

..................................................................................................................................................................

..................................................................................................................................................................

..................................................................................................................................................................

..................................................................................................................................................................

..................................................................................................................................................................

Page 244: PHYSICS 9702/11 READ THESE INSTRUCTIONS FIRSTmaxpapers.com/wp-content/uploads/2012/11/9702_s17… ·  · 2017-10-17PHYSICS 9702/11 Paper 1 Multiple Choice May/June 2017 1 hour 15

4

9702/51/M/J/17© UCLES 2017

..................................................................................................................................................................

..................................................................................................................................................................

..................................................................................................................................................................

..................................................................................................................................................................

..................................................................................................................................................................

..................................................................................................................................................................

..................................................................................................................................................................

..................................................................................................................................................................

..................................................................................................................................................................

..................................................................................................................................................................

..................................................................................................................................................................

..................................................................................................................................................................

..................................................................................................................................................................

..................................................................................................................................................................

..................................................................................................................................................................

..................................................................................................................................................................

..................................................................................................................................................................

..................................................................................................................................................................

..................................................................................................................................................................

..................................................................................................................................................................

..................................................................................................................................................................

..................................................................................................................................................................

..................................................................................................................................................................

..................................................................................................................................................................

..................................................................................................................................................................

..................................................................................................................................................................

..................................................................................................................................................................

[Total: 15]

Page 245: PHYSICS 9702/11 READ THESE INSTRUCTIONS FIRSTmaxpapers.com/wp-content/uploads/2012/11/9702_s17… ·  · 2017-10-17PHYSICS 9702/11 Paper 1 Multiple Choice May/June 2017 1 hour 15

5

9702/51/M/J/17© UCLES 2017 [Turn over

2 A student is investigating how the time for an electrical pulse to travel in a coaxial cable varies with the length of the cable. The pulse is reflected at one end of the cable. An oscilloscope is used to display the initial pulse and the reflected pulse.

The trace on the oscilloscope is shown in Fig. 2.1.

d

Fig. 2.1

The time t for the pulse to travel to the end of the cable and back is determined by measuring the distance d between the pulses on the screen, and then using the time-base and the relationship

t = d × time-base.

The initial length of the cable is L. A total length Z is removed from the cable and the experiment is repeated.

It is suggested that t and Z are related by the equation

v = 2 (L – Z   )t

where v is the speed of the pulse.

(a) A graph is plotted of t on the y-axis against Z on the x-axis.

Determine expressions for the gradient and the y-intercept.

gradient = ...............................................................

y-intercept = ............................................................... [1]

Page 246: PHYSICS 9702/11 READ THESE INSTRUCTIONS FIRSTmaxpapers.com/wp-content/uploads/2012/11/9702_s17… ·  · 2017-10-17PHYSICS 9702/11 Paper 1 Multiple Choice May/June 2017 1 hour 15

6

9702/51/M/J/17© UCLES 2017

(b) Values of Z and d are given in Fig. 2.2. The time-base is 0.1 µs cm–1.

Z / m d / cm t / µs

0.0 8.0 ± 0.1

4.0 7.7 ± 0.1

8.0 7.3 ± 0.1

12.0 7.0 ± 0.1

16.0 6.6 ± 0.1

20.0 6.2 ± 0.1

Fig. 2.2

Calculate and record values of t / µs in Fig. 2.2. Include the absolute uncertainties in t. [2]

(c) (i) Plot a graph of t / µs against Z / m. Include error bars for t. [2]

(ii) Draw the straight line of best fit and a worst acceptable straight line on your graph. Both lines should be clearly labelled. [2]

(iii) Determine the gradient of the line of best fit. Include the absolute uncertainty in your answer.

gradient = .......................................................... [2]

Page 247: PHYSICS 9702/11 READ THESE INSTRUCTIONS FIRSTmaxpapers.com/wp-content/uploads/2012/11/9702_s17… ·  · 2017-10-17PHYSICS 9702/11 Paper 1 Multiple Choice May/June 2017 1 hour 15

7

9702/51/M/J/17© UCLES 2017 [Turn over

Z / m

0.600 4 8 12 16 20 24

0.62

0.64

0.66

0.68

0.70

0.72

0.74

0.76

0.78

0.80

0.82

t / μs

Page 248: PHYSICS 9702/11 READ THESE INSTRUCTIONS FIRSTmaxpapers.com/wp-content/uploads/2012/11/9702_s17… ·  · 2017-10-17PHYSICS 9702/11 Paper 1 Multiple Choice May/June 2017 1 hour 15

8

9702/51/M/J/17© UCLES 2017

(iv) Determine the y-intercept of the line of best fit. Include the absolute uncertainty in your answer.

y-intercept = .......................................................... [2]

(d) (i) Using your answers to (a), (c)(iii) and (c)(iv), determine the values of L and v. Include appropriate units.

L = ...............................................................

v = ............................................................... [2]

(ii) Determine the percentage uncertainties in L and v.

percentage uncertainty in L = ........................................................... %

percentage uncertainty in v = ........................................................... %[2]

[Total: 15]

To avoid the issue of disclosure of answer-related information to candidates, all copyright acknowledgements are reproduced online in the Cambridge International Examinations Copyright Acknowledgements Booklet. This is produced for each series of examinations and is freely available to download at www.cie.org.uk after the live examination series.

Page 249: PHYSICS 9702/11 READ THESE INSTRUCTIONS FIRSTmaxpapers.com/wp-content/uploads/2012/11/9702_s17… ·  · 2017-10-17PHYSICS 9702/11 Paper 1 Multiple Choice May/June 2017 1 hour 15

This document consists of 8 printed pages.

DC (LK/SG) 127833/2© UCLES 2017 [Turn over

Cambridge International ExaminationsCambridge International Advanced Subsidiary and Advanced Level

*2509053205*

PHYSICS 9702/52Paper 5 Planning, Analysis and Evaluation May/June 2017 1 hour 15 minutesCandidates answer on the Question Paper.No Additional Materials are required.

READ THESE INSTRUCTIONS FIRST

Write your Centre number, candidate number and name on all the work you hand in.Write in dark blue or black pen.You may use an HB pencil for any diagrams or graphs.Do not use staples, paper clips, glue or correction fluid.DO NOT WRITE IN ANY BARCODES.

Answer all questions.

Electronic calculators may be used.You may lose marks if you do not show your working or if you do not use appropriate units.

At the end of the examination, fasten all your work securely together.The number of marks is given in brackets [ ] at the end of each question or part question.

Page 250: PHYSICS 9702/11 READ THESE INSTRUCTIONS FIRSTmaxpapers.com/wp-content/uploads/2012/11/9702_s17… ·  · 2017-10-17PHYSICS 9702/11 Paper 1 Multiple Choice May/June 2017 1 hour 15

2

9702/52/M/J/17© UCLES 2017

1 A student is investigating the motion of a small cube on a turntable connected to an electric motor as shown in Fig. 1.1.

turntable

cube

r

Fig. 1.1

The cube is placed at a distance r from the centre of the turntable. It is suggested that the relationship between r and the maximum frequency f of the turntable for which the cube does not move relative to the turntable is

K = 4π2mfr

where m is the mass of the cube and K is a constant.

Design a laboratory experiment to test the relationship between f and r. Explain how your results could be used to determine a value for K. You should draw a diagram, on page 3, showing the arrangement of your equipment. In your account you should pay particular attention to

• the procedure to be followed,• the measurements to be taken,• the control of variables,• the analysis of the data,• any safety precautions to be taken.

[15]

Page 251: PHYSICS 9702/11 READ THESE INSTRUCTIONS FIRSTmaxpapers.com/wp-content/uploads/2012/11/9702_s17… ·  · 2017-10-17PHYSICS 9702/11 Paper 1 Multiple Choice May/June 2017 1 hour 15

3

9702/52/M/J/17© UCLES 2017 [Turn over

Diagram

..................................................................................................................................................................

..................................................................................................................................................................

..................................................................................................................................................................

..................................................................................................................................................................

..................................................................................................................................................................

..................................................................................................................................................................

..................................................................................................................................................................

..................................................................................................................................................................

..................................................................................................................................................................

..................................................................................................................................................................

..................................................................................................................................................................

..................................................................................................................................................................

..................................................................................................................................................................

..................................................................................................................................................................

..................................................................................................................................................................

Page 252: PHYSICS 9702/11 READ THESE INSTRUCTIONS FIRSTmaxpapers.com/wp-content/uploads/2012/11/9702_s17… ·  · 2017-10-17PHYSICS 9702/11 Paper 1 Multiple Choice May/June 2017 1 hour 15

4

9702/52/M/J/17© UCLES 2017

..................................................................................................................................................................

..................................................................................................................................................................

..................................................................................................................................................................

..................................................................................................................................................................

..................................................................................................................................................................

..................................................................................................................................................................

..................................................................................................................................................................

..................................................................................................................................................................

..................................................................................................................................................................

..................................................................................................................................................................

..................................................................................................................................................................

..................................................................................................................................................................

..................................................................................................................................................................

..................................................................................................................................................................

..................................................................................................................................................................

..................................................................................................................................................................

..................................................................................................................................................................

..................................................................................................................................................................

..................................................................................................................................................................

..................................................................................................................................................................

..................................................................................................................................................................

..................................................................................................................................................................

..................................................................................................................................................................

..................................................................................................................................................................

..................................................................................................................................................................

..................................................................................................................................................................

..................................................................................................................................................................

[Total: 15]

Page 253: PHYSICS 9702/11 READ THESE INSTRUCTIONS FIRSTmaxpapers.com/wp-content/uploads/2012/11/9702_s17… ·  · 2017-10-17PHYSICS 9702/11 Paper 1 Multiple Choice May/June 2017 1 hour 15

5

9702/52/M/J/17© UCLES 2017 [Turn over

2 A student is investigating the current in a circuit. The circuit is set up as shown in Fig. 2.1.

A

E

P Q

I

Fig. 2.1

Two resistors P and Q are connected to a power supply of e.m.f. E and negligible internal resistance. The current I is measured.

The resistance of resistor P is P. The experiment is repeated for different values of P.

It is suggested that I and P are related by the equation

E = I(P + Q)

where Q is the resistance of resistor Q.

(a) A graph is plotted of 1I on the y-axis against P on the x-axis.

Determine expressions for the gradient and the y-intercept.

gradient = ...............................................................

y-intercept = ...............................................................[1]

Page 254: PHYSICS 9702/11 READ THESE INSTRUCTIONS FIRSTmaxpapers.com/wp-content/uploads/2012/11/9702_s17… ·  · 2017-10-17PHYSICS 9702/11 Paper 1 Multiple Choice May/June 2017 1 hour 15

6

9702/52/M/J/17© UCLES 2017

(b) Values of P and I are given in Fig. 2.2. The tolerance of each value of P is ±5%.

P / Ω I / mA1I / A–1

180 ± 34

220 ± 28

330 ± 19

470 ± 14

560 ± 12

680 ± 10

Fig. 2.2

Calculate and record values of 1I / A–1 in Fig. 2.2.

Determine the absolute uncertainties in P. [2]

(c) (i) Plot a graph of 1I / A–1 against P / Ω.

Include error bars for P. [2]

(ii) Draw the straight line of best fit and a worst acceptable straight line on your graph. Both lines should be clearly labelled. [2]

(iii) Determine the gradient of the line of best fit. Include the absolute uncertainty in your answer.

gradient = ...........................................................[2]

Page 255: PHYSICS 9702/11 READ THESE INSTRUCTIONS FIRSTmaxpapers.com/wp-content/uploads/2012/11/9702_s17… ·  · 2017-10-17PHYSICS 9702/11 Paper 1 Multiple Choice May/June 2017 1 hour 15

7

9702/52/M/J/17© UCLES 2017 [Turn over

P / Ω

10100 200 300 400 500 600 700 800

20

30

40

50

60

70

80

90

100

110

120

1I

/ A–1

Page 256: PHYSICS 9702/11 READ THESE INSTRUCTIONS FIRSTmaxpapers.com/wp-content/uploads/2012/11/9702_s17… ·  · 2017-10-17PHYSICS 9702/11 Paper 1 Multiple Choice May/June 2017 1 hour 15

8

9702/52/M/J/17© UCLES 2017

(iv) Determine the y-intercept of the line of best fit. Include the absolute uncertainty in your answer.

y-intercept = ...........................................................[2]

(d) (i) Using your answers to (a), (c)(iii) and (c)(iv), determine the values of E and Q. Include appropriate units.

E = ...............................................................

Q = ...............................................................[2]

(ii) Determine the percentage uncertainties in E and Q.

percentage uncertainty in E = ........................................................... %

percentage uncertainty in Q = ........................................................... %[2]

[Total: 15]

To avoid the issue of disclosure of answer-related information to candidates, all copyright acknowledgements are reproduced online in the Cambridge International Examinations Copyright Acknowledgements Booklet. This is produced for each series of examinations and is freely available to download at www.cie.org.uk after the live examination series.

Page 257: PHYSICS 9702/11 READ THESE INSTRUCTIONS FIRSTmaxpapers.com/wp-content/uploads/2012/11/9702_s17… ·  · 2017-10-17PHYSICS 9702/11 Paper 1 Multiple Choice May/June 2017 1 hour 15

*1039941446*

This document consists of 8 printed pages.

DC (LK) 142406© UCLES 2017 [Turn over

Cambridge International ExaminationsCambridge International Advanced Subsidiary and Advanced Level

PHYSICS 9702/53Paper 5 Planning, Analysis and Evaluation May/June 2017 1 hour 15 minutesCandidates answer on the Question Paper.No Additional Materials are required.

READ THESE INSTRUCTIONS FIRST

Write your Centre number, candidate number and name on all the work you hand in.Write in dark blue or black pen.You may use an HB pencil for any diagrams or graphs.Do not use staples, paper clips, glue or correction fluid.DO NOT WRITE IN ANY BARCODES.

Answer all questions.

Electronic calculators may be used. You may lose marks if you do not show your working or if you do not use appropriate units.

At the end of the examination, fasten all your work securely together.The number of marks is given in brackets [ ] at the end of each question or part question.

Page 258: PHYSICS 9702/11 READ THESE INSTRUCTIONS FIRSTmaxpapers.com/wp-content/uploads/2012/11/9702_s17… ·  · 2017-10-17PHYSICS 9702/11 Paper 1 Multiple Choice May/June 2017 1 hour 15

2

9702/53/M/J/17© UCLES 2017

1 A student is investigating the motion of a wooden block on an inclined plane, as shown in Fig. 1.1. A falling body causes the wooden block to accelerate.

θ

P

Q

pulley

falling bodywooden block

string

plane

Fig. 1.1

The wooden block is initially at rest at point P and has velocity v at point Q.

It is suggested that the relationship between v and the angle θ of the plane to the horizontal is

(B + m)v  22s = Bg – mg sin θ

where B is the mass of the falling body, m is the mass of the wooden block, s is the distance between P and Q and g is the acceleration of free fall.

Design a laboratory experiment to test the relationship between v and θ. Explain how your results could be used to determine a value for g. You should draw a diagram, on page 3, showing the arrangement of your equipment. In your account you should pay particular attention to

• the procedure to be followed, • the measurements to be taken, • the control of variables, • the analysis of the data, • any safety precautions to be taken. [15]

Page 259: PHYSICS 9702/11 READ THESE INSTRUCTIONS FIRSTmaxpapers.com/wp-content/uploads/2012/11/9702_s17… ·  · 2017-10-17PHYSICS 9702/11 Paper 1 Multiple Choice May/June 2017 1 hour 15

3

9702/53/M/J/17© UCLES 2017 [Turn over

Diagram

..................................................................................................................................................................

..................................................................................................................................................................

..................................................................................................................................................................

..................................................................................................................................................................

..................................................................................................................................................................

..................................................................................................................................................................

..................................................................................................................................................................

..................................................................................................................................................................

..................................................................................................................................................................

..................................................................................................................................................................

..................................................................................................................................................................

..................................................................................................................................................................

..................................................................................................................................................................

..................................................................................................................................................................

..................................................................................................................................................................

Page 260: PHYSICS 9702/11 READ THESE INSTRUCTIONS FIRSTmaxpapers.com/wp-content/uploads/2012/11/9702_s17… ·  · 2017-10-17PHYSICS 9702/11 Paper 1 Multiple Choice May/June 2017 1 hour 15

4

9702/53/M/J/17© UCLES 2017

..................................................................................................................................................................

..................................................................................................................................................................

..................................................................................................................................................................

..................................................................................................................................................................

..................................................................................................................................................................

..................................................................................................................................................................

..................................................................................................................................................................

..................................................................................................................................................................

..................................................................................................................................................................

..................................................................................................................................................................

..................................................................................................................................................................

..................................................................................................................................................................

..................................................................................................................................................................

..................................................................................................................................................................

..................................................................................................................................................................

..................................................................................................................................................................

..................................................................................................................................................................

..................................................................................................................................................................

..................................................................................................................................................................

..................................................................................................................................................................

..................................................................................................................................................................

..................................................................................................................................................................

..................................................................................................................................................................

..................................................................................................................................................................

..................................................................................................................................................................

..................................................................................................................................................................

..................................................................................................................................................................

[Total: 15]

Page 261: PHYSICS 9702/11 READ THESE INSTRUCTIONS FIRSTmaxpapers.com/wp-content/uploads/2012/11/9702_s17… ·  · 2017-10-17PHYSICS 9702/11 Paper 1 Multiple Choice May/June 2017 1 hour 15

5

9702/53/M/J/17© UCLES 2017 [Turn over

2 A student is investigating how the time for an electrical pulse to travel in a coaxial cable varies with the length of the cable. The pulse is reflected at one end of the cable. An oscilloscope is used to display the initial pulse and the reflected pulse.

The trace on the oscilloscope is shown in Fig. 2.1.

d

Fig. 2.1

The time t for the pulse to travel to the end of the cable and back is determined by measuring the distance d between the pulses on the screen, and then using the time-base and the relationship

t = d × time-base.

The initial length of the cable is L. A total length Z is removed from the cable and the experiment is repeated.

It is suggested that t and Z are related by the equation

v = 2 (L – Z   )t

where v is the speed of the pulse.

(a) A graph is plotted of t on the y-axis against Z on the x-axis.

Determine expressions for the gradient and the y-intercept.

gradient = ...............................................................

y-intercept = ............................................................... [1]

Page 262: PHYSICS 9702/11 READ THESE INSTRUCTIONS FIRSTmaxpapers.com/wp-content/uploads/2012/11/9702_s17… ·  · 2017-10-17PHYSICS 9702/11 Paper 1 Multiple Choice May/June 2017 1 hour 15

6

9702/53/M/J/17© UCLES 2017

(b) Values of Z and d are given in Fig. 2.2. The time-base is 0.1 µs cm–1.

Z / m d / cm t / µs

0.0 8.0 ± 0.1

4.0 7.7 ± 0.1

8.0 7.3 ± 0.1

12.0 7.0 ± 0.1

16.0 6.6 ± 0.1

20.0 6.2 ± 0.1

Fig. 2.2

Calculate and record values of t / µs in Fig. 2.2. Include the absolute uncertainties in t. [2]

(c) (i) Plot a graph of t / µs against Z / m. Include error bars for t. [2]

(ii) Draw the straight line of best fit and a worst acceptable straight line on your graph. Both lines should be clearly labelled. [2]

(iii) Determine the gradient of the line of best fit. Include the absolute uncertainty in your answer.

gradient = .......................................................... [2]

Page 263: PHYSICS 9702/11 READ THESE INSTRUCTIONS FIRSTmaxpapers.com/wp-content/uploads/2012/11/9702_s17… ·  · 2017-10-17PHYSICS 9702/11 Paper 1 Multiple Choice May/June 2017 1 hour 15

7

9702/53/M/J/17© UCLES 2017 [Turn over

Z / m

0.600 4 8 12 16 20 24

0.62

0.64

0.66

0.68

0.70

0.72

0.74

0.76

0.78

0.80

0.82

t / μs

Page 264: PHYSICS 9702/11 READ THESE INSTRUCTIONS FIRSTmaxpapers.com/wp-content/uploads/2012/11/9702_s17… ·  · 2017-10-17PHYSICS 9702/11 Paper 1 Multiple Choice May/June 2017 1 hour 15

8

9702/53/M/J/17© UCLES 2017

(iv) Determine the y-intercept of the line of best fit. Include the absolute uncertainty in your answer.

y-intercept = .......................................................... [2]

(d) (i) Using your answers to (a), (c)(iii) and (c)(iv), determine the values of L and v. Include appropriate units.

L = ...............................................................

v = ............................................................... [2]

(ii) Determine the percentage uncertainties in L and v.

percentage uncertainty in L = ........................................................... %

percentage uncertainty in v = ........................................................... %[2]

[Total: 15]

To avoid the issue of disclosure of answer-related information to candidates, all copyright acknowledgements are reproduced online in the Cambridge International Examinations Copyright Acknowledgements Booklet. This is produced for each series of examinations and is freely available to download at www.cie.org.uk after the live examination series.